Download as pdf or txt
Download as pdf or txt
You are on page 1of 172

P U P

Instructional Materials in

e
Us
MATH 20063
Elementary Differential Equations

l
cia
er
m

compiled by
om

DMS Faculty
C
or

College of Science
tF

Polytechnic University of the Philippines


No

2020
For the sole noncommercial use of the
Faculty of the Department of Mathematics and Statistics
Polytechnic University of the Philippines
2020

e
l Us
cia
er
m
om

Contributors:
C
or
tF

Atienza, Jacky Boy E.


Berico, Edwin O.
No

Bernardino, Rhea R.
Cabanig, Sarah Jean Q.
Costales, Jeffrey A.
Pelayo, Sharon Joy F.
Poloyapoy, Oscar L.
Publico, Juan Jr. L.
Torres, Aureluz L.
Zablan, Michael G.
Republic of the Philippines
POLYTECHNIC UNIVERSITY OF THE PHILIPPINES
COLLEGE OF SCIENCE
Department of Mathematics and Statistics

Course Title : ELEMENTARY DIFFERENTIAL EQUATIONS

Course Code : MATH 20063

Course Credit : 3 UNITS

Pre-Requisite : MATH 20053 Calculus 2

Course Description : This course is an introductory course in ordinary


differential equations (ODEs). It focuses primarily on

e
techniques for finding solutions to ODEs. Topics include

Us
the existence and uniqueness of solutions, first order
ordinary differential equations, linear differential
equations, linear equations with constant coefficients,
nonhomogeneous equations, undetermined coefficients

l
cia
and variation of parameters, This also involves solving
initial value problems using the Laplace transform and its
inverse, some applications of fist-order differential
er
equations, and system of first-order differential equations.
m
om

COURSE LEARNING PLAN


C

Week Dates Topics and Subtopics

10/05/20- • Definitions and Terminology


or

Week 1
10-11/20 • Elimination of Arbitrary Constants
tF

• Families of Curves
10/12/20-
Week 2 • Initial-Value Problems
No

10/18/20
• Direction Fields

10/19/20- • Separable Variables


Week 3
10/25/20 • Linear First-Order Equation
10/26/20- • Linear First-Order Equation
Week 4
11/01/20 • Exact Equations

11/02/20- • Differential Equations with Homogeneous


Week 5
11/08/20 Coefficients

11/09/20-
Week 6 • Other Methods for First-Order ODE
11/15/20

11/16/20-
Week 7 • Other Methods for First-Order ODE
11/22/20

11/23/20- • Homogeneous Linear Equations with Constant


Week 8
11/29/20 Coefficients

e
11/30/20-

Us
• Nonhomogeneous Higher Order Differential
Week 9
12/06/20 Equations

12/07/20-

l
cia
Week 10 • Variation of Parameters
12/13/20

12/14/20- • Definition of Laplace Transforms


er
Week 11
12/20/20 • Inverse Transforms
m

• Solving Linear ODE Using Transforms


om

01/04/21-
Week 12 • Translation Theorems and Additional Operational
01/10/21
Properties
C

01/11/21-
Week 13 • Applications of First Order Differential Equations
01/17/21
or

01/18/21-
tF

Week 14 • System of First-Order Differential Equations


01/24/21
No
COURSE GRADING SYSTEM

The final grade will be based on the weighted average of the student’s scores on
each test assigned at the end of each lesson. The final SIS grade equivalent will be
based on the following table according to the approved University Student
Handbook.

Class Standing (CS) = (((Weighted Average of all the Chapter/Unit Tests/Case


Study) x 50 )+ 50)

Midterm and/or Final Exam (MFE) = (((Weighted Average of the Midterm and/or
FinalTests) x 50)+50)

Final Grade = (70% x CS) + (30% x MFE)

SIS Grade Final Grade Equivalent Description

e
1.00 97.00-100 Excellent

Us
1.25 94.00-96.99 Excellent

l
1.50 91.00-93.99 Very Good

1.75
cia
88.00-90.99 Very Good
er
2.00 85.00-87.99 Good
m
2.25 82.00-84.99 Good
om

2.50 79.00-81.99 Satisfactory

2.75 77.00-78.99 Satisfactory


C

3.00 75.00-76.99 Passing


or

5.00 65.00-74.99 Failure


tF

INC Incomplete

W Withdrawn
No

Prepared by:

Rhea R. Bernardino
Faculty Member, Department of Mathematics and Statistics
College of Science
Contents

1 Introduction to Differential Equations


1.1 Definitions and Terminology…………………………………………….. 1
1.2 Elimination of Arbitrary Constants…………………………………….. 7
1.3 Families of Curves…………………………………………………………. 9
1.4 Initial-Value Problems……………………………………………………. 12
1.5 Direction Fields……………………………………………………………. 15
2 Some Techniques of Solving Ordinary Differential Equations
2.1 Separable Variables………………………………………………………. 17
2.2 Linear First-Order Equation……………………………………………. 24
2.3 Exact Equations…………………………………………………………… 33
2.4 Differential Equations with Homogeneous Coefficients……….……. 38
2.5 Other Methods for First-Order ODE…………………………………… 42

e
3 Higher-Order Differential Equations

Us
3.1 Homogeneous Linear Equations with Constant Coefficients………. 59
3.2 Nonhomogeneous Higher Order Differential Equations……………. 68
3.3 Variation of Parameters………………………………………………….. 76

l
4 The Laplace Transforms

cia
4.1 Definition of Laplace Transforms……………………………………….. 83
4.2 Inverse Transforms………………………………………………………… 88
er
4.3 Solving Linear ODE Using Transforms………………………………… 90
4.4 Translation Theorems and Additional Operational Properties…….. 93
m

5 Applications of First Order Differential Equations


om

5.1 Differential Equations of Plane Curves……………………………….... 101


5.2 Isogonal and Orthogonal Trajectories………………………………...… 104
5.3 Newton’s Law of Cooling (and Heating) ……………………………..… 109
C

5.4 Exponential Law of Growth and Decay………………………………… 114


.5 Simple Electric Circuits…………………………………………………..… 123
or

5.6 Mixture Problems………………………………………………………….. 129


tF

5.7 Velocity of Escape from the Earth………………….…………………… 134


5.8 Newton’s Second Law……………………………………………………… 137
5.9 Flow of Water Through an Orifice……………………………….……… 138
No

6 System of First-Order Differential Equations


6.1 System of Differential Equations……………………………………….. 139
6.2 Homogeneous Linear Systems with Constant Coefficients………… 146
6.3 Matrix Exponential………………………………………………………… 153
Unit Test No. 1
Classifications of Differential Equations, Elimination of Arbitrary Constants,
Families of Curves, Separation of Variables, Reducible to Separation of Vari-
ables

GENERAL INSTRUCTIONS
1. Work INDEPENDENTLY and HONESTLY.

2. Use permanent black or blue-inked pens only.

3. Write the questions, your answers and solutions in short white bond paper (8.5”
× 11”).

e
4. Write your FULL NAME, COURSE, YEAR, SECTION on the upper left part

Us
of each page of your answer sheets.

l
cia
A. COMPLETE THE TABLE. For each given differential equation, determine the dependent
variable(s) (DV) and independent variable(s) (IV), classify according to type (write only
ODE for ordinary differential equation and PDE for partial differential equation), order and
er
linearity (L for linear and NL for nonlinear differential equation) and give its degree.
m

Differential Equation DV IV Type Order Degree Linearity


om

dy
(1) = 4x2 y − y 2
dx
(2) 4xy 00 + 5y 0 − y sin x = 0
C

s   4 2
3
dy dy
or

(3) + 3y =
dx dx4
tF

∂y ∂w
(4) + =0
∂x ∂x
y(y 0 )4 − 5(y 00 )2 + 2 = 0
No

(5)
s  3
2
dy dy
(6) 2
= 5+
dx dx
(7) (2x − y)dx + (2x − 3y)dy = 0
d3 y dy
(8) sin θ 3
− cos θ
dθ dθ
B. SOLVING. Answer each of the following. Write your solution as neatly as possible.
Simplify your final answers by (a) converting logarithmic expressions into single logarithm,
if there are any (b) simplifying complex fractions.

1
1. By determinants, eliminate the arbitrary constants c1 and c2 : y = c1 e2x + c2 ex +
2
2. Eliminate the arbitrary constants c1 and c2 : y = c1 ex sin x + c2 ex cos x

3. Find the differential equation of the family of ellipses with center at the origin a the
and major axis on the y − axis.

4. Find the differential equation of the family circles entered at (h, k) passing through
the origin and (0, 4).

e
5. Solve: (e−y + 1)−2 ex dx + (e−x + 1)−3 ey dy = 0

Us
π
6. Solve: sin x cos2 y dx + cos2 x dy = 0, y(0) =
4

l
7. Solve: (2x − 2y − 1)dx − (x − y + 1)dy = 0

8. Find a continuous solution satisfying cia


er

m
dy  x if 0 ≤ x < 1
+ 2xy = f (x) where f (x) = , y(0) = 2
dx  0
om

if x ≥ 1
C
or
tF
No
Unit Test No. 2
Techniques of Solving Differential Equations

GENERAL INSTRUCTIONS
1. Work INDEPENDENTLY and HONESTLY.

2. Use permanent black or blue-inked pens only.

3. Write the questions, your answers and solutions in short white bond paper (8.5”
× 11”).

4. Write your FULL NAME, COURSE, YEAR, SECTION on the upper left part
of each page of your answer sheets.

e
Us
A. (Linear Differential Equations) Find the general (or particular) solution of the given
differential equations and give the largest interval over which the general solution is defined.

l
1. cos2 x sin x dy + (y cos3 x − 1)dx = 0
  cia
er
dy π
2. sin x + y cos x = x, y =2
dx 2
m

B. (Exact Equations) Find the general (or particular) solution of the given differential equa-
om

tions.
 
1. ex + 2xey + x2 dx + x2 ey + y 2 dy = 0
C

2. (4xy + 3x2 )dx + (2y + 2x2 )dy = 0, y(0) = −2


or

C. (Differential Equations with Homogenous Coefficients) Find the general (or particular)
tF

solution of the given differential equations.



1. 6x2 − 7y 2 dx − 14xydy = 0
No


2. x + yey/x dx − xey/x dy = 0, y(1) = 0

D. (Bernoulli Differential Equation) Find the general (or particular) solution of the given
differential equations.
dx x
1. + = −y 5 x9
dy y
dy 1
2. x2 − 2xy = 3y 4 , y(1) =
dx 2
Unit Test No. 3
Techniques of Solving Differential Equations, Higher-Order Homogeneous Linear
Equations with Constant Coefficients

GENERAL INSTRUCTIONS
1. Work INDEPENDENTLY and HONESTLY.

2. Use permanent black or blue-inked pens only.

3. Write the questions, your answers and solutions in short white bond paper (8.5”
× 11”).

4. Write your FULL NAME, COURSE, YEAR, SECTION on the upper left part

e
of each page of your answer sheets.

Us
A. Solve the following differential equations by using appropriate method or substitutions.

l
1.
dy
dx

= 2 + y − 2x + 3
cia
er
2. y 0 − (4x − y1 )2 = 0
m

3. (x + y − 1)dx + (y − x − 5)dy = 0
om

4. 6xydx + (4y + 9x2 )dy = 0


C

B. Determine the general solution for each given higher order differential equation. Write
your solutions as neatly as possible. Show computations for the roots of the auxiliary equa-
or

tion.
tF

1. y 000 − 2y 00 − 15y 0 = 0

2. y 00 + 4y 0 + 29y = 0
No

3. y 000 − 5y 00 + 3y 0 + 9y = 0
d4 y d2 y
4. 16 + 24 + 9y = 0
dx4 dx2
3 2 3
5. D2 + 9 D2 − 4D + 4 D2 − 7D − 18 y = 0
Unit Test No. 4
Higher-Order Nonhomogeneous Linear Equations with Constant Coefficients,
Laplace and Inverse Laplace Transforms
GENERAL INSTRUCTIONS
1. Work INDEPENDENTLY and HONESTLY.

2. Use permanent black or blue-inked pens only.

3. Write the questions, your answers and solutions in short white bond paper (8.5”
× 11”).

4. Write your FULL NAME, COURSE, YEAR, SECTION on the upper left part

e
of each page of your answer sheets.

Us
A. Find a linear differential operator that annihilates the given function.

l
1. 13x + 9x2 − sin 4x
2. 3 + ex cos 2x
cia
er
B. Determine the form of a particular solution for each differential equation.
1. y 00 − 2y 0 + y = 10e−2x cos x
m

2. y 000 − 4y 00 + 4y 0 = 5x2 − 6x + 4xe2x + 3e5x


om

C. Solve the given differential equation by undetermined coefficients.


1. y 00 + 6y 0 + 9y = −xe4x
C

2. y (4) − 2y 000 + y 00 = ex + 1
or

D. Solve the given differential equation by variations of parameters.


tF

1. y 00 + y = cos2 x
2. y 00 + 2y 0 + y = e−t ln t
No

E. Evaluate the following.


1. L{e3t + cos 6t − e3t cos 6t}
2. L{3 sinh 2t + 3 sin 2t}
 
−1 6s 3
3. L +
s2 + 25 s2 + 25
 
−1 3s − 2
4. L
2s2 − 6s − 2
 
−1 se−4s
5. L
(3s + 2)(s − 2)
F. Solve: y 00 − 6y 0 + 15y = 2 sin 3t, y(0) = −1, y 0 (0) = −4
Final Exam
GENERAL INSTRUCTIONS
1. Work INDEPENDENTLY and HONESTLY.

2. Use permanent black or blue-inked pens only.

3. Write the solutions (if necessary) and letter of choice in short white bond paper
(8.5” × 11”). If your answer is not among the choices, choose letter E.

4. Write your FULL NAME, COURSE, YEAR, SECTION on the upper left part
of each page of your answer sheets.

e
Us
Multiple Choice.

l
cia
1. Determine the order and degree of the differential equation
 2  4
d4 y d2 y
er
2
−2x + 4x − 1 = 0.
dx4 dx2
m
om

(a) order: 2, degree:4 (b) order: 4, degree:2


(c) order: 2, degree:2 (d) order: 4, degree:4
C

2. The equation y 2 = cx is the general solution of which of the following differential equa-
or

tion?
dy 2y dy y dy 2y dy y
tF

(a) = (b) = (c) =− (d) =−


dx x dx 2x dx x dx 2x
3. Find the equation of the curve at every point of which the tangent line has a slope of
No

4x.
(a) y = −2x2 + C (b) x = 2y 2 + C (c) x = −2y 2 + C (d) y = 2x2 + C

4. Which of the following represents the differential equation of the family of parabolas
having their vertices at the origin and their foci on the negative x−axis?
(a) 2xdy + ydx = 0 (b) xdy + ydx = 0 (c) 2xdy − ydx = 0 (d) 2ydx − xdy = 0

5. Which of the following equations is not a variable separable?


dy dy xy + 3x − y − 3
(a) = e3x+2y (b) =
dx dx xy − 2x + 4y − 8
 dy xy + 2y − x − 2
(c) 2ydx = x2 + 2xy + y 2 dx (d) =
dx xy − 3y + x − 3
6. Which of the following equations is an exact differential equation?

(a) x2 + 1 dx − xydy = 0 (b) xdy + (3x − 2y)dx = 0

(c) 2xdy + 2 + x2 dy = 0 (d) x2 ydy − ydx = 0

7. Which of the following is a differential equation with homogeneous coefficient?


 
p
(a) y − x + y dx − x2 dy = 0
2 2 2

(b) (x + 2y − 1)dx = (x + 2y − 3)dy


 
p
(c) y − x2 + y 2 dx − xdy = 0
      
y y y

e
(d) tan dy − tan + 1 dx = 0

Us
x x x
8. Determine the general solution of the the differential equation: (x + y)dy = (x − y)dx

l
(a) x2 − 2xy + y 2 = c (b) x2 + 2xy + y 2 = c
(c) x2 − 2xy − y 2 = c cia
(d) x2 + y 2 = c
er
For the next five questions, consider the following differential equation:
m


om

(y 2 + xy 3 )dx + 5y 2 − xy + y 3 sin y dy = 0
C

Let M = y 2 + xy 3 and N = 5y 2 − xy + y 3 sin y


∂M
or

9. =?
∂y
tF

(a) 2y + 3xy 2 (b) 2y − 3xy 2 (c) y 2 x + y 3 (d) y 2 x − y 3


No

∂N
10. =?
∂x
(a) y (b) −y (c) 5y 2 + y 3 sin y (d) 5y 2 − y + y 3 sin y
 
∂M ∂N
11. − =?
∂y ∂x
(a) 3y − 3xy 2 (b) y − 3xy 2 (c) 3y + 3xy 2 (d) y + 3xy 2
 
1 ∂M ∂N
12. − =?
M ∂y ∂x
3 3
(a) (b) − (c) 3y (d) −3y
y y

Page 2
13. The integrating factor for the given differential equation is
1
(a) y 3 (b) 3 (c) 3y (d) −3y
y
14. Radium decomposes at a rate proportional to the amount at any instant. In 100 years,
100 mg of radium decomposes to 96mg. How many mg will be left after 200 years?
(a) 88.60 (b) 95.32 (c) 92.16 (d) 90.72

15. Radium decomposes at a rate proportional to the amount present. If half of the original
amount disappears after 1000 years, what is the (approximate) percentage lost in 100
years?
(a) 6.70% (b) 4.50% (c) 5.36% (d) 4.30%

e
Us
16. According to Newton?s Law of Cooling, the rate at which a substance cools in air is
directly proportional to the difference between the temperature of the substance and

l
that of air. If the temperature of the air is 30o and the substance cools from 100o to 70o

cia
in 15 minutes, approximately, how long will it take to cool 100o to 50o ?
er
(a) 33.58 minutes (b) 43.50 minutes (c) 35.39 minutes (d) 45.30 minutes
m
17. Determine the equation of the family of orthogonal trajectories of the the family of
om

curves defined by y 2 = 2x + c
(a) y = ce−x (b) y = ce2x (c) y = ce−2x (d) y = cex
C

18. Determine the roots of the auxiliary equation of y 000 + 3y 00 − 4y 0 − 12y = 0


or

(a) −3, −2, 2 (b) −3, −2, 3 (c) 0, 2, −2 (d) −3, 2, 3


tF

19. Evaluate W (1 + x, x, x2 )
No

(a) −2 (b) −2x (c) 2x (d) 2

20. Which of the following best represents the particular solution of the differential equation:

y 00 − 6y 0 + 9y = 6x2 + 2 − 12e3x ?

(a) yp = Ax2 + Bx + C + Ee3x (b) yp = Ax2 + Bx + C + Ex2 e3x


(c) yp = Ax2 + Bx + C + Exe3x (d) yp = Ax2 + Bx + C + Ex3 e3x

21. The complementary solution, yc , of the equation y 000 + 2y 00 + y 0 = 10 is


(a) yc = c1 e−x + c2 e−x + c3 (b) yc = c1 ex + c2 xex + c3
(c) yc = c1 e−x + c2 ex + c3 (d) yc = c1 e−x + c2 xe−x + c3

Page 3
22. Solve: y 000 + 2y 00 + y 0 = 10
(a) y = c1 + c2 e−x + c3 xe−x + 10x2 (b) y = c1 + c2 e−x + c3 xe−x + 10x
(c) y = c1 x + c2 e−x + c3 xe−x + 10x (d) y = c1 + c2 e−x + c3 xe−x + 10
 
4s
23. Evaluate L
4s2 + 1
t t
(a) cos (b) sin (c) cos 2t (d) sin 2t
2 2
24. Evaluate L{te2t sin 6t}
12s − 24 12s − 24 12s − 24 12s − 24
(a) 2 2
(b) 2
(c) (d)
[(s − 2) + 36] [(s − 2) + 36] [(s − 2)2 + 6]2 [(s + 2)2 + 36]2

e
 

Us
−1 2s + 5
25. Evaluate L
(s − 3)2

l
(a) e−t + e−3t (b) 2e−t + 11e−3t (c) 2e−t + 11te−3t (d) e−t + 11te−3t

cia
er
m
C om
or
tF
No

Page 4
1

Chapter 1: Introduction to Differential Equa-


tions
Chapter Overview
This chapter focused on the discussion of some definitions and concepts in differential
equations as well as its classifications. Differential equations representing families of
curves and an introduction to initial value problem are also included in this chapter.

Learning Outcomes
At the end of this chapter, the student is expected to

e
1. define a differential equation

Us
2. classify a differential equation according to its type, order, degree and linearity

l
3. obtain a differential equation by eliminating arbitrary constants

cia
er
1.1 Definitions and Terminology
m

Definition 1: Differential Equation


om

An equation containing the derivatives of one or more dependent variables, with respect
to one or more independent variables, is said to be a differential equation (DE).
C
or

A differential equation can be classified according to its type, order, and linearity.
tF

I. Classification by Type
No

An equation involving only ordinary derivatives of one or more dependent variables with
respect to a single independent variable it is said to be an ordinary differential equation
(ODE).

Example 1.
dy d3 y d2 y dx dy
+ 5y = e2x , 3
− 2
+ 5y = 0, and + = ex + y are ordinary differential
dx dx dx dt dt
equations.

An equation involving partial derivatives of one or more dependent variables of two or more
independent variables is called a partial differential equation (PDE).

Example 2.
∂ 2u ∂ 2u ∂ 2u ∂ 2u ∂u ∂u ∂v
+ = 0, = − 2 , and = − are partial differential equations.
∂x2 ∂y 2 ∂x2 ∂t2 ∂t ∂y ∂x
2

Notations:
dy d2 y d3 y dn y
Throughout the discussion, we will use the Leibniz notation , , , . . . , ;
dx dx2 dx3 dxn
or the prime notation y 0 , y 00 , y 000 , y (4) , y (5) , . . . , y (n) to denote the order of the derivative.

II. Classification by Order

The order of a differential equation (either ODE or PDE) is the order of the highest
derivative in the equation.

Example 3.
d3 y d2 y
1. − + 5y = 0 is a third-order ordinary differential equation
dx3 dx2

e
Us
∂ 2u ∂ 2u ∂u
2. 2
= 2
−2 is a second-order partial differential equation
∂x ∂t ∂t
 3

l
d2 y dy

cia
3. 2
+ 3 − 5y = ex+1 is a second-order ordinary differential equation
dx dx
er
We can express an nth-order ordinary differential equation in one dependent variable by the
general form 
m
F x, y, y 0 , y 00 , . . . , y (n) = 0 (1)
om

dn y
where F is a real-valued function of n + 2 variables: x, y, y 0 , . . . , y (n) and y (n) = . In
dxn
this case, we can write
C

dn y 
= f x, y, y 0 , . . . , y (n−1) (2)
or

dx n
tF

where f is a real-valued continuous function and for which, equation (2) is called the nor-
mal form of equation (1).
No

The degree of a DE is the degree of the highest derivative occurring in it after the equation
has been free from radicals or rational exponents as far as the derivatives are concerned.

III. Classification by Linearity



An nth-order ordinary differential equation F x, y, y 0 , y 00 , . . . , y (n) = 0 is said to be linear
if F is linear in y, y 0 , y 00 , . . . , y (n) . That is, an nth-order ODE is linear when

F x, y, y 0 , y 00 , . . . , y (n) = 0

is written as an (x)y (n) + an−1 (x)y (n−1) + · · · + a1 (x)y 0 + a0 (x)y − g(x) = 0 or

dn y dn−1 y dy
an (x) n
+ a n−1 (x) n−1
+ · · · + a1 (x) + a0 (x)y = g(x).
dx dx dx
3

dy
If n = 1, then a1 (x) + a0 (x)y = g(x) is called linear first-order ODE.
dx
d2 y dy
If n = 2, then a2 (x) 2 + a1 (x) + a0 (x)y = g(x) is called linear second-order ODE.
dx dx
A nonlinear ordinary differential equation is simply one that is not linear.

Remarks 1. Characteristics of a linear ODE:


1. The dependent variable y and all its derivatives: y 0 , y 00 , . . . , y (n) are of the first degree,
that is, the power of each term involving y is 1.
2. The coefficients a0 , a1 , . . . , an of y, y 0 , . . . , y (n) depend at most on the independent vari-
able x.

e
Us
Example 4.
1. (y − x) dx + 4x dy = 0 is a linear first-order ODE

l
2. y 00 − 2y 0 + y = 0 is a linear second-order ODE
d3 y dy cia
er
3. 3
+x − 5y = ex is a linear third-order ODE
dx dx
m
4. (1 − y)y 0 + 2y = ex is a nonlinear first-order ODE since (1 − y)y 0 is a nonlinear term
om

with coefficient that depends on y.


d2 y
5. + sin y = 0 is a nonlinear second-order ODE since sin y is a nonlinear function of
C

dx2
y
or

d4 y
6. + y 3 = 0 is a nonlinear fourth-order ODE since y 3 is a nonlinear function of y
dx4
tF

(degree of y is not equal to 1).


No

Definition 2: Solution of an ODE


Any function φ, defined on an interval I and possessing at least n derivatives that
are continuous on I, which when substituted into an nth-order ordinary differential
equation reduces the equation to an identity, is said to be a solution of the equation
on the interval.

In other words, a solution of an nth-order ordinary differential equation (1) is a function φ


that possesses at least n derivatives and for which

F x, φ(x), φ0 (x), φ00 (x), . . . , φ(n) (x) = 0 for all x ∈ I.

For our discussion, we assume that φ is a real-valued function. We can also denote a solution
by the symbol y(x).
4

The interval I in Definition 2 is called as the interval of definition, the interval of ex-
istence, the interval of validity, or the domain of the solution and can be an open
interval, a closed interval, an infinite interval, and so on.

Example 5. Verify that the indicated function is a solution of the given differential equation
on the interval (−∞, ∞).
dy √ 1
1. − x y = 0 ; y(x) = x4
dx 16
Solution:   r !
dy √ d 1 4 1 4
− x y = 0 ⇐⇒ x −x x =0
dx dx 16 16

e
 

Us
1 3 1 2
⇐⇒ x − x x =0
4 4
1 1
⇐⇒ x3 − x3 = 0

l
cia
4 4
2. y 00 − 2y 0 + y = 0 ; y(x) = xex
er
Solution: Note that y 0 = xex + ex and y 00 = xex + 2ex . Then
m

y 00 − 2y 0 + y = 0 ⇐⇒ (xex + 2ex ) − 2(xex + ex ) + xex = 0


om

⇐⇒ xex + 2ex − 2xex − 2ex + xex = 0.


C

Notice that the constant solution y = 0 is a solution for both the differential equations
dy √
− x y = 0 and y 00 − 2y 0 + y = 0. A solution of a differential equation which is identically
or

dx
zero on an interval I is said to be a trivial solution.
tF

The graph of a solution of an ordinary differential equation isa called a solution curve.
No

It is mentioned earlier that a solution φ of an ODE is differentiable, thus it is continuous


on its interval I of definition. In means that there may be a difference between the graph
of a function φ and the graph of a solution φ which can also be stated as, the domain of a
function φ is not necessarily the same as the domain of the solution φ.

Definition 3: Implicit Solution of an ODE

A relation G(x, y) = 0 isa said to be an implicit solution of an ordinary differential


equation on an interval I, provided there exists at least one function φ that satisfies
the relation as well as the differential equation on I.
5

Example 6. Verify that the relation x2 + y 2 = 9 is an implicit solution of the differential


dy x
equation = − on the interval −3 < x < 3.
dx y
Solution: By implicit differentiation,

x2 + y 2 = 9
dy
2x + 2y =0
dx
dy x
=−
dx y
The graph on the right represents the implicit
solution x2 + y 2 = 9. Moreover, solving y√in
terms of x from x2 + y 2 = 9√gives y = ± 9 − x2

e
2
√ two functions y = 9 − x and

Us
and the
y = − 9 − x2 are both explicit solutions defined
on the interval −3 < x < 3. The graphs of these

l
explicit solutions are shown in Figure 1.

cia
er
m
C om
or
tF
No

√ √
Figure 1: explicit solutions: y = 9 − x2 and y = − 9 − x2

When solving for a first-order differential equation F (x, y, y) = 0, a solution containing an


arbitrary constant c is obtained. A solution containing an arbitrary constant represents a
set G(x, y, c) = 0 of solutions called one-parameter family of solutions.

In general, when solving for an nth-order differential equation F (x, y, y 0 , ..., y n ), we are to
determine an n-parameter family of solutions G(x, y, c1 , c2 , ..., cn ) = 0. This means that
a differential equation can have an infinite number of solutions corresponding to the unlim-
ited number of choices for the parameters.
6

A solution of a differential equation that is free from arbitrary parameters is referred to as


particular solution. There are instances that a differential equation has a solution that is
not a member of a family of solutions of the equation, such solution that can not be obtained
by specializing any of the parameters in the family of solutions is called a singular solution.

If every solution of an nth -order differential equation F (x, y, y 0 , ..., y n ) = 0 on an interval


I can be determined from an n-parameter family G(x, y, c1 , c2 , ..., cn ) = 0 by appropriate
choices of parameters c1 , c2 , ..., cn , then that family is the general solution of the differen-
tial equation.

Almost all of the examples presented previously in this lesson have used x and y to denote
the independent and dependent variables, respectively. We should consider that we could
also denote these variables with other symbols. For example, we could have a differential

e
d2 x

Us
equation 2 + 16x = 0 where the dependent variable is x and the independent variable is t.
dt
Activity 1

l
A. For each differential equation, give the dependent and independent variable(s), state

cia
whether the equation is ordinary or partial, linear or nonlinear and give also its order and
degree. Note: DV: dependent variable(s), IV: Independent Variables
er
Differential Equation DV IV Type Order Degree Linearity
m
dy
(1) = 3x − 2
om

dx
dy
(2) cos x + (sin x)y = 1
dx
C

∂ 2V ∂ 2V ∂ 2V
(3) + + =0
∂x2 ∂y 2 ∂z 2
or

(4) y” − 4y 0 + 4y = (x + 1)e2x
tF

∂ 2u ∂ 2u ∂ 2u
(5) + 4 + 5 =0
∂x2 ∂x∂t ∂t2
s
No

 3
d2 y dy
(6) 2
= 5+
dx dx
(7) (2x − y)dx + (2x − 3y)dy = 0
d3 y dy
(8) sin θ 3
− cos θ
dθ dθ

B. Verify that the indicated family of functions/expression is a solution of the given differ-
ential equation.
c1 et dP
1. P 0 = P (1 − P ); P = t
, where P 0 =
1 + c1 e dt
2. y 00 − 4y 0 + 4y = 0; y = c1 e2x + c2 xe2x
7

1.2 Elimination of Arbitrary Constants


From the previous lesson, we have already stated that when solving for an nth-order differ-
ential equation F (x, y, y 0 , ..., y n ), we are actually to determine an n-parameter family of
solutions G(x, y, c1 , c2 , ..., cn ) = 0. In this lesson, what we are going to do is to determine
the differential equation whose family of solutions is the given n-parameter family of
solutions G(x, y, c1 , c2 , ..., cn ) = 0. To be able to this, we are going to use differentiation
techniques to eliminate the arbitrary constants in the given n-parameter family of solutions

Consider the following in eliminating the arbitrary constants.

1. the number of times of differentiating the given equation is the same as the number of
arbitrary constants to be eliminated

e
Us
2. the order of the differential equation is the same as the number of arbitrary constants
in the equation

3. the obtained equation is free from arbitrary constant

l
cia
Example 7. Eliminate the arbitrary constants in each equation and express the final answers
in the following form: an (x)y (n) + an−1 (x)y (n−1) + · · · + a1 (x)y 0 + a0 (x)y − g(x) = 0 .
er
m
1. y = cx2
y
Solution: Since there is only one arbitrary constant to eliminate, we write c = 2
om

x
and then differentiate both sides with respect to x.
y
C

c=
x2
x2 y 0 − y(2x)
or

0=
x4
tF

1 2 2y
T hus, 2 y 0 − 3 y = 0 or y 0 =
x x x
No

2y
This actually means that a one-parameter family of solutions of y 0 = is of the form
x
y = cx2 .

For the next chapter, our goal is to solve some differential equations.

2. y = c1 e2x + c2 e−3x
Solution:
Differentiate the given up to second order. Add 3 times equation (1) and equation (2)
y = c1 e2x + c2 e−3x (1) 3y = 3c1 e2x + 3c2 e−3x
y 0 = 2c1 e2x − 3c2 e −3x
(2) + y0 = 2c1 e2x − 3c2 e−3x
y 00 = 4c1 e2x + 9c2 e −3x
(3) 0
y + 3y = 5c1 e2x (4)
8

Add 3 times equation (2) and equation (3) Add −2 times equation (4) and equation
3y 0 = 6c1 e2x − 9c2 e−3x −2y 0 − 6y = −10c1 e2x
+ y 00 = 4c1 e2x + 9c2 e−3x (5) + y 00 + 3y 0 = 10c1 e2x
00 0 2x 00 0
y + 3y = 10c1 e (5) y + y − 6y = 0

Therefore, the desired differential equation is y 00 + y 0 − 6y = 0.

Alternative Solution:
Suppose that each of the functions f1 (x), f2 (x),...,fn (x) possesses at least n − 1 deriva-
tives. The determinant

f1 (x) f2 (x) ··· fn (x)


f10 (x) f20 (x) ··· fn0 (x)
.. .. ..

e
. . ··· .

Us
(n−1) (n−1)
f1 (x) f2 (x) · · · fn (n−1)

where primes denote derivatives, is called the Wronskian of the functions.

l
cia
Given y = c1 e2x + c2 e−3x , let f1 (x) = y, f2 (x) = c1 e2x and f3 (x) = c2 e−3x . Equate then
the determinant to 0. That is,
er
y c1 e2x c2 e−3x
m
0
y 2c1 e 2x
−3c2 e−3x = 0
y 00 4c1 e2x 9c2 e−3x
om

Assuming c1 , c2 6= 0, by some properties of determinants and since e2x , e−3x 6= 0 for all
C

x in R, we can write
or

y 1 1 y 1 1
2x
c1 e c2 e−3x
y 2 −3 = 0 =⇒ y 0 2 −3 = 0
0
tF

y 00 4 9 y 00 4 9

We now have, (18y − 3y 00 + 4y 0 ) − (2y 00 − 12y + 9y 0 ) = 0 =⇒ y 00 + y 0 − 6y = 0.


No

3. y = Aex + Bxex , where A and B are the parameters to be eliminated.


Solution:
y = Aex + Bxex (1)
0
y = Ae + Bxe + Be , but y = Aex + Bxex
x x x

=⇒ y 0 = y + Bex (2)
00 0 x
y = y + Be (3)
Subtracting equation (3) and equation (2), we
will have,
Therefore, the differential equation is
y 00 = y0 + Bex y 00 − 2y 0 + y = 0.
− (y 0 = y + Bex )
y − y0
00
= y0 − y
9

4. y = c1 sin kt + c2 cos kt where k is a parameter not to. be eliminated.


Solution:
y = c1 sin kt + c2 cos kt (1)
0
y = kc1 cos kt − kc2 sin kt (2)
00 2 2
y = −k c1 sin kt − k c2 cos kt (3)
Add k 2 times equation (1) and equation (2)

k2y = k 2 c1 sin kt + k 2 c2 cos kt


+ y 00 = −k 2 c1 sin kt − k 2 c2 cos kt
∴ y0 + k2y = 0

1.3 Families of Curves

e
Us
A solution of a differential equation is sometimes referred to as an integral of the equation
and its graph is called an integral curve.

l
cia
Example 8. At any point (x, y) on a curve, the tangent line has a slope equal to 2x. De-
termine the equation of the curve.
Solution:
er
Let y = f (x) be the equation of the curve. The slope of the tangent line mT L at a point
m
(x, y) on the graph of the curve is given by f 0 (x) = 2x. Then we have,
Z
om

f (x) = 2xdx

f (x) = x2 + C, where C is an arbitrary constant


C

The curves represented by f (x) = x2 + C are actually parabolas opening upwards whose
or

dy
vertex lies on the y − axis and whose differential equation is given by = 2x.
tF

dx
An n-parameter family of solutions of an nth-order differential equation can be represented
graphically by a family of curves. In this section, we are going to determine a differential
No

equation of a family of curves given certain conditions on the curves, by eliminating arbitrary
constants.

Example 9. Determine the differential equation of the family of curves described in each
of the following.

1. straight lines with equal slope and y-intercept


Solution:
Consider the slope-intercept form of a line, y = mx + b. From the given condition,
the slope and y-intercept are equal, that is m = b. The equation representing the
straight lines with equal slope and y-intercept now becomes y = mx + m, where m is
the arbitrary constant to be eliminated.
Differentiating y = mx + m with respect to x gives us y 0 = m. Replace y 0 = m in
y = mx + m. We now have, y = y 0 x + y 0 or (x + 1)y 0 − y = 0.
10

Figure 2: some solutions of (x + 1)y 0 − y = 0

e
Us
2. circles with center on the x-axis
Solution:

l
cia
Standard equation of a circle centred at (h, k) with radius r: (x − h)2 + (y − k)2 = r2
Given condition: center of the circle on the x-axis, that is, k = 0
Working equation: (x − h)2 + y 2 = r2 where h and r are the arbitrary constants to be
er
eliminated
m
om

Since there are two arbitrary constants to be


eliminated, determine the second derivative of
(x − h)2 + y 2 = r2 with respect to x.
C

(x − h)2 + y 2 = r2
or

2(x − h) + 2yy 0 = 0
tF

or x − h + yy 0 = 0
1 + y(y 00 ) + y 0 (y 0 ) = 0
No

Since the second derivative is now free from any arbitrary constant, then we have the
differential equation, y(y 00 ) + (y 0 )2 + 1 = 0. The circles illustrated above are some
solutions of the differential equation y(y 00 ) + (y 0 )2 + 1 = 0.

3. parabolas with vertex at (1, 3) and axis of symmetry parallel to the x − axis
Solution:
Since the axis of symmetry is parallel to the x − axis, then the parabolas are either
opening to the right or left.
Standard equation of parabolas with horizontal axis of symmetry: (y−k)2 = ±4a(x−h)
Given condition: vertex (h, k) at (1, 3)
Working equation: (y − 3)2 = ±4a(x − 1), where a is the arbitrary constant to be
eliminated
11

Since we only have one arbitrary constant to eliminate, we differentiate (y − 3)2 =


±4a(x − 1) once with respect to x.
(y − 3)2 = ±4a(x − 1)
2(y − 3)y 0 = ±4a
By substitution, we have (y − 3)2 = 2(y − 3)y 0 (x − 1) =⇒ 2(x − 1)y 0 − y + 3 = 0.

4. ellipses with center at the origin and major axis is on the x − axis
Solution:
Standard equation of an ellipse with center at the origin and major axis on the x−axis:
x2 y 2

e
+ 2 =1
a2

Us
b
2 2 2 2 2 2
which can be written as b x + a y = a b
From this equation, there are two arbitrary constants to eliminate, a and b, so we

l
determine up to the second order derivative.

cia
Differentiate b2 x2 + a2 y 2 = a2 b2 with respect to x: 2b2 x + 2a2 yy 0 = 0
er
yy 0 b2
m
Isolate the constants: =− 2
x a
om

x[y(y 00 ) + y 0 (y 0 )] − yy 0 (1)
Differentiate with respect to x: =0
x2
C

Simplifying the resulting differential equation: xyy 00 + x(y 0 )2 − yy 0 = 0


Activity 2
or
tF

A. Eliminate the arbitrary constants in each equation and express the final answers in the
following form: an (x)y (n) + an−1 (x)y (n−1) + · · · + a1 (x)y 0 + a0 (x)y − g(x) = 0 .
No

1. x3 − y 2 = cy 4. y = c1 e2x + c2 ex

2. y = c(x2 − 4x + 1) 5. y = c1 ex sin x + c2 ex cos x

3. y = c1 x + c2 x2 + sin x 6. y = c1 ex + c2 e2x + c3 e3x


B. Find the differential equation of the family of curves described in each of the following.
1. Straight lines with slope and x − intercept equal.
2. Circles with fixed radius r and tangent to the x − axis.
3. Parabolas whose vertex is at the origin and focus on the x − axis.
4. Ellipses center at the origin where the major axis on the y − axis.
5. Circles with renters on the y − axis.
12

1.4 Initial-Value Problems


We are often interested in problems in which we seek a solution y(x) of a differential equation
so that y(x) satisfies prescribed side conditions − that is, conditions imposed on the unknown
y(x) or its derivatives. On some interval I containing x0 the problem:
dn y 0 (n−1)

Solve: = f x, y, y , . . . , y
dxn
Subject to: y(x0 ) = y0 , y 0 (x0 ) = y1 , y 00 (x0 ) = y2 , . . . , y (n−1) (x0 ) = yn−1 ,
where y0 , y1 , y2 , . . . , yn−1 are arbitrary specified real constants, is called an initial-value
problem (IVP). The values of y(x) and its first n−1 derivatives at a single point x0 , y(x0 ) =
y0 , y 0 (x0 ) = y1 , y 00 (x0 ) = y2 , . . . , y (n−1) (x0 ) = yn−1 , are called initial conditions.
The problem given above is also called an nth-order initial-value problem. The first and

e
second-order initial-value problems are given as follows:

Us
dy
Solve: = f (x, y)
dx

l
Subject to: y(x0 ) = y0
and
d2 y cia
= f (x, y, y 0 )
er
Solve:
dx2
y(x0 ) = y0 , y 0 (x0 ) = y1 .
m
Subject to:
For a first order IVP, we are actually to determine a solution of the differential equation on an
om

interval I which contains x0 so that a solution curve will pass through the given point (x0 , y0 ).
C

For a second-order IVP, we want to find a solution of the differential equation whose graph
passes through (x0 , y0 ) so that the slope of the curve at this point is y1 .
or

Example 10. We can easily verify that y = cex is a one-parameter family of solutions of
tF

the DE y 0 = y on (−∞, ∞). If we specify an initial condition, say y(0) = 3, then solving for
c gives us c = 3. Hence, the function y = 3ex is a solution of the IVP y 0 = y, y(0) = 3.
No

Example 11. Given that x = c1 cos t + c2 sin t is a two-parameter family of solutions of the
differential equation x00 + x = 0, find a solution of the IVP
   
00 π 1 0 π
x + x = 0, x = ,x = 0.
6 2 6
Solution:
t, then x0 = −c1 sin t + c2 cos t.
Given that x = c1 cos t + c2sin 
π 1
Using the first condition, x = , in x = c1 cos t + c2 sin t,
6 2
   
1 π π
= c1 cos + c2 sin
2 6 6
√   
1 3 1
= c1 + c2 (1)
2 2 2
13
 
π
0
Apply now the second condition x = 0 in x0 = −c1 sin t + c2 cos t
6
   
π π
0 = −c1 sin + c2 cos
6 6
  √ 
1 3
0 = −c1 + c2 (2)
2 2

3
Solving simultaneously for c1 and c2 from equation (1) and equation (2) gives c1 = and
√ 4
1 3 1
c2 = . Therefore, x = cos t + sin t is a solution of the given IVP.
4 4 4

e
Take note of the use of the phrase a solution instead of the solution. This only means that

Us
there is a possibility that other solutions may exist.

l
Example 12. Without determining a one-parameter family of solutions of the equation

cia
y 0 = 3y 2/3 , we can still verify that the functions y = 0 and y = x3 both satisfy the IVP,
dy
er
= 3y 2/3 , y(0) = 0.
dx
m

Theorem 1: Existence of a Unique Solution


om

Let R be a rectangular region in the xy-plane defined by a ≤ x ≤ b, c ≤ y ≤ d that


∂f
C

contains the point (x0 , y0 ) in its interior. If f (x, y) and are continuous on R then
∂y
there exist some interval I0 : x0 − h < x < x0 + h, h > 0, , contained in a ≤ x ≤ b and
or

a unique function y(x) defined on I0 , that is a solution of the initial value problem
tF

dy
Solve: = f (x, y)
dx
No

Subject to: y(x0 ) = y0

dy
Example 13. From example 12, we have seen that the differential equation = 3y 2/3 has
dx
at least two solutions whose graphs pass through the point (0, 0). Consider the functions

∂f 2
f (x, y) = 3y 2/3 and = 1/3 .
∂y y
This shows that they are continuous in the half-planes defined by either y > 0 or y < 0.
Hence by Theorem 1, we can conclude that through any point (x0 , y0 ), y0 > 0ory0 < 0, there
is some interval centred at x0 on which the given differential equation has a unique solution.
14

Example 14. The existence of a unique solution guarantees that there are no other solutions
of the IVP y 0 = y, y(0) = 3 other than y = 3ex . This follows from the fact that f (x, y) = y
∂f
and = 1 are continuous through out the xy plane.
∂y

Remarks 2. 1. Suppose that y(x) is a solution of an initial value problem. Then the
following sets may not be the same: domain of the function y(x), interval I over which
the solution y(x) is defined or exists, and the interval I0 of existence and uniqueness.

2. Theorem 1 does not give any indication of the sizes of the intervals I and I0 . The
interval of definition need not be as wide as the region R, and the interval I0 of existence
and uniqueness may not be as large as I.

3. If the conditions stated in the hypothesis of Theorem 1 do not hold, then the IVP may

e
still have a unique solution, several solutions or no solution at all.

Us
Activity 3

l
cia
1
For problems 1 and 2, use the the given that y = is a one-parameter family of
1 + c1 e−x
solutions of y 0 = y − y 2 to find a solution of the initial value problem consisting of the given
er
differential equation and the indicated initial conditions.
m

1 2. y(−1) = 2
1. y(0) = −
om

3
For problems 3 and 4, use the the given that x = c1 cos t + c2 sin t is a two-parameter family
C

dx
of solutions of x00 + x = 0, where x0 = , to find a solution of the initial value problem
dt
or

consisting of the given differential equation and the indicated initial conditions.
   
tF

3. x(0) = −1, x0 (0) = 8 π √ 0 π



4. x = 2, x =2 2
4 4
No

For problems 5 and 6, use the the given that y = c1 ex + c2 e−x is a two-parameter family of
solutions of y 00 − y = 0 to find a solution of the initial value problem consisting of the given
differential equation and the indicated initial conditions.

5. y(−1) = 5, y 0 (−1) = −5 6. y(0) = 1, y 0 (0) = 2

For numbers 7 to 10, determine a region of the xy-plane for which the given differential
equation would have a unique solution whose graph passes through a point (x0 , y0 ) in the
region.

dy 9. 4 − y 2 y 0 = x2
7. = y 2/3
dx
dy 
8. x =y 10. x2 + y 2 y 0 = y 2
dx
15

1.5 Direction Fields


If we systematically evaluate f over a rectangular grid of points in the xy−plane and draw
a lineal element at each point (x, y) of the grid with slope f (x, y) , then the collection of all
these lineal elements is called a direction field or slope field of the differential equation
dy
= f (x, y).
dx
dy
Recall that one if > 0 for all x in an interval I, then a differentiable function y = y(x) is
dx
dy
increasing on I. Similarly, if < 0 for all x in an interval I, then a differentiable function
dx
y = y(x) is decreasing on I.

e
Example 15. Sketch the direction field for the following differential equation. Sketch the

Us
set of integral curves for this differential equation.
dy

l
=y−x

cia
dx
Solution: To sketch direction fields for this kind of differential equation we first identify
er
places where the derivative will be constant. To do this we set the derivative in the differential
equation equal to a constant, say c. This gives us a family of equations, called isoclines,
m
that we can plot and on each of these curves the derivative will be a constant value of c. We
om

set c = y − x. Sketch some curves for several values of c.


C
or
tF
No

Figure 3:

Now, on each of these lines, or isoclines, the derivative will be constant and will have a value
of c. On the c = 0 isocline the derivative will always have a value of zero and hence the
tangents will all be horizontal. On the c = 1 isocline the tangents will always have a slope
16

of 1, on the c = −2 isocline the tangents will always have a slope of −2, and so on. Below
are a few tangents put in for each of these isoclines.

e
l Us
Figure 4:cia
er
m
To add more arrows for those areas between the isoclines start at say, c = 0 and move up to
c = 1 and as we do that we increase the slope of the arrows (tangents) from 0 to 1.
om

We can then add in integral curves for this differential equation as shown below.
C
or
tF
No

Figure 5:
17

Chapter 2: Some Techniques of Solving Ordi-


nary Differential Equations
Chapter Overview
This chapter focused on solving some differential equations. We also determine appro-
priate techniques in solving ordinary differential equations.

Learning Objectives
At the end of this chapter, the student is expected to

1. solve ordinary differential equations by separation of variables, if applicable

e
Us
2. select an appropriate technique in solving ordinary differential equations of order
one

l
2.1 Separable Variables
cia
er
We now begin solving differential equations with the simplest of all the differential equations:
first-order ordinary differential equations with separable variables. In this point forward,
m
the students are urged to refresh their knowledge in basic and advanced techniques of in-
om

tegration (integration by substitution, integration by parts, partial fraction decomposition,


etc.).
C

Definition 1: Separable Equation


or

A first-order differential equation of the form


tF

dy
= g(x)h(y)
dx
No

is said to be separable or to have separable variables.

dy dy
For example, = y 2 xe3x+4y is separable, while = y + sin x is nonseparable.
dx dx
dy dy   dy
= y 2 xe3x+4y can be written as = y 2 xe3x e4y = xe3x y 2 e4y but for = y + sin x,
dx dx dx
there is no way that y + sin x can be written as a product of a function of x and a function
of y.
18

Example 1. Find the general solution of the following separable DE.


dy x2 + 2
1. − =0
dx y

Solution: By separating variables, and applying integral to each term, we have

y dy = (x2 + 2) dx
Z Z
y dy = (x2 + 2) dx
y2 x3
= + 2x + c1
2 3
Solve for y by multiplying both sides of the equation by 2 and then extracting square

e
roots of both sides.

Us
2x3
y2 = + 4x + 2c1 but 2c1 can be written as C
3
r r

l
p

cia
2x3 2x3
∴ |y| = + 4x + C or y = ± + 4x + C since y 2 = |y|
3 3
er
m
dy
2. x = (1 − 2x2 ) tan y
dx
om

Solution: By separating variables, we have


C

dy 1 − 2x2
= dx
tan y x
or

Z Z  
1
cot y dy = − 2x dx
tF

x
ln |sin y| = ln |x| − x2 + c1 ←− take the exponential of both sides
No

2 +c
eln|sin y| = eln|x|−x 1

2 2
|sin y| = eln|x| · e−x · ec1 = |x| · e−x · ec1 ←− by rules of exponents
c1 −x2
sin y = ±e · x · e ←− definition of absolute value
−x2
sin y = Cxe ←− note that ± ec1 = C since ec1 is constant
 2

∴ y = sin−1 Cxe−x ←− general solution
19

3. sec2 x dy + csc y dx = 0

Solution: By separating the variables, we have


Z
dy dx
+ = 0dx
csc y sec2 x
Z Z Z
1 + cos 2θ
sin y dy + cos x dx = 0dx ←− Use identity cos2 θ =
2
2
Z Z   Z
1 + cos 2x
sin ydy + dx = 0dx
2
Z Z Z Z
1 cos 2x
sin ydy + dx + dx = 0dx
2 2
1 1 1

e
− cos y + x + · sin 2x = c1 ←− Multiply both sides by 4
2 2 2

Us
∴ −4 cos y + 2x + sin 2x = 4c1 or 4 cos y = 2x + sin 2x + C where C = −4c1

l
dy xy + 3x − y − 3
4.
dx
=
xy − 2x + 4y − 8
cia
er
Solution: Express the right side of the DE in factored form.
m
dy x(y + 3) − (y + 3)
=
om

dx x(y − 2) + 4(y − 2)
dy (y + 3)(x − 1)
=
dx (y − 2)(x + 4)
C

Separate the variables and then integrate both sides.


or

Z   Z  
y−2 x−1
dy = dx
tF

y+3 x+4
Z   Z  
5 5
1− dy = 1− dx
No

y+3 x+4

y − 5 ln |y + 3| = x − 5 ln |x + 4| + C

We can also write the answer as


(y + 3)5
y − x = 5 ln |y + 3| − 5 ln |x + 4| + C =⇒ y − x = ln +C
(x + 4)5
(y + 3)5
ln +C
(x + 4) 5
=⇒ ey−x =e
 5
y+3
=⇒ ey−x =K ; K = eC
x+4
20

dP
5. = P − P2
dt
Solution: By separating the variables and applying integral on both sides, we have
Z Z
dP dP
= dt =⇒ = dt
P − P2 P (1 − P )
Z
dP
can be solved using partial fraction decomposition. That is,
P (1 − P )

1 A B
= +
P (1 − P ) P 1−P
1 = A(1 − P ) + B(P )

e
Us
Let P = 0, 1 = A
Let P = 1, 1 = B

l
cia
Hence, Z Z
dP
= dt
P (1 − P )
er
Z   Z
1 1
+ dP = dt
m
P 1−P
Z Z Z
dP dP
om

+ = dt
P 1−P
ln |P | − ln |1 − P | = t + c1
C

To solve for P in terms of t, apply some properties of exponents and logarithms.


or

P
ln |P | − ln |1 − P | = t + c1 =⇒ ln = t + c1
tF

1−P
P
ln
e 1 − P = et+c1 = et · ec1
No

P
= Cet , C = ±ec1
1−P
P = Cet − P · Cet

P 1 + Cet = Cet
Cet
∴, P =
1 + Cet
21

dy
Remarks 1. To solve for a separable differential equation of the form = g(x)h(y), we
dx
dy
tend to write the equation as = g(x)dx. In this case, there is a possibility to have a
h(y)
variable divisor that could be zero. For an instance, if a is a zero of the function h(y), then
dy
y = a is a constant solution of the differential equation = g(x)h(y). Applying separation
dx
dy
of variables, the left-hand side of = g(x)dx is now undefined at a. As a result, y = a
h(y)
may not come up in the family of solutions obtained after applying integration and using
some simplifications. Such solution is called a singular solution (which have been mentioned
in Chapter 1).

Example 2. Solve the following IVP by separation of variables.

e
dy x

Us
1. + = 0 ; y(4) = −3
dx y

l
Solution: By separating variables and taking the integral of both sides, we have

Z
y dy + x dx = 0
Z Z cia
er
y dy + x dx = 0 dx
m
y 2 x2
+ = c1
om

2 2
x2 + y 2 = C ←− general solution
C

The general solution x2 + y 2 = C determines a solution which represents a family of


concentric circles centered at the origin.
or
tF

Now, the initial condition y = −3 when


No

x = 4 implies C = (4)2 + (−3)2 = 25.


Thus a solution to the IVP is
x2 + y 2 = 25 (which is a circle with
radius 5 and centered at the origin). The

explicit solutions are y = ± 25 − x2
and the solution curve that satisfies the
condition is the lower semi-circle which
passes through (4, −3).
22

dy
2. (e2y − y) cos x = ey sin(2x) ; y(0) = 0
dx
Solution: By separating variables, we have

e2y − y sin(2x)
y
dy = dx
Z e Z cos x
 2 sin x cos x
ey − ye−y dy = dx ←− Use sin 2θ = 2 sin θ cos θ
cos x
Z Z Z
ey dy − ye−y dy = 2 sin x dx ←− use Integration by Parts
 
ey − −ye−y − (e−y ) = 2(− cos x) + C
ey + ye−y + e−y = −2 cos x + C ←− general solution

e
Now, the initial condition y = 0 when x = 0 implies C = e0 + (0)e0 + e0 + 2 cos(0) = 4.

Us
Thus a solution to the IVP is ey + ye−y + e−y = 4 − 2 cos x .
Z
Z Z

l
*For ye−y dy

cia ye dy = −ye − −e−y dy


−y −y
−y
Let u = y and dv = e dy
Then du = dy and v = −e−y = −ye−y − e−y + c1
er
m
Some Applications
om

I. Newton’s Law of Cooling/Warming: The rate at which the temperature of a body


changes is proportional to the difference between the temperature of the body and the tem-
C

perature of the surrounding medium, the so-called ambient temperature. If T (t) represents
the temperature of a body at time t > 0; Tm the temperature of the surrounding medium,
or

dT
and the rate at which the temperature of the body changes, then Newton’s law of cool-
tF

dt
ing/warming translates into the separable differential equation
No

dT
= k(T − Tm ),
dt

where k is a constant of proportionality. In either case, cooling or warming, if Tm is a con-


stant, it stands to reason that k < 0.

Example 3. A copper ball is heated to 100◦ C. It is then placed in water that is maintained
at 30◦ C. After 3 minutes, the temperature of the ball is 60◦ C. Find the time at which the
temperature of the ball will be 31◦ C.
Solution: We identify Tm = 30◦ C and T (3) = 60. We set the IVP

dT
= k(T − 30), T (0) = 100
dt
23

and determine the value of t so that T (t) = 31◦ C. By separating variables,

Z Z
dT dT
= k dt ⇐⇒ = k dt
T − 30 T − 30
ln |T − 30| = kt + C1 ←− take the exponential of both sides
T − 30 = ekt+C1 = ekt eC1
T = 30 + Cekt ←− note that eC1 = C is constant
When t = 0, then T = 100. So that, 100 = 30 + Cek(0) =⇒ C = 70.
Substitute C = 70 to T = 30 + Cekt . We have T = 30 + 70ekt .
Now, T = 60 when t = 3 (minutes). So that 60 = 30 + 70e3k ⇐⇒ 30 = 70e3k .

e
30

Us
Solving for k we have e3k = ←− take the ln of both sides
  70
 
3 1 3
3k = ln =⇒ k = ln =⇒ k ≈ −0.2824.

l
7 3 7

cia
Substitute k = −0.2824 to T = 30 + Cekt . We have T = 30 + 70e−0.2824t .
er
We want t when T = 31. Thus,
31 = 30 + 70e−0.2824t ⇐⇒ 1 = 70e−0.2824t
m
 
−0.2824t 1 1
e = ⇐⇒ −0.2824t = ln
om

70 70
 
1 1
t=− ln
C

0.2824 70
∴ t = 15.04 minutes
or
tF

II. Water Tank Problem: Suppose that water is draining out from a large cylindrical
No

tank through a small hole or tap at the bottom. Then the depth of water remaining in the
tank is constantly changing, call it h(t) where t is time. From the theory of fluid mechanics
(Bernoulli’s equation), it can be shown that h(t) must satisfy the differential equation

dh √
= −k h ,
dt

where k is some constant depending on the tap radius at the bottom, the tank radius, and
the acceleration due to gravity.
Problem: Let H be the initial depth of the tank, so that h(0) = H. Determine the time t
so that the tank will be empty, that is, h(t) = 0.
24

Solution:
dh √
We set the IVP: = −k h , h(0) = H.
dt
By separating variables, we have
Z Z
dh −1/2
1/2
= −k dt ⇐⇒ h dh = −k dt ⇐⇒ 2h1/2 = −kt + C
h
Since h(0) = H, t = 0 and h = H, then 2H 1/2 = −k(0) + C =⇒ C = 2H 1/2 .
Substitute C = 2H 1/2 to 2h1/2 = −kt + C. We have,
2h1/2 = −kt + 2H 1/2 ←− solve for h
 1/2 2
1/2 1/2 2H − kt
2h = 2H − kt ⇐⇒ h =
2

e
Us
Solve for t when h = 0. ←− this is the case where the tank is empty
 1/2 2
2H − kt
0= ⇐⇒ kt = 2H 1/2
2

l
cia

2H 1/2 2 H
∴ t= or t =
k k
er
More examples on applications of differential equations will be discussed on a separate chap-
m
ter.
C om

2.2 Linear First-Order Equation


or

From Chapter 1, we already discussed the linearity of ODEs. In this section, we focus on
tF

finding solutions for the linear first-order ODEs.

Definition 2: Linear Equation


No

A first-order differential equation of the form


dy
a1 (x) + a0 (x)y = g(x) (1)
dx
is said to be a linear equation in the dependent variable y.
25

Remarks 2.

1. When g(x) = 0, the linear equation is said to be homogeneous; otherwise, it is called


nonhomogeneous.

2. By dividing both sides of (1) by the leading coefficient a1 (x), we obtain a more useful
form, the standard form, of a linear equation:
dy
+ P (x)y = f (x) (2)
dx
Our goal is to seek a solution of (2) for which both coefficient functions P (x) and f (x)
are continuous.

3. The differential equation (2) has the property that its solution is the sum of the two

e
solutions: y = yc + yp , where yc (called complementary solution) is a solution of

Us
the associated homogeneous equation
dy

l
+ P (x)y = 0 (3)
dx
cia
and yp is a particular solution of the nonhomogeneous equation (2).
er
Steps in Solving a Linear First-Order Differential Equations (Integrating Factor
m
Method)
om

1. Put a linear equation of form (1) into the standard form (2).
C

2. From
R the standard form identify P (x) and then find the integrating factor: I.F. =
e P (x)dx .
or

3. Multiply the standard form of the equation by the integrating factor. The left-hand
tF

side of the resulting equation is automatically the derivative of the integrating factor
and y:
d h R P (x)dx i R
y = e P (x)dx f (x)
No

e (4)
dx
4. Integrate both sides of this last equation (4).
26

Example 4. Find the solution of the following linear DE.


dy
1. − y = e2x
dx

Solution: TheR given is already in standard form. We now identify P (x) = −1.
Then I.F. = e (−1) dx = e−x . (Note: In the previous integral, we do not include +C
since this will be included in the general solution later.) Multiply I.F. = e−x to the
given DE (since the given DE is already in standard form), we have

dy 
e−x − e−x y = e−x e2x
| dx{z }
d −x 
e y = ex ←− refer to Step #3 and #4

e
dx
Z   Z

Us
y
d x = ex dx ←− integration cancels out the derivative
e
y
= ex + C ←− solve for y

l
ex

cia
∴ y = e2x + Cex ←− general solution on (−∞, ∞)
er
dy
2. x − 4y = x6 ex
m
dx
om

dy 4
Solution: Dividing both sides by x, we obtain the standard form: − y = x5 e x .
dx x
4 R 1
C

−4 dx/x −4 ln x ln x−4
We identify P (x) = − . Then I.F. = e =e =e = x−4 = 4
x x
1
or

Multiply I.F. = 4 to the standard form, we have


x
 
tF

1 dy 4 1 5 x

− y = x e
x4 dx x x4
  
No

d 1
4
y = xex ←− refer to Step #3 and #4
dx x
Z    Z Z
1 x
d 4
y = xe dx ←− use Integration by Parts to xex dx
x
y
= xex − ex + C ←− solve for y
x4
∴ y = x5 ex − x4 ex + Cx4 ←− general solution on (0, ∞)
27

1 dy
3. · + y = 2 ; y(0) = 3
2x dx
dy
Solution: Multiplying both sides by 2x, we obtain the standard form: + 2xy = 4x
R 2
dx
We identify P (x) = 2x. Then I.F. = e 2xdx = ex .
2
Multiply I.F. = ex to the standard form, we have
 
x2 dy 2
e + 2xy = ex (4x)
dx
d  x2  2
e y = 4xex ←− refer to Step #3 and #4
Zdx   Z
x2 2
d e y = 4xex dx
Z

e
x2 2

Us
ye = 2 ex (2xdx)
2 2
yex = 2ex + C ←− use u = x2 and du = 2xdx

l
C
e
cia
y = 2 + x2 ←− general solution
C
er
Since y(0) = 3, substitute x = 0 and y = 3 to solve for C : 3 = 2 + 0 =⇒ C = 1.
e
m
1
∴ y = 2 + x2 ←− solution to the IVP
e
om

dy
4. + (tan x)y = sin(2x) ; y(0) = 1
dx
C

Solution: TheR given is already in standard form, so we identify P (x) = tan x.


or

Then I.F. = e tan x dx = eln sec x = sec x Multiply I.F. = sec x to the standard form, we
have
tF

 
dy 1
sec x + (tan x)y = sec x [sin(2x)] ←− sec x = , sin(2x) = 2 sin x cos x
dx cos x
No

 
d 1
[(sec x)y] = (2 sin x cos x)
dx cos x
Z Z
d
[(sec x)y] = 2 sin x dx
dx
y sec x = −2 cos x + C
y
= −2 cos x + C
cos x
y = −2 cos2 x + C cos x ←− general solution
Since y(0) = 1, substitute x = 0 and y = 1 to solve for C :
1 = −2 (cos 0)2 + C cos 0 ⇐⇒ 1 = −2(1)2 + C(1) =⇒ C = 3.
∴ y = −2 cos2 x + 3 cos x ←− solution to the IVP
28

Example 5. Find a continuous solution satisfying



dy  1 if 0 ≤ x ≤ 3
+ 2y = f (x) wheref (x) = , y(0) = 0
dx  0 if x > 3

Solution Take note that f (x) has a discontinuity at x = 3.Consequently, we solve the
problem in two parts corresponding to the two intervals over which f is defined.
dy R
For 0 ≤ x ≤ 3, we have the standard form + 2y = 1 with I.F.: e 2dx = e2x .
dx Z Z
2x dy d 2x  
Hence, e 2x 2x
+ 2e y = e =⇒ 2x
e y = e =⇒ d e y = e2x dx.
2x
dx dx
1 1
From the last equation, we have e2x y = e2x + c1 or y = + c1 e−2x . Since y(0) = 0, then

e
2 2

Us
1 1 1 −2x
c1 = − . Therefore, y = − e if 0 ≤ x ≤ 3
2 2 2

l
dy R

cia
For x > 3, we have the standard form + 2y = 0 with I.F.: e 2dx = e2x .
dx Z Z
2x dy 2x d 2x
 2x

Hence, e + 2e y = 0 =⇒ e y = 0 =⇒ d e y = 0 dx.
er
dx dx
From the last equation, we have e2x y = c2 or y = c2 e−2x .
m
We can now write 
 1 − 1 e−2x if 0 ≤ x ≤ 3
om

y= 2 2

c2 e−2x if x > 3
C

In order for y to be a continuous function, we definitely want lim− y(x) = lim+ y(x) = y(3).
x→3 x→3
or

1 1
Determining the value of c2 , we have c2 = e6 − .
2 2
tF

Therefore, 
 1 1 −2x

 − e if 0 ≤ x ≤ 3
2 2
No

y=  

 1 6 1 −2x
 e − e if x > 3
2 2
29

Example 6. Solve: ydx − 4(x + y 6 )dy = 0


Solution: Take note that the given differential equation is not linear in y because of the
term y 6 dy. Dividing both sides of the equation by ydy we have,
dx 4(x + y 6 ) dx 4
Standard Form: − = 0 =⇒ − x = y 5 (linear in x)
R dy y dy y
−4dy/y −4 ln y ln y −4 −4 1
IF: e =e =e = y = y4
Multiply both sides of the standard form by the IF
1 dx 4
4
− 5x = y
y dy y
| {z }
 
d 1
x =y
dy y 4
Z   Z

e
1
d 4 x = ydy

Us
y
1 y2
x = + c1 or 2x = y 6 + Cy 4 , y > 0
y4 2

l
cia
Remarks 3. If a differential equation is linear in x, then the standard form is
dx
dy
+ R(y)x =
er
 
R d R R
g(y) and IF is e R(y)dy which implies that we are going to have e R(y)dy · x = e R(y)dy ·
m
dy
g(y).
om

Some Applications
C

I. Series Circuits: For a series circuit containing only a resistor and an inductor,
 Kirch-

di
or

hoff?s second law states that the sum of the voltage drop across the inductor L and
dt
tF

the voltage drop across the resistor (Ri) is the same as the impressed voltage (E(t)) on the
circuit.
No

Thus we obtain the linear differential equation for the current i(t),

di
L + Ri = E(t),
dt
where L and R are constants known as the inductance and the resistance, respectively.
The current i(t) is also called the response of the system.
30

1
Example 7. A 12-volt battery is connected to a series circuit in which the inductance is
2
henry and the resistance is 10 ohms. Determine the current i if the initial current is zero.
Solution:
1
We identify L = (henry), R = 10 (ohms), and E = 12 (volts). Then the IVP linear DE
2
will be
1 di
· + 10i = 12 ; i(0) = 0.
2 dt
di
Multiplying both sides by 2, we obtain the standard form: + 20i = 24. We identify
R R dt
P (t) = 20. Then I.F. = e P (t)dt = e 20dt = e20t .
Multiply I.F. = e20t to the standard form, we have
 

e
20t di
+ 20i = e20t (24)

Us
e
dt
d 20t 
e i = 24e20t

l
Z dt Z

cia

d e i = 24 e20t dt
20t

Z
er
20t 1
ie = 24 · e20t (20dt) ←− use u = 20t and du = 20dt
20
m
6
ie20t = e20t + C
om

5
6 C
i = + 20t ←− general solution
5 e
C

Since i(0) = 0, substitute t = 0 and i = 0 to solve for C :


6 C 6
or

0 = + 20(0) =⇒ C = − .
5 e 5
tF

6 6
Therefore the current/response is i = − 20t
5 5e
No

II. Free Falling Body with Air Resistance: This concerns a body falling under gravity
with air resistance. Let x(t) be the downward displacement at time t from the point of
release, then the differential equation is
d2 x dx
2
=g−k ,
dt dt
dx
where k is the air resistance with constant k and g is the acceleration due to gravity.
dt
Problem: Determine the solution x(t) to the above differential equation subject to the
initial conditions: x(0) = 0 and x0 (0) = 0. [Note that x(0) = 0 implies that the particle is
initially at the point and release, and x0 (0) = 0 implies that the particle is released from
rest, so that the initial velocity is zero.]
31

dx
Solution: Let v = . Then the differential equation can be rewritten as
dt
dv dv
= g − kv ⇐⇒ + kv = g
dt dt
which isRnow a linear
R differential equation in standard form. We identify P (t) = k and
P (t)dt kdt
I.F. = e =e = ekt . Multiply I.F. = ekt to the standard form, we have
 
kt dv
e + kv = ekt (g)
dt
d kt 
e v = gekt
Z dt Z

d e v = gekt dt
kt

e
Us
Z
kt 1 
ve = g · ekt (kdt) ←− g, k are constants
k
g

l
vekt = ekt + C1

cia
k
g C1
v = + kt ←− general solution
er
k e
0
Since x (0) = 0, so v(0) = 0. Substitute t = 0 and v = 0 to solve for C1 :
m
g C1 g
0 = + k(0) =⇒ C1 = −
om

k e k
g g g g −kt
Hence, v = − kt ⇐⇒ v = − e
k ke k k
C

dx dx g g
But v = , so that = − e−kt
dt dt k k
or

Integrating with respect to t, we have


tF

Z Z h
g g −kt i
dx = − e dt
k k
Z Z Z
No

g g
dx = dt − e−kt dt
k k
g
Z g   1 Z
x= dt − − e−kt (−kdt)
k k k
gt g
x= + 2 e−kt + C2
k k
Since x(0) = 0, substitute t = 0 and x = 0 to solve for C2 :
g(0) g g
0= + 2 e−k(0) + C2 =⇒ C2 = − 2
k k k
gt g g
Therefore the downward displacement is x = + 2 e−kt − 2
k k k
32

g g g g
Nice to know: From the equation v = − e−kt , if t → ∞, then v → . We call
k k k k
the terminal velocity. For a free falling parachutist, the terminal velocity is about 193
km/hour (before the parachute opened!).

Activity 1

A. Solve the following differential equations by separation of variables.


dy
1. = e3x+2y
dx
 2
dx y+1
2. y ln x =
dy x

e
2 3
3. ey + 1 e−y dx + ex + 1 e−x dy = 0

Us
π
4. sin x cos2 y dx + cos2 x dy = 0, y(0) =
4

l
cia

p √ 3
5. 1 − y 2 dx − 1 − x2 dy = 0, y(0) =
2
er
B. Find the general solution of the given differential equations and give the largest interval
m
over which the general solution is defined.
om

dy
1. x + 4y = x3 − x
dx
C

dy
2. (x + 1) + (x + 2)y = 2xe−x
dx
or

dr
3. + r sec θ = cos θ
tF


4. cos2 x sin x dy + (y cos3 x − 1)dx = 0
No

dy
5. x + (3x + 1)y = e−3x
dx
C. Find a continuous solution satisfying

dy  x if 0 ≤ x < 1
+ 2xy = f (x) where f (x) = , y(0) = 2
dx  0 if x ≥ 1
33

2.3 Exact Equations


Recall from Calculus 1: If z = f (x, y) is a function of two variables with continuous first
partial derivatives in a region R of the xy-plane, then its differential is
∂f ∂f
dz = dx + dy (1)
∂x ∂y

In the special case when f (x, y) = c, where c is a constant, then (1) implies

∂f ∂f
dx + dy = 0 (2)
∂x ∂y

In other words, given a one-parameter family of functions f (x, y) = c, we can generate a

e
first-order differential equation by computing the differential of both sides of the equality.

Us
Example 8. If f (x, y) = x2 − 5xy + y 3 = c, then (2) gives the first order DE:

l
∂f ∂f

cia
dx + dy = 0
∂x ∂y
∂ 2 ∂ 2
(x − 5xy + y 3 ) dx + (x − 5xy + y 3 ) dy = 0
er
∂x ∂y
m
(2x − 5y)dx + (−5x + 3y 2 )dy = 0
[Note that in partial derivative with respect to x, we treat y as a constant (and vice versa).]
om

Notations:
∂f ∂f
C

We use the notation := M (x, y) and := N (x, y). Then (2) can be rewritten as
∂x ∂y
or

M (x, y)dx + N (x, y)dy = 0.


tF

∂f ∂f
We can see from Example 8 that = M (x, y) = 2x − 5y and = N (x, y) = −5x + 3y 2 .
∂x ∂y
No

Definition 3: Exact Equation

A differential expression M (x, y)dx + N (x, y)dy is an exact differential in a region


R of the xy-plane if it corresponds to the differential of some function f (x, y) defined
in R.
A first-order differential equation of the form

M (x, y)dx + N (x, y)dy = 0 (3)

is said to be an exact equation if the expression on the left-hand side is an exact


differential.
34

Theorem 1: Criterion for an Exact Differential


Let M (x, y) and N (x, y) be continuous and have continuous first partial derivatives
in a rectangular region R defined by a < x < b, c < y < d. Then a necessary and
sufficient condition that M (x, y)dx + N (x, y)dy be an exact differential is

∂M ∂N
= (4)
∂y ∂x

Remarks 4. The idea of the previous theorem relies on the fact that
   
∂ ∂f ∂ 2f ∂ ∂f ∂ 2f
= = =
∂y ∂x ∂y ∂x ∂x ∂y ∂x ∂y

e
Us
Therefore,    
∂M ∂ ∂f ∂ ∂f ∂N
= = =
∂y ∂y ∂x ∂x ∂y ∂x

l
cia
Method of Solution: Given an equation in the differential form M (x, y)dx+N (x, y)dy = 0,
determine whether the equality in (4) holds. If it does, then there exists a function f for
er
which
∂f
= M (x, y).
m
∂x
We can find f by integrating M (x, y) with respect to x while holding y constant:
om

Z
f (x, y) = M (x, y)dx + g(y), (5)
C

where the arbitrary function g(y) is the constant of integration. Now differentiate (5) with
or

∂f
respect to y and assume that = N (x, y) :
tF

∂y
Z
∂f ∂
= M (x, y) dx + g 0 (y) = N (x, y).
No

∂y ∂y
This gives Z
0 ∂
g (y) = N (x, y) − M (x, y) dx (6)
∂y
Finally, integrate (6) with respect to y and substitute the result in (5). The (implicit)
general solution of the exact DE is of the form f (x, y) = C.
∂f
We can also start the foregoing procedure with the assumption that = N (x, y). After
∂y
integrating N with respect to y and then differentiating that result, we would find the
analogues of (5) and (6) to be, respectively,
Z Z
0 ∂
f (x, y) = N (x, y) dy + h(x) and h (x) = M (x, y) − N (x, y) dy.
∂x
35

Example 9. Find the solution of the following exact DE.


1. 2xy dx + (x2 − 1) dy = 0

Solution: We identify M (x, y) = 2xy and N (x, y) = x2 − 1. Then


∂M ∂
= (2xy) = 2x
∂y ∂y
∂N ∂ 2
= (x − 1) = 2x
∂x ∂x
∂M ∂N
∴ =
∂y ∂x
Thus, the DE is exact. By Theorem 1, there exists a function f (x, y) such that
∂f ∂f

e
M (x, y) = = 2xy and N (x, y) = = x2 − 1.

Us
∂x ∂y
∂f
Integrating M (x, y) = = 2xy with respect to x, we have
∂x

l
Z Z Z

cia
∂f
dx = f (x, y) = 2xy dx = 2y x dx ←− y is treated as a constant
∂x
 2
er
x
f (x, y) = 2y + g(y) ←− g(y) is the constant of integration
2
m
f (x, y) = x2 y + g(y)
om

Taking the partial derivative of f (x, y) = x2 y + g(y) with respect to y and equating to
N (x, y), we have
C

∂ ∂ 2
f (x, y) = [x y + g(y)] = x2 + g 0 (y) = x2 − 1 = N (x, y)
∂y ∂y
or

It follows that g 0 (y) = −1. Integrating g 0 (y) with respect to y, we have


Z Z
tF

0
g (y) dy = −dy ⇐⇒ g(y) = −y.
No

Take note that no arbitrary constant is needed in obtaining g(y) since there is one
already on the right side in the solution f (x, y) = C
Hence, f (x, y) = x2 y + g(y) = x2 y − y.
Therefore, the (implicit) general solution to the exact DE is
f (x, y) = C ⇐⇒ x2 y − y = C

∂f
Alternative Solution: In this case, we choose to integrate N (x, y) = = x2 − 1
∂y
with respect to y, we have
Z Z Z
∂f 2 2
dy = f (x, y) = (x − 1) dy = (x − 1) dy ←− x treated as constant
∂y
f (x, y) = (x2 − 1)y + h(x) ←− h(x) is the constant of integration
f (x, y) = x2 y − y + h(x)
36

Taking the partial derivative of f (x, y) = x2 y − y + h(x) with respect to x and equating
to M (x, y), we have

∂ ∂ 2
f (x, y) = [x y − y + h(x)] = 2xy + h0 (x) = 2xy = M (x, y)
∂x ∂x
It follows that h0 (x) = 0. Integrating h0 (x) = 0 with respect to x will give us h(x) = c1 .
Hence, f (x, y) = x2 y − y + h(x) = x2 y − y + c1 , where c1 is constant
Therefore, the (implicit) general solution to the exact DE is

f (x, y) = C ⇐⇒ x2 y − y = C

2. [e2y − y cos(xy)] dx + [2xe2y − x cos(xy) + 2y] dy = 0

e
Us
Solution: We identify M (x, y) = e2y − y cos(xy) and N (x, y) = 2xe2y − x cos(xy) + 2y.
Then
∂M ∂ 2y

l
[e − y cos(xy)] = 2e2y + xy sin(xy) − cos(xy)

cia
=
∂y ∂y
∂N ∂
= [2xe2y − x cos(xy) + 2y] = 2e2y + xy sin(xy) − cos(xy)
er
∂x ∂x
∂M ∂N
m
∴ =
∂y ∂x
om

Thus, the DE is exact. By Theorem 1, there exists a function f (x, y) such that
C

∂f ∂f
M (x, y) = = e2y − y cos(xy) and N (x, y) = = 2xe2y − x cos(xy) + 2y
∂x ∂y
or
tF

[Note: The choice between M (x, y) or N (x, y) in solving for f (x, y) is up to the reader.
But in this example, we choose N (x, y) as our working function.]
∂f
No

Integrating N (x, y) = = 2xe2y − x cos(xy) + 2y with respect to y (treat x as


∂x
constant), we have
Z Z
∂f
dy = f (x, y) = [2xe2y − x cos(xy) + 2y]dy
∂y
Z Z Z
2y
f (x, y) = 2x e dy − x cos(xy)dy + 2 y dy
Z Z Z
1 2y

f (x, y) = 2x · e (2dy) − [cos(xy)](x dy) + 2 y dy
2
f (x, y) = xe2y − sin(xy) + y 2 + h(x); h(x) is constant of integration
37

Taking the partial derivative of f (x, y) = xe2y − sin(xy) + y 2 + h(x) with respect to x
and equating to M (x, y), we have

∂ ∂
f (x, y) = [xe2y − sin(xy) + y 2 + h(x)]
∂x ∂x
= e2y − y cos(xy) + h0 (x)
= e2y − y cos(xy) = M (x, y)

It follows that h0 (x) = 0. Integrating h0 (x) = 0 with respect to x will give us h(x) = c1 .
Hence, f (x, y) = xe2y − sin(xy) + y 2 + c1
Therefore, the (implicit) general solution to the exact DE is

f (x, y) = C ⇐⇒ xe2y − sin(xy) + y 2 = C

e
Us
3. (cos x sin x − xy 2 )dx + y(1 − x2 )dy = 0 ; y(0) = 2 ←− IVP Exact DE

l
Solution: We identify M (x, y) = cos x sin x − xy 2 and N (x, y) = y(1 − x2 ). Then

∂M
=

cia
(cos x sin x − xy 2 ) = −2xy
er
∂y ∂y
∂N ∂
m
= [y(1 − x2 )] = −2xy
∂x ∂x
om

∂M ∂N
∴ =
∂y ∂x
C

Thus, the DE is exact. By Theorem 1, there exists a function f (x, y) such that
or

∂f ∂f
M (x, y) = = (cos x sin x − xy 2 ) and N (x, y) = = y(1 − x2 )dy
∂x ∂y
tF

∂f
Integrating N (x, y) = = y(1 − x2 ) with respect to y, we have
No

∂y
Z Z
∂f
dy = f (x, y) = y(1 − x2 ) dy
∂y
Z
2
f (x, y) = (1 − x ) y dy ←− x is treated as a constant
y2
f (x, y) = (1 − x2 ) + h(x) ←− h(x) is the constant of integration
2
y2
Taking the partial derivative of f (x, y) = (1 − x2 ) + h(x) with respect to x and
2
equating to M (x, y), we have
 
∂ ∂ y2
f (x, y) = (1 − x ) + h(x) = −xy 2 + h0 (x) = cos x sin x − xy 2 = M (x, y)
2
∂x ∂x 2
38

It follows that h0 (x) = cos x sin x. Integrating h0 (x) with respect to x will give us
Z
1
h(x) = − (cos x)(− sin x dx) = − cos2 x
2
y2 1
Hence, f (x, y) = (1 − x2 ) − cos2 x
2 2
Therefore, the (implicit) general solution to the exact DE is

y2 1
f (x, y) = C ⇐⇒ (1 − x2 ) − cos2 x = C1
2 2
f (x, y) = C ⇐⇒ y (1 − x ) − cos2 x = C ; 2C1 = C is constant
2 2

Since y(0) = 2, substitute x = 0 and y = 2 to solve for C :


(22 )(1 − 02 ) − (cos 0)2 = C ⇐⇒ 4 − 1 = C =⇒ C = 3.

e
Us
∴ y 2 (1 − x2 ) − cos2 x = 3 ←− solution to the IVP

2.4 Differential Equations with Homogeneous Coefficients

l
Definition 4: Homogeneous Equations cia
er
If a function f satisfies the property f (λx, λy) = λα f (x, y) for some real number α,
m
then f is said to be a homogeneous function of degree α. A first-order DE
om

M (x, y) dx + N (x, y) dy = 0 (1)

is said to be homogeneous* if both coefficient functions M and N are homogeneous


C

functions of the same degree. That is, (1) is homogeneous if


or

M (λx, λy) = λα M (x, y) and N (λx, λy) = λα N (x, y).


tF

Note: *Here the word homogeneous does not mean the same as it did in Section 2.2. Recall
dy
No

that a linear first-order equation a1 (x) + a0 (x)y = g(x) is homogeneous when g(x) = 0.
dx
Example 10.

1. f (x, y) = x3 + y 3 is homogeneous of degree 3 since


f (λx, λy) = (λx)3 + (λy)3 = λ3 x3 + λ3 y 3 = λ3 (x3 + y 3 ) = λ3 f (x, y)

2. f (x, y) = x2 y 2 + xy is not homogeneous since


f (λx, λy) = (λx)2 (λy)2 + (λx)(λy) = λ4 (x2 y 2 ) + λ2 (xy) 6= λα f (x, y) for some α ∈ R.
39

Method of Solution: If M and N are homogeneous functions of degree α, we can write


y
M (x, y) = xα M (1, u) and N (x, y) = xα N (1, u), where u = , (2)
x
x
M (x, y) = y α M (v, 1) and N (x, y) = y α N (v, 1), where v = , (3)
y
The substitutions that can be used to solve a homogeneous differential equation. Specifically,
either of the substitutions y = ux or x = vy, where u and v are new dependent variables,
will reduce a homogeneous equation to a separable first-order differential equation.

Example 11.
1. Show that (x2 + y 2 )dx + (x2 − xy)dy = 0 is a homogeneous DE and then find its

e
solution.

Us
Solution: We identify M (x, y) = x2 + y 2 and N (x, y) = x2 − xy. Observe that
M and N are both homogeneous of degree 2 since

l
cia
M (λx, λy) = (λx)2 + (λy)2 = λ2 (x2 + y 2 ) = λ2 M (x, y)
N (λx, λy) = (λx)2 − (λx)(λy) = λ2 (x2 − xy) = λ2 N (x, y)
er
Let y = ux, then dy = udx + xdu. Substitute these to the given DE, we have
 2 
m
x + (ux)2 dx + [x2 − x(ux)](udx + xdu) = 0
om

(x2 + u2 x2 ) dx + (x2 − ux2 )(udx + xdu) = 0


(x2 + u2 x2 ) dx + (ux2 − u2 x2 ) dx + (x3 − ux3 ) du = 0
x2 (1 + u) dx + x3 (1 − u)du = 0
C

 
dx 1−u
+ du = 0 ←− by separating variables
or

x 1+u
 
tF

dx 2
+ −1 + du = 0 ←− by long division
x 1+u
Z Z   Z
dx 2
No

+ −1 + du = 0 dx ←− integrate both sides


x 1+u
y
ln |x| − u + 2 ln |1 + u| = C ←− back substitute u =
x
y y
∴ ln |x| − + 2 ln 1 + = C ←− (implicit) general solution
x x

Note: It will be tedious to solve for the explicit general solution for y so we skip that
part. The reader can also verify that the general solution may be written as
(x + y)2 y
ln = or (x + y)2 = Cxey/x
Cx x
using the properties of logarithms. We can also use the substitution x = vy whenever
the function M (x, y) is simpler than N (x, y).
40

2. Show that (x3 − y 3 )dx + xy 2 dy = 0 is a homogeneous DE and solve the IVP: y(1) = 2.

Solution: We identify M (x, y) = x3 − y 3 and N (x, y) = xy 2 . Observe that M and N


are both homogeneous of degree 3 since

M (λx, λy) = (λx)3 − (λy)3 = λ3 (x3 − y 3 ) = λ3 M (x, y)


N (λx, λy) = (λx)(λy)2 = λ3 (xy 2 ) = λ3 N (x, y)

Let x = vy, then dx = vdy + ydv. Substitute these to the given DE, we have
 
(vy)3 − y 3 (vdy + ydv) + (vy)y 2 dy = 0
(v 3 y 3 − y 3 )(vdy + ydv) + vy 3 dy = 0

 

v 4 y 3 dy + v 3 y 4 dv −  vy3
dy − y 4 dv + vy 3
dy = 0

e
4 3 4 3
v y dy + y (v − 1)dv = 0

Us
 3 
dy v −1
+ dv = 0 ←− by separating variables
y v4
Z Z Z Z

l
cia
dy dv −4
+ − v dv = 0 dx ←− integrate both sides
y v
 −3 
v
er
ln |y| + ln |v| − =C
−3
m
1 x
ln |y| + ln |v| + 3 = C ←− back substitute v =
om

3v y
x 1
ln |y| + ln +  3 = C
y x
C

3
y
or

3
ln  + ln |x| − ln
|y| + y = C
|y|
  3x3
tF

y3
ln |x| + 3 = C ←− (implicit) general solution
3x
No

Since y(1) = 2, substitute x = 1 and y = 2 to solve for C :


(2)3 8 8
ln |1| + 3
= C ⇐⇒ 0 + = C =⇒ C = .
3(1) 3 3
y3 8
∴ ln |x| + 3
= or 3x3 ln |x| + y 3 = 8x3 ←− (implicit) solution to the IVP
3x 3
41

Activity 2

A. Determine the value of k so that the given differential equation is exact.


 
1. y 3 + kxy 3 − 2x dx + 3xy 2 + 20x2 y 3 dy = 0
 
2. 6xy 3 + cos y dx + 2kx2 y 2 − x sin y dy = 0

B. Determine whether the given differential equation is exact. If exact, solve it.
 
1. 2xy 2 − 3 dx + 2x2 y + 4 dy = 0
dy
2. x = 2xex − y + 6x2
dx
 

e
3. 4t3 y − 15t2 − y dt + t4 + 3y 2 − t dy = 0

Us
C. Solve the given differential equations with homogeneous coefficients using appropriate
substitutions.

l
cia
√ 
1. −ydx + x + xy dy = 0

er
2. x + yey/x dx − xey/x dy = 0
m
D. Solve the following initial-value problems.
om

1. (4y + 2t − 5)dt + (6y + 4t − 1)dy = 0, y(−1) = 2


 
2. y 2 cos x − 3x2 y − 2x dx + 2y sin x − x3 + ln y dy = 0, y(0) = e
C

dy
3. xy 2 = y 3 − x3 , y(1) = 2
or

dx
tF
No
42

2.5 Other Methods for First-Order ODE


Reduction to Separation of Variables

A differential equation of the form

dy
= f (Ax + By + C)
dx

can always be reduced to a differential equation with separable variables by using the sub-
stitution

u = Ax + By + C, B 6= 0.

e
Us
Example 12. Solve the following differential equations.

l
dy

cia
1. = (x + y + 1)2
dx
er
Solution:
du dy dy du
Let u = x + y + 1. Then =1+ =⇒ = − 1.
m
dx dx dx dx
The differential equation now becomes
om

du du
− 1 = u2 =⇒ = u2 + 1.
dx dx
C

Separate the variables and take the integral of both sides


or

Z Z
tF

du
= dx
u2 + 1
tan−1 u = x + c1 ←− but u = x + y + 1
No

tan−1 (x + y + 1) = x + C

Solving the last equation for y: x + y + 1 = tan(x + C) =⇒ y = tan(x + C) − x − 1

dy
2. = tan2 (x + y)
dx
Solution:
du dy dy du
Let u = x + y. Then =1+ =⇒ = − 1.
dx dx dx dx
The differential equation now becomes

du du
− 1 = tan2 u =⇒ = tan2 u + 1.
dx dx
43

Separate the variables and take the integral of both sides


Z Z
du 1
2
= dx ←− use identities tan2 u + 1 = sec2 u ; cos2 u =
tan u + 1 sec2 u
Z Z
1 + cos 2u
cos2 udu = dx ←− use the identity cos2 u =
2
Z Z
1 + cos 2u
du = dx
2
Z   Z
1 cos 2u
+ du = dx
2 2
1 1
u + sin 2u = x + c1
2 4

e
Multiply both sides of the last equation by 4 and substitute u = x + y. Hence, we

Us
have
2(x + y) + sin 2(x + y) = 4x + C or 2y − 2x + sin 2(x + y) = C

l
cia
Coefficients Linear in Two Variables er
Consider a differential equation of the form
 
m
a1 x + b1 y + c1 dx + a2 x + b2 y + c2 dy = 0 (1)
om

where a, b, c are constants. If c1 and c2 are both 0, then (1) becomes a differential equation
with homogeneous coefficient.
C

Now, consider the lines formed by the coefficients of dx and dy,

a1 x + b1 y + c1 = 0
or

(2)
a2 x + b 2 y + c 2 = 0
tF

a1 b1
• If the lines in (2) are parallel, that is,
= , then (1) can be solved by reduction
a2 b2
No

to separation of variables. In this case, we use the substitution u = a1 x + b1 y or


u = a2 x + b2 y.
a1 b1
• If the lines in (2) are intersecting, that is,
6= , then (1) can be solved by reduction
a2 b2
to differential equation with homogeneous coefficients. In this case, we use the
substitutions x = u + h and y = v + k, where (h, k) is the intersection point (2).
44

Example 13.
1. Solve: (x + 2y − 1)dx + (3x + 6y)dy = 0
Solution: From the given differential equation, the linear coefficients are parallel, that
1 3
is, = .
3 6
Let u = x + 2y. Then du = dx + 2dy =⇒ dx = du − 2dy. We now have,
(x + 2y − 1)dx + 3(x + 2y)dy = 0
(u − 1)(du − 2dy) + 3udy = 0
(u − 1)du − 2(u − 1)dy + 3udy = 0
(u − 1)du + (u + 2)dy = 0 ←− separate the variables
Z Z
u−1
du = − dy ←− integrate both sides
u+2

e
Z   Z

Us
3
1− du = − dy ←− by long division
u+2
u − 3 ln |u + 2| = −y + C ←− but u = x + 2y

l
cia
x + 2y − 3 ln |x + 2y + 2| = −y + C or x + 3y − 3 ln |x + 2y + 2| = C
er
dy 3x + 2y
2. Solve: = , y(−1) = −1
dx 3x + 2y + 2
m
a1 b1
Solution: It can be clearly seen from the given that = .
a2 b2
om

du + 3dx
Let u = 3x + 2y. Then du = 3dx + 2dy =⇒ dy = .
2
C

(Note: From the substitution, we can choose to solve for either dx or dy.)
We now have,
or

(3x + 2y + 2)dy = (3x + 2y)dx


 
tF

du + 3dx
(u + 2) = udx ←− multiply both sides by 2
2
(u + 2)(du + 3dx) = 2udx
No

(u + 2)du + (3u + 6)dx = 2udx


(u + 2)du + (u + 6)dx = 0 ←− separate the variables
Z Z
u+2
du = − dx ←− integrate both sides
u+6
Z   Z
4
1− du = − dx ←− by long division
u+6
u − 4 ln |u + 6| = −x + c1 ←− but u = 3x + 2y
3x + 2y − 4 ln |3x + 2y + 6| = −x + c1 or 2x + y − 2 ln |3x + 2y + 6| = C
Since y(−1) = −1, substitute x = −1 and y = −1 to solve for C:
2(−1) + (−1) − 2 ln |3(−1) + 2(−1) + 6| = C =⇒ C = −3
∴ 2x + y − 2 ln |3x + 2y + 6| = −3 ←− solution to IVP
45

3. Solve: (2x − y)dx + (4x + y − 6)dy = 0

Solution: The lines 2x − y = 0 and 4x + y − 6 = 0 intersect at (1, 2).


Let x = u + 1 and y = v + 2.
Then dx = du and dy = dv.
We now have, [2(u + 1) − (v + 2)]du + [4(u + 1) + (v + 2) − 6]dv = 0
Simplify the coefficients of du and dv: (2u − v)du + (4u + v)dv = 0
From the last equation, we use the substitution, u = wv and du = (wdv + vdw).

(2wv − v)(wdv + vdw) + (4wv + v)dv =0


2w vdv + 2wv 2 dw − vwdv − v 2 dw + 4wvdv + vdv
2
=0
(2w2 v + 3wv + v)dv + (2wv 2 − v 2 )dw =0
v(2w2 + 3w + 1)dv + v 2 (2w − 1)dw = 0 ←− use separation of variables

e
Z Z Z

Us
dv 2w − 1
+ dw = 0 dv
v 2w2 + 3w + 1

l
By partial fraction decomposition, we have,
Z
dv
+
Z 
−4
+ cia3
 Z
dw = 0 dv
er
v 2w + 1 w + 1
1
m
ln |v| − 4 · ln |2w + 1| + 3 ln |w + 1| = c1
2
om

u
Replace w = , u = x − 1, and v = y − 2. Hence, we have
v
 
C

x−1 x−1
ln |y − 2| − 2 ln 2 + 1 + 3 ln + 1 = c1
y−2 y−2
or

Applying some properties of logarithms, we can write the solution as


tF

(x+y-3)3 = C(2x + y − 4)3 .


No

Bernoulli’s Equation

Sometimes it is possible to solve a nonlinear equation by making a change of the dependent


variable that converts it into a linear equation. In this case, we can solve such equation by
means of integrating factor method. The most important such equation has the form
dy
+ R(x)y = S(x)y n (3)
dx
where n ∈ R, called the Bernoulli’s equation or Bernoulli DE. If n = 0 or n = 1, then
(3) is linear.
46

Method of Solution: For n 6= 1


If we multiply both sides of (3) by y −n , it becomes
dy
y −n + y 1−n R(x) = S(x) (4)
dx
du dy
Let u = y 1−n . Then = (1 − n)y −n .
dx dx
Multiply both sides of (4) by (1 − n).
dy
(1 − n)y −n + (1 − n)y 1−n R(x) = (1 − n)S(x)
dx
The Bernoulli differential equation is now transformed into the form
du

e
+ (1 − n)R(x) u = (1 − n)S(x) (5)
dx

Us
The above equation is now linear in u and can be solved by integrating factor method.
du

l
+ P (x) u = f (x) (6)

cia
dx
Example 14. Find the solution of the following Bernoulli DE.
er
dy y y −2
1. + =
m
dx x x
dy y y −2
om

+ = ←− multiply both sides by y 2


dx x x
3
dy y 1
y2 + = (1)
C

dx x x
du dy
Let u = y 3 . Then = 3y 2 . Multiply both sides of (1) by 3.
or

dx dx
tF

2 dy 3y 3 3
3y + =
dx x x
du 3 3
No

+ u= ←− standard form, linear in u


dx x x
R 3
3dx/x
IF: e = e3 ln x = eln x = x3
du
x3 + 3ux2 = 3x2 ←− multiply both sides of standard form by IF: x3
| dx {z }
 
d
x3 u = 3x2
dx
Z   Z
d x u = 3x2 dx
3

x3 u = x3 + C ←− but u = y 3
x3 y 3 = x3 + C or y 3 = 1 + Cx3
47

dy
2. x + y = x2 y 2 ln x
dx
Solution: Dividing by x we obtain the Bernoulli DE:
dy 1
+ y = (x ln x)y 2
dx x
1 1
We identify R(x) = , S(x) = x ln x, and n = 2, so that u = y 1−n = y −1 = .
x y
The linear equation form of the Bernoulli DE, from (5), will be

du 1 du 1
+ (1 − 2) u = (1 − 2) x ln x ⇐⇒ − u = −x ln x
dx x dx x

e
We can now solve this by integrating factor method.

Us
1 R −1 1
We identify P (x) = − and I.F. = e (−1/x)dx = e− ln x = eln x = x−1 = .
x x
1
Multiply I.F. = to the standard form, we have

l
x

1 du 1

1
− u = (−x ln x) cia
er
x dx x x
1 du 1 1
m
− 2 u = (−x ln x)
|x dx {z x } x
om

 
d 1
u = − ln x ←− refer to Step #3 and #4 in Section 2.2
dx x
Z   Z Z
C

u
d = − ln x dx ←− use Integration by Parts to ln x dx
x
or

u
= −(x ln x − x) + C = −x ln x + x + C
x
tF

1
u = −x2 ln x + x2 + Cx ←− back substitute u =
y
No

1
= −x2 ln x + x2 + Cx
y
1
∴ y= ←− general solution
−x ln x + x2 + Cx
2

R Z Z
*For ln x dx
Let u = ln x and dv = dx ln x dx = x ln x − dx
dx
Then du = and v = x = x ln x − x + c1
x
48

dy
3. + y = xy 4 ; y(0) = 1
dx
1
Solution: We identify R(x) = 1, S(x) = x, and n = 4. So that u = y 1−n = y −3 = .
y3
The linear equation form of the Bernoulli DE will be
du du
+ (1 − 4)(1)u = (1 − 4)x ⇐⇒ − 3u = −3x
dx dx
We can now solve this by integrating factor
R method.
−3 dx
We identify P (x) = −3 and I.F. = e = e−3x .
Multiply I.F. = e−3x to the standard form, we have
 
−3x du
e − 3u = e−3x (−3x)

e
dx

Us
d −3x 
e u = −3xe−3x ←− refer to Step #3 and #4 in Section 2.2
Zdx Z Z
−3x
 −3x
d ue = −3 xe dx ←− use Integration by Parts to xe−3x dx

l
ue −3x

= −3 − xe
1 −3x 1 −3x
3
− e
9 cia 
+ C1
er
u −3x 1 −3x
= xe + e + C1
m
e3x 3
1 1
om

u = x + + C1 e3x ←− back substitute u = 3


3 y
1 3x + 1 + 3C1 e3x
C

= ←− take the reciprocal ; 3C1 = C


y3 3
3
or

y3 = ←− general solution
3x + 1 + Ce3x
tF

Since y(0) = 1, substitute x = 0 and y = 1 to solve for C :


3
(1)3 = ⇐⇒ 1 + C = 3 =⇒ C = 2
No

3(0) + 1 + Ce3(0)
r
3 3
∴ y =3
3x
or y = 3 ←− (explicit) solution to the IVP
3x + 1 + 2e 3x + 1 + 2e3x
R
*For xe−3x dx
Let u = x and dv = e−3x dx
1
Then du = dx and v = − e−3x
3
Z Z  
−3x 1 −3x 1 −3x
xe dx = − xe − − e dx
3 3
 
1 −3x 1 1 −3x
= − xe + − e + c1
3 3 3
1 1
= − xe−3x − e−3x + c1
3 9
49

Integrating Factors Found by Inspection

Recall from Lesson 2.2 that we can solve a linear differential equation with the aid of an
integrating factor. The same idea sometimes can be applied for a non exact differential
equation M (x, y)dx + N (x, y)dy = 0. For now, we will deal with equations that are simple
enough to determine the integrating factors by inspection. To do this, we have to be familiar
and be able to recognize such exact common differentials (and sometimes, it depends upon
experience). The following are exact differentials that frequently appear.
d(xy) = xdy + ydx 
d exy = exy (xdy + ydx)
   
x ydx − xdy y xdy − ydx
d = d =
y y2 x x2

e
 

Us
ydx − xdy  
−1 x xdy − ydx
d tan = −1 y
y x2 + y 2 d tan =
x x2 + y 2

l
cia
xdy + ydx  
d(ln xy) = −1 xdy + ydx
xy d n−1
=
(n − 1)(xy) (xy)n
er
m
Most likely, when we regroup terms with exact differentials, we will be regrouping terms
with dx and dy. In addition, a differential involving only one variable, like x dx, is an exact
om

differential.

Example 15. Solve the following differential equations by regrouping exact differentials.
C

1. xdy + ydx = 3x2 dx


or

Solution:
Observe that the terms on the right side of the given equation is an exact differential
tF

and the right side is a function of x alone. We have,

xdy + ydx = 3x2 dx


No

| {z }
d(xy) = 3x2 dx ←− take integral of both sides
Z Z
d(xy) = 3x2 dx

xy = x3 + C
50

2. xdy − ydx − x2 + y 2 dx = 0
Solution:

xdy − ydx − x2 + y 2 dx = 0 ←− divide by x2 + y 2
xdy − ydx
−dx = 0
x2 + y 2
| {z }
Z   Z Z
−1 y
d tan − dx = 0dx ←− take integral of both sides
x
y
tan−1 − x = C
x

3. [1 + y tan(xy)]dx + x tan(xy)dy = 0
Solution:

e
Us
dx + x tan(xy)dy = 0 ←− distribute the terms
dx + y tan(xy)dx + x tan(xy)dy = 0 ←− regroup terms with tan(xy)

l
dx + [y tan(xy)dx + x tan(xy)dy] = 0
dx + tan(xy) (ydx + xdy) = 0
Z Z Z cia
er
dx + tan(xy) d(xy) = 0dx ←− take integral of both sides
m
x + sec2 (xy) = C
om

 
4. y x3 exy − y dx + x y + x3 exy dy = 0
Solution:
C

 
y x3 exy − y dx + x y + x3 exy dy = 0 ←− distribute the terms
or

yx3 exy dx − y 2 dx + xydy + x4 exy dy = 0 ←− regroup terms with exy


x3 exy (ydx + xdy) + y(xdy − ydx) = 0 ←− divide by x3
tF

  
xy y xdy − ydx
e (ydx + xdy) + =0
x x2
No

Z Z     Z
xy y y
e d(xy) + d = 0dx ←− take integral of both sides
x x
 2
xy 1 y
e + = c1 or 2x2 exy + y 2 = Cx2 , C = 2c1
2 x
51

Determination of Integrating Factor

We have already mentioned in the previous lesson the aid of integrating factor in solving a
first-order linear differential equation and that the same idea sometimes works for a nonexact
differential equation

M (x, y)dx + N (x, y)dy = 0 (1).

We mean to say that it is sometimes possible to determine an integrating factor u(x, y)


such that after multiplying it to a given nonexact equation (1), the left side of the resulting
equation
u(x, y)M (x, y)dx + u(x, y)N (x, y)dy = 0 (2)

e
becomes an exact differential.

Us
To determine such integrating factor u(x, y), recall the criterion for exactness. Equation (2)
is exact if and only if

l
∂ ∂

∂M
∂y
(uM ) =
∂u
∂x
∂N cia
(uN )
∂u
er
u +M =u +N
∂y ∂y ∂x ∂x
 
m
∂M ∂N ∂u ∂u
or u − =N −M (3)
∂y ∂x ∂x ∂y
om

∂M ∂N
If u satisfies (3), then u is an integrating factor for (1). Note that M, N, , and are
C

∂y ∂x
known functions of x and y. To determine such function u, from (3), we have to solve for
or

a partial differential equation, but were not yet ready to do that. Thus, we first make an
assumption that u is a function of only one variable.
tF

∂u du ∂u
Let us first assume that u depends on x alone. Then, = and = 0. Hence, (3)
∂x dx ∂y
becomes
No

 
∂M ∂N du
u − =N
∂y ∂x dx
 
du 1 ∂M ∂N
= − dx (4)
u N ∂y ∂x
It is still not easy to determine u even after taking the integral of both sides of (4) if its
right side dependson both x and
 y. However, if after some algebraic simplications are made,
1 ∂M ∂N
the expression − turns out to be dependent alone on the variable y, then (4)
N ∂y ∂x
is now a first-order ordinary differential equation and we can finally solved for u in (4) by
separation of variables. That is,
Z Z  
du 1 ∂M ∂N
= − dx
u N ∂y ∂x
52
Z  
∂M 1 ∂N
− dx
Solving for u, we have u(x) = e ∂y N ∂x .
Similarly, if u depends on y alone, then we will have,
 
∂M ∂N du
u − = −M
∂y ∂x dy
 
du 1 ∂M ∂N
=− − dy
u M ∂y ∂x
Z  
1 ∂M ∂N
− − dy
Solving for u, u(y) = e M ∂y ∂x
To summarize, given a nonexact differential equation

e
M (x, y)dx + N (x, y)dy = 0 (1)

Us
 
1 ∂M ∂N
• If − is a function of x alone, then an integrating factor for (1) is

l
N ∂y ∂x
Z
1

∂M
−cia
∂N

dx
er
u(x) = e N ∂y ∂x (5)
m
 
1 ∂M ∂N
• If −
om

− is a function of y alone, then an integrating factor of (1) is


M ∂y ∂x
Z  
1 ∂M ∂N
C

− − dy
u(y) = e M ∂y ∂x (6)
or

Remarks 5. If neither of the two works, we can only say that (1) does not have an integrating
tF

factor that is a function of x or y alone.


No
53

Example 16. Solve the following differential equation or IVP by finding an appropriate
integrating factor.

1. 2y 2 + 3x dx + 2xydy = 0

Solution: We identify M = 2y 2 + 3x and N = 2xy. Then


∂M ∂ ∂N ∂
= [2y 2 + 3x] = 4y and = [2xy] = 2y
∂y ∂y ∂x ∂x
∂M ∂N
− = 4y − 2y = 2y
∂y ∂x
Notice that the difference between the partial derivatives has only one term as well as
N . (It could be a deciding factor, whether to try to divide it by N or M .)
 

e
1 ∂M ∂N 1 1 R
=⇒ IF: u(x) = e dx/x = eln x = x

Us
− = (2y) =
N ∂y ∂x 2xy x
Multiply the original DE by IF = x


l
New differential equation: 2xy 2 + 3x2 dx + 2x2 ydy = 0

Test for Exactness:



cia
[2xy 2 + 3x2 ] = 4xy and

[2x2 y] = 4xy
er
∂y ∂x
Hence, the resulting equation is now exact. We now determine a solution f (x, y) = C
m
∂f ∂f
for the exact differential equation such that = 2xy 2 + 3x2 and = 2x2 y.
om

∂x ∂y
∂f
Let us start with = 2xy 2 + 3x2 . Integrate this last equation with respect to x.
∂x
C

Z Z
∂f
dx = f (x, y) = (2xy 2 + 3x2 ) dx ←− treat y as constant
or

∂x
Z Z
tF

2
f (x, y) = y 2x dx + 3x2 dx

f (x, y) = y 2 (x2 ) + x3 + g(y) ←− g(y) is the constant of integration


No

f (x, y) = x2 y 2 + x3 + g(y)

Take the partial derivative of the last equation with respect to y


∂ ∂  2 2 
f (x, y) = x y + x3 + g(y)
∂y ∂y
∂ ∂f
f (x, y) = 2x2 y + g 0 (y) ←− but = 2x2 y
∂y ∂y
2x2 y + g 0 (y) = 2x2 y =⇒ g 0 (y) = 0 =⇒ g(y) = c1

Therefore, f (x, y) = x2 y 2 + x3 + c1 ⇐⇒ x2 y 2 + x3 = C .
54

2. 10 − 6y + e−3x dx − 2dy = 0

Solution: We identify M = 10 − 6y + e−3x and N = −2. Then


∂M ∂ ∂N ∂
= [10 − 6y + e−3x ] = −6 and = [−2] = 0
∂y ∂y ∂x ∂x
∂M ∂N
− = −6 − 0 = −6
∂y ∂x
 
1 ∂M ∂N 1 R
− = (−6) = 3 =⇒ IF: u(x) = e 3dx = e3x
N ∂y ∂x −2
Multiply the original DE by IF = e3x

New differential equation: 10e3x − 6ye3x + 1 dx − 2e3x dy = 0

e
∂ ∂

Us
Test for Exactness: [10e3x − 6ye3x + 1] = −6e3x and [−2e3x ] = −6e3x
∂y ∂x
Hence, the resulting equation is now exact. We now determine a solution f (x, y) = C

l
∂f ∂f

cia
for the exact differential equation such that = 10e3x − 6ye3x + 1 and = −2e3x .
∂x ∂y
∂f
Let us start with = −2e3x . Integrate this last equation with respect to y.
er
∂y
Z Z
m
∂f 
dy = f (x, y) = − 2e3x dy ←− treat x as constant
om

∂y
Z
3x
f (x, y) = −2e dy
C

f (x, y) = −2e3x y + h(x) ←− h(x) is the constant of integration


f (x, y) = −2e3x y + h(x)
or
tF

Take the partial derivative of the last equation with respect to x


∂ ∂  
f (x, y) = − 2e3x y + h(x)
No

∂x ∂x
∂ ∂f
f (x, y) = −6e3x y + h0 (x) ←− but = 10e3x − 6ye3x + 1
∂x ∂x
−6e3x y + h0 (x) = 10e3x − 6ye3x + 1 =⇒ h0 (x) = 10e3x + 1
Z
 10
h(x) = 10e3x + 1 dx = e3x + x + c1
3

10 3x 10 3x
Therefore, f (x, y) = −2e3x y + e + x + c1 ⇐⇒ -2e3x y + e +x=C .
3 3
55

3. y(x + y + 1)dx + x(x + 3y + 2)dy = 0

Solution: We identify M = y(x + y + 1) and N = x(x + 3y + 2). Then


∂M ∂ ∂N ∂
= [y(x + y + 1)] = x + 2y + 1 and = [x(x + 3y + 2)] = 2x + 3y + 2
∂y ∂y ∂x ∂x
∂M ∂N
− = (x + 2y + 1) − (2x + 3y + 2) = −x − y − 1
∂y ∂x
 
1 ∂M ∂N −x − y − 1
Notice that − = is not a function of x alone, but
N ∂y ∂x x(x + 3y + 2)
 
1 ∂M ∂N −x − y − 1 −(x + y + 1) 1
− − = = = is a function of y alone.
M ∂y ∂x −y(x + y + 1) −y(x + y + 1) y
R

e
Hence, IF=u(y) = e dy/y = eln y = y

Us
Multiply the original DE by IF = y
 
New differential equation: xy 2 + y 3 + y 2 dx + x2 y + 3xy 2 + 2xy dy = 0

l
cia
∂ ∂ 2
Test for Exactness: [xy 2 + y 3 + y 2 ] = 2xy + 3y 2 + 2y and [x y + 3xy 2 + 2xy] =
∂y ∂x
2xy + 3y 2 + 2y
er
Hence, the resulting equation is now exact. We now determine a solution f (x, y) = C
m
∂f ∂f
for the exact differential equation such that = xy 2 + y 3 + y 2 and = x2 y + 3xy 2 +
∂x ∂y
om

2xy.
∂f
Let us start with = xy 2 + y 3 + y 2 . Integrate this last equation with respect to x.
C

∂x
Z Z
∂f
dx = f (x, y) = (xy 2 + y 3 + y 2 ) dx ←− treat y as constant
or

∂x
Z Z Z
tF

2 3 2
f (x, y) = y x dx + y dx + y dx
 2
x
f (x, y) = y 2 + y 3 x + y 2 x + g(y) ; g(y) is the constant of integration
No

2
1 2 2
f (x, y) = x y + xy 3 + xy 2 + g(y)
2
Take the partial derivative of the last equation with respect to y
 
∂ ∂ 1 2 2 3 2
f (x, y) = x y + xy + xy + g(y)
∂y ∂y 2
∂ ∂f
f (x, y) = x2 y + 3xy 2 + 2xy + g 0 (y) ←− but = x2 y + 3xy 2 + 2xy
∂y ∂y
x2 y + 3xy 2 + 2xy + g 0 (y) = x2 y + 3xy 2 + 2xy =⇒ g 0 (y) = 0 =⇒ g(y) = c1

1 1 2 2
Therefore, f (x, y) = x2 y 2 + xy 3 + xy 2 + c1 ⇐⇒ x y + xy 3 + xy 2 = C .
2 2
56

Activity 3

A. Solve the following differential equation by the indicated method.


dy √
1. = 2 + y − 2x + 3 (reduction to separation of variables)
dx
2. (6x − 3y + 2)dx − (2x − y − 1)dy = 0 (coefficients linear in two variables)

3. (y − 2)dx − (x − y − 1)dy = 0 (coefficients linear in two variables)


dy
4. x2 + y 2 = xy (Bernoulli DE)
dx
5. y(x + y)dx + (x + 2y − 1)dy = 0 (determination of integrating factor)

e
Us
B. Solve the following initial-value problems.
dy π
1. = cos(x + y) y(0) =
dx 4

l
cia

2. 4xy + 3x2 dx + 2y + 2x2 ) = 0, y(0) = −2
er
m
C om
or
tF
No
57

Chapter 3: Higher-Order Differential Equa-


tions
Chapter Overview
In this chapter, we discuss the solution of ordinary differential equations of order
two or higher. We determine methods in solving homogeneous and non-homogeneous
differential equations.

Learning Objectives
At the end of this chapter, the student is expected to

e
Us
1. manifest skills in solving homogeneous differential equations with constant coef-
ficients using appropriate methods

l
cia
2. solve nonhomogeneous differential equations with appropriate methods (unde-
termined coefficients, variation of parameters)
er
m
In Chapter 1 Lesson 1.4, we defined an initial-value problem for a general nth-order dif-
ferential equation. Given a linear differential equation, we have an nth-order initial-value
om

problem of the form


C

dn y dn−1 y dy
Solve: an (x) n
+ a n−1 (x) n−1
+ · · · + a1 (x) + a0 (x)y = g(x)
dx dx dx (1)
or

0 n−1
Subject to : y(x0 ) = y0 , y (x0 ) = y1 , . . . , y (x0 ) = yn−1
tF

Theorem 1: Existence of a Unique Solution


No

Let an (x), an−1 (x), . . . , a1 (x), a0 (x) and g(x) be continuous on an interval I, and let
an (x) 6= 0 for every x in this interval. If x = x0 is any point in this interval, then a
solution y(x) of the initial-value problem (1) exists on the interval and is unique.

A problem of the form

d2 y dy
Solve: a2 (x) + a 1 (x) + a0 (x)y = g(x)
dx2 dx (2)
Subject to : y(a) = y0 , y(b) = y1

is called a boundary-value problem (BVP) where y(a) = y0 and y 0 (b) = y1 are called
the boundary conditions. In a boundary-value problem (which consists of solving linear
58

differential equations of order two or higher), the dependent variable y or its derivatives
are specified at different points. For a second-order differential equations, the boundary
conditions could also be
y 0 (a) = y0 , y(b) = y1
y(a) = y0 , y 0 (b) = y1
y 0 (a) = y0 , y 0 (b) = y1

Example 1.

1. Consider the following IVP:

y” − 4y = 12x, y(0) = 4, y 0 (0) = 1

e
Us
By some methods which will be discussed in this chapter, we will be able to solve for
a solution of the given IVP, that is, the function y = 3e2x + e−2x − 3x.

l
cia
Note that in the given linear differential equation, the coefficients as well as the right
side g(x) = 12x are all continuous and a2 (x) = 1 6= 0 on any interval I containing x = 0.
er
Therefore we can say that (from Theorem 1), the given function y = 3e2x + e−2x − 3x
m
is the unique solution of the given IVP on I.
om

2. Consider the two-parameter family of solutions y = c1 ex cos x + c2 ex sin x of the differ-


ential equation, y 00 − 2y 0 + 2y = 0. Determine the solution of the differential equation
C

for each given set of boundary conditions.


or

• y(0) = 1, y 0 (π) = 0
tF

Solution: Observe that the first condition 1 = c1 e0 cos 0 + c2 e0 sin 0 =⇒ c1 = 1.


 
Note that y 0 = ex − c1 sin x + c2 cos x + ex c1 cos x + c2 sin x . Using the second
 
condition and since c1 = 1, 0 = eπ −c1 cos π+c2 cos π +eπ c1 cos π+c2 sin π =⇒
No

c2 = −1. Thus, y = ex cos x − ex sin x is a solution of the boundary-value problem


y 00 − 2y 0 + 2y = 0, y(0) = 1, y 0 (π) = 0.
• y(0) = 1, y(π) = −1
Solution: The first condition still implies that c1 =. Now, using c1 = and y(π) =
−1, we will have −1 = eπ cos π + c2 eπ sin P = eπ cos π which is a contradiction.
Hence, the boundary-value problem y 00 − 2y 0 + 2y = 0, y(0) = 1, y(π) = −1 has
no solution.
59

• y(0) = 0, y(π) = 0
Solution: The first condition 0 = c1 e0 cos 0 + c2 e0 sin 0 =⇒ c1 = 0. With c1 = 0
and from the second condition, 0 = c2 sin 0 is satisfied for any choice of c2 . Hence,
the boundary-value problem y 00 − 2y 0 + 2y = 0, y(0) = 0, y(π) = 0 has infinitely
many solutions of the form y = c2 ex sin x.

Remarks 1.

1. If the conditions in Theorem 1 are not satisfied, then the solution of a linear differential
equation may not be unique or even exist.

2. A BVP can have many, one, or no solutions.

e
Us
3.1 Homogeneous Linear Equations with Constant Coefficients

l
Definition 1
A linear nth-order differential equation of the form
cia
er
an (x)y (n) + an−1 (x)y (n−1) + · · · + a1 y 0 + a0 (x)y = 0 (3)
m
om

is said to be homogeneous, whereas the differential equation

an (x)y (n) + an−1 (x)y (n−1) + · · · + a1 (x)y 0 + a0 (x)y = g(x), (4)


C
or

with g(x) 6= 0, is said to be nonhomogeneous.


tF

Note: The term homogeneous in this context does not refer to coefficients that are homo-
No

geneous functions, as in Chapter 2 Lesson 2.4.

Theorem 2
Let y1 , y2 , . . . , yk be solutions of the homogeneous nth-order differential equation (3)
on an interval I. Then the linear combination

y = c1 y1 (x) + c2 y2 (x) + · · · ck yk (x), (5)

where the ci , i = 1, 2, 3, . . . , k are arbitrary constants, is also a solution on the interval.


60

Definition 2: Linear Dependence/Independence

A set of functions f1 (x), f2 (x), . . . , fn (x) is said to be linearly dependent on an


interval I if there exist constants c1 , c2 , . . . , cn , not all zero, such that

c1 f1 (x) + c2 f2 (x) + · · · + cn fn (x) = 0

for every x in the interval. If the set of functions is not linearly dependent on the
interval, it is said to be linearly independent.

Remarks 2.

e
1. In other words, a set of functions is linearly independent on an interval I if the only

Us
constants for which
c1 f1 (x) + c2 f2 (x) + · · · + cn fn (x) = 0

l
for ever x in the interval are c1 = c2 = · · · = cn = 0.
cia
er
2. A set of two functions f1 (x) and f2 (x) is linearly independent when neither function is
m
a constant multiple of the other on the interval.
om

3. A set of functions f1 (x), f2 (x), . . . , fn (x) is linearly dependent on an interval if at least


one function can be expressed as a linear combination of the remaining functions.
C

Example 2.
or

1. The set of functions f1 (x) = 5, f2 (x) = cos2 x andf3 (x) = sin2 x is linearly dependent
tF

on the interval (−∞, ∞) since 5c1 + c2 cos2 x + c3 sin2 x = 0 when c1 = 1, c2 = c3 = −5.

2. The set of functions f1 (x) = x, f2 (x) = x + 1 andf3 (x) = x2 is linearly independent on


No

the interval (−∞, ∞).

Definition 3: Wronskian
Suppose each of the functions f1 (x), f2 (x),...,fn (x) possesses at least n − 1 derivatives.
The determinant
f1 f2 ··· fn
 f10 f20 ··· fn0
W f1 , f2 , . . . , fn = .. .. ..
. . ··· .
(n−1) (n−1)
f1 f2 ··· fn (n−1)
where primes denote derivatives, is called the Wronskian of the functions.
61

Theorem 3: Criterion for Linearly Independent Solutions


Let y1 , y2 , . . . , yn be n solutions of the homogeneous linear nth -order differential equa-
tion (3) on an interval I. Then the set of solutions is linearly independent on I if

and only if W f1 , f2 , . . . , fn 6= 0 for every x in the interval.

Definition 4: Fundamental Set of Solutions


Any set y1 , y2 , . . . , yn of n linearly independent solutions of the homogeneous linear
nth-order differential equation (3) on an interval I is said to be a fundamental set

e
of solutions on the interval.

Us
Theorem 4: General Solution - Homogeneous Equations

l
cia
Let y1 , y2 , . . . , yn be a fundamental set of solutions of the homogeneous linear nth-order
differential equation (3) on an interval I. Then general solution of the equation on
er
the interval is
m
y = c1 y1 (x) + c2 y2 (x) + · · · + cn yn (x),
om

where ci , i = 1, 2, . . . , n are arbitrary constants.


C

Method of Solution
or

Observe that if we substitute y = emx and y 0 = memx to the first-order DE ay 0 + by = 0, we


tF

have
amemx + bemx = 0 ⇐⇒ emx (am + b) = 0.
No

Since emx is never zero for all real values of x, the last equation is satisfied only when m is
a solution or root of the first-degree polynomial equation am + b = 0.

Example 3. Given the first-order DE: 2y 0 + 5y = 0. In this case, we just need to solve for
5
m in the equation 2m + 5 = 0, which gives m = − . Therefore, a particular solution to the
2
− 52 x 5
the DE is y = e . The general solution is y = c1 e− 2 x .

The procedure gives us an idea that we can find exponential solutions for homogeneous linear
higher-order DE.
62

Auxiliary Equation
Given the homogeneous linear higher-order DE

an (x)y (n) + an−1 (x)y (n−1) + · · · + a1 y 0 + a0 (x)y = 0

The corresponding auxiliary equation of the above DE is

an mn + an−1 mn−1 + · · · + a1 m + a0 = 0 (6)

Now, consider the homogeneous linear second-order DE

e
ay 00 + by 0 + c = 0 (7)

Us
where a, b, and, c are constants. The corresponding auxiliary equation of (7) is

l
cia
am2 + bm + c = 0 (8)
er
which is a quadratic equation in variable m. By quadratic formula, the roots of the quadratic
m
equation (8) are
√ √
om

−b + b2 − 4ac −b − b2 − 4ac
m1 = and m2 =
2a 2a
C

Recall from algebra that these roots will generate three cases (for the general solution of the
or

DE):
tF

1. (discriminant: b2 − 4ac > 0) m1 and m2 are real and distinct,


No

2. (discriminant: b2 − 4ac = 0) m1 and m2 are real and equal (repeated roots), and

3. (discriminant: b2 − 4ac < 0) m1 and m2 are conjugate complex numbers

CASE 1: DISTINCT REAL ROOTS


Assuming the quadratic equation (8) has two unequal roots m1 and m2 , then we find two
linearly independent solutions for the second-order DE (7) which are y1 = em1 x and y2 = em2 x .
It follows that the general solution of (7) is

y = c1 em1 x + c2 em2 x

Notice that we can extend this to homogeneous linear higher-order DE if all the roots of the
63

corresponding auxiliary equation are real and distinct. The general solution is

y = c1 em1 x + c2 em2 x + · · · + cn−1 emn−1 x + cn emn x

CASE 2: REPEATED REAL ROOTS


When m1 = m2 , we necessarily obtain one exponential solution, y = em1 x . From the
−b
quadratic formula we find that m1 = since the only way to have m1 = m2 is to have
2a
b2 − 4ac = 0. It follows that the second solution y2 will be
Z Z
m1 x e2m1 x
y2 = e dx = em1 x = dx = xem1 x .
e2m1 x

e
Thus, the general solution for the second-order DE (7) is

Us

y = c1 em1 x + c2 xem1 x or y = em1 x c1 + c2 x

l
cia
For the homogeneous linear higher-order DE, if all the k roots of the corresponding auxiliary
er
equation are equal, the general solution is
m
y = c1 em1 x + c2 xem1 x + c3 x2 em1 x + · · · + ck xk−1 em1 x or

om

y = em1 x c1 + c2 x + c3 x2 + · · · + ck xk−1
C

CASE 3: CONJUGATE COMPLEX ROOTS


If m1 and m2 are conjugate complex numbers, then we can write m1 = α+iβ and m2 = α−iβ,
or

where α, β > 0 are real numbers and i2 = −1. Formally, there is no difference between this
tF

case and CASE 1, hence the general solution is


No

y = c1 e(α+iβ)x + c2 e(α−iβ)x

Important Note:
In this course, we prefer to work with real functions rather that the complex exponen-
tials. To do this, we use the Euler’s formula:

eiθ = cos θ + i sin θ,

where θ is any real number. It follows that

eiβx = cos βx + i sin βx and e−iβx = cos βx − i sin βx (9)


64

We reduce the general solution by some algebraic manipulations, thus

y = c1 eαx cos(βx) + c2 eαx sin(βx)


y = eαx [c1 cos(βx) + c2 sin(βx)]

CASE 4 (CASE 2 + CASE 3): In the case when the auxiliary equation has repeated
complex roots, that is, if m1 = α + iβ, β > 0 is a complex root of multiplicity k of an
auxiliary equation with real coefficients, then its conjugate m2 = α − iβ is also a root of
multiplicity k. With the aid of Euler’s formula, the general solution of the homogeneous
linear higher order DE must contain a linear combination of 2k real linearly independent
solutions:

e
eαx cos(βx), xeαx cos(βx), x2 eαx cos(βx), . . . , xk−1 eαx cos(βx),

Us
eαx sin(βx), xeαx sin(βx), x2 eαx sin(βx), . . . , xk−1 eαx sin(βx)
Thus we have the general solution,

l
y = eαx

cia 
c1 + c2 x + c3 x2 + · · · + ck xk−1 cos(βx)
er

+ ck+1 + ck+2 x + ck+3 x2 + · · · + c2k xk−1 sin(βx)
m

Remarks 3. Any linear differential equation can be expressed in terms of the D notation.
om

The symbol D is called a differential operator which transforms a differentiable function


into another function.
C

Example 4. The differential equation y 00 +5y 0 +6y = 0 can be written as D2 y +5Dy +6y = 0
or

with corresponding auxiliary equation m2 + 5m + 6 = 0.


tF

Example 5. Solve the following differential equations. If the operator D is used, consider
that the independent variable is x.
No

1. 2y 00 − 5y 0 − 3y = 0

Solution: We give the auxiliary equations, the roots, and the general solution.

2m2 − 5m − 3 = 0 ←− auxiliary equation


(2m + 1)(m − 3) = 0 ←− determine the roots by factoring
1 1
m1 = − , m2 = 3 ←− Case 1, similarly we can have m1 = 3 and m2 = −
2 2
∴ y = c1 e−x/2 + c2 e3x ←− Note: We can also have y = c1 e3x + c2 e−x/2
65

2. y 00 − 10y 0 + 25 = 0

Solution: We give the auxiliary equations, the roots, and the general solution.

m2 − 10m + 25 = 0 ←− auxiliary equation


(m − 5)2 = 0 ←− determine the roots by factoring
m1 = m2 = 5 ←− Case 2

∴ y = c1 e5x + c2 xe5x ←− Note: We can also have y = e5x c1 + c2 x

3. y 00 + 4y 0 + 7y = 0

e
Us
Solution: We give the auxiliary equations, the roots, and the general solution.

m2 + 4m + 7 = 0 ←− auxiliary equation

l
√ √

cia
m1 = −2 + 3i, m2 = −2 − 3i ←− Case 3 (by quadratic formula)

α = −2, β = 3
er
h √ √ i
∴ y = e−2x c1 cos( 3x) + c2 sin( 3x)
m
om

4. y 000 + 3y 00 − 4y = 0
C

Solution: We give the auxiliary equations, the roots, and the general solution.
or

m3 + 3m2 − 4 = 0 ←− auxiliary equation


tF

(m − 1)(m2 + 4m + 4) = 0 ←− by factoring or by synthetic division


(m − 1)(m + 2)2 = 0
No

m1 = 1, m2 = m3 = −2 ←− by Case 1 and Case 2



∴ y = c1 ex + c2 e−2x + c3 xe−2x or y = c1 ex + e−2x c2 + c3 x
66

5. IVP: 4y 00 + 4y 0 + 17y = 0, y(0) = −1, y 0 (0) = 2

Solution: We give the auxiliary equations, the roots, the general solution, and the
solution of the IVP.

4m2 + 4m + 17 = 0 ←− auxiliary equation


1 1
m1 = − + 2i, m2 = − − 2i ←− Case 3 (by quadratic formula)
2 2
1
α=− , β=2
2
−x/2
y=e [c1 cos(2x) + c2 sin(2x)] ←− General solution

e
For the IVP: Since y(0) = −1, then x = 0 and y = −1. We have,

Us
−1 = e0 [c1 cos 0 + c2 sin 0] =⇒ c1 = −1

l
=⇒ y = e−x/2 [− cos(2x) + c2 sin(2x)]

cia
Now, differentiating y = e−x/2 [− cos(2x) + c2 sin(2x)], we have
er
   
m
0 −x/2 1 −x/2 1
y =e 2 sin(2x) + cos(2x) + c2 e 2 cos(2x) − sin(2x)
2 2
om

Using y 0 (0) = 2, then x = 0 and y 0 = 2. We have,


C

   
0 1 0 1
2 = e 2 sin(0) + cos(0) + c2 e 2 cos(0) − sin(0)
or

2 2
1
tF

2 = 2c2 +
2
3
c2 =
No

Therefore, the solution of the IVP is


 
−x/2 3
y=e −2 cos(2x) + sin(2x)
4
67

6. y (4) + 2y 00 + y = 0

Solution: We give the auxiliary equations, the roots, and the general solution.

m4 + 2m2 + 1 = 0 ←− auxiliary equation


(m2 + 1)2 ←− by factoring
m1 = m3 = i and m2 = m4 = −i ←− by Case 4
α = 0, β = 1
y = e0 (c1 cos x + c2 sin x) + xe0 (c3 cos x + c4 sin x)
 
∴ y = c1 cos x + c2 sin x + x(c3 cos x + c4 sin x) or y = c1 +c2 x cos x+ c3 +c4 x sin x

e
Us

7. D4 + D3 + D2 y = 0

l
Solution: We give the auxiliary equations, the roots, and the general solution.

m4 + m3 + m2 = 0 ←− auxiliary equation cia


er

m2 m2 + m + 1 = 0 ←− by factoring and quadratic formula
m
√ √
1 3 1 3
om

m1 = m2 = 0 m3 = − + i m4 = − − i ←− by Case 2 and Case 3


2√ 2 2 2
1 3
α=− , β=
C

2 2
 √   √ 
or

0x −x/2 3 3
y = e c1 + c2 x) + e c3 cos x + c4 sin x
2 2
tF

 √   √ 
3 3
∴ y = c1 + c2 x + e −x/2
c3 cos x + c4 sin x
2 2
No

d5 u d4 u d3 u d2 u du
8. + 5 − 2 − 10 + + 5u = 0
dr5 dr4 dr3 dr2 dr
Solution: We give the auxiliary equations, the roots, and the general solution.

m5 + 5m4 − 2m3 − 10m2 + m + 5 = 0 ←− auxiliary equation


(m + 5)(m + 1)2 (m − 1)2 = 0 ←− by synthetic division
m = −5 m = −1(multiplicity 2) m = 1(multiplicity 2) ←− by Case 1 and Case 2
 
∴ u = c1 e−5r + e−r c2 + c3 r + er c4 + c5 r
68

3.2 Nonhomogeneous Higher Order Differential Equations

Theorem 5: General Solution of Nonhomogenous Equations

Let yp (x) be any particular solution of the nonhomogeneous linear nth-order dif-
ferential equation (4), and let y1 , y2 , . . . , yn be a fundamental set of solutions of the
associated homogeneous differential equation (3). Then the general solution of the
equation is

y = c1 y1 (x) + c2 y2 (x) + · · · + cn yn (x) + yp (x) = yc (x) + yp (x)

where the ci , i = 1, 2, . . . , n are arbitrary constants and yc (x) is called the comple-

e
Us
mentary function which is the general solution of (3).

l
cia
Method of Undetermined Coefficients − Annihilator Approach
er
Differential Operator
m
In calculus, differentiation is often denoted by the capital letter D, that is,
om

dy dn y
Dy = , or generally, Dn y = n .
dx dx
C

The symbol D is called a differential operator because it transforms a differentiable func-


or

tion into another function.


Polynomial expressions involving D, such as D+3, D2 −4D+2, and 5x3 D3 −6x2 D2 +4xD+9
tF

are also differential operators. For example, if we treat D as an algebraic quantity, then the
operator D2 + 5D + 6 can be factored as (D + 2)(D + 3) or as (D + 2)(D + 3). Thus if a
No

function y = f (x) possesses a second derivative, then

(D2 + 5D + 6)y = (D + 2)(D + 3)y = (D + 3)(D + 2)y.

Furthermore, a differential equation such as y 00 + 4y 0 + 4y = 0 can be written as

(D2 + 4D + 4)y = 0 or (D + 2)(D + 2)y = 0 or (D + 2)2 y = 0.

In general, we define an nth-order differential operator or polynomial operator L to


be
L = an (x)Dn + an−1 (x)Dn−1 + · · · + a1 (x)D + a0 (x).
69

Using this, we can write

(homogeneous DE) an (x)y (n) + an−1 (x)y (n−1) + · · · + a1 y 0 + a0 (x)y = 0; and


(nonhomogeneous DE) an (x)y (n) + an−1 (x)y (n−1) + · · · + a1 (x)y 0 + a0 (x)y = g(x)

as L(y) = 0 and L(y) = g(x), respectively.

Remarks 4. The differential operator L is a linear operator, that is, L(y) satisfies the
linearity property
L[af (x) + bg(x)] = aL(f (x)) + bL(g(x)),

e
Us
where a and b are constants.

Annihilator Operator

l
cia
If L is a linear differential operator with constant coefficients and y = f (x) is a sufficiently
differentiable function such that
er
L(f (x)) = 0,
m

then L is said to be an annihilator of the function y = f (x).


C om

Remarks 5.
or

5.1 The differential operator L = Dn annihilates each of the functions


tF

1, x, x2 , . . . , xn−1
No

This means that a polynomial function in x can be annihilated by finding an operator


that annihilates the highest power of x.

5.2 The differential operator L = (D − α)n annihilates each of the functions

eαx , xeαx , x2 eαx , . . . , xn−1 eαx

To see this, note that the auxiliary equation of the homogeneous equation
(D − α)n y = 0 is (m − α)n = 0. Since α is a root of multiplicity n, the general solution
is
y = c1 eαx + c2 xeαx + · · · + cn xn−1 eαx .
70

5.3 The differential operator L = [D2 − 2αD + (α2 + β 2 )]n annihilates each of the functions

eαx cos (βx), xeαx cos (βx), x2 eαx cos (βx), . . . , xn−1 eαx cos (βx)

eαx sin (βx), xeαx sin (βx), x2 eαx sin (βx), . . . , xn−1 eαx sin (βx)

Example 6. Find a differential operator L that annihilates the given function.

1. f (x) = 8x3 − 5x2 + 1

Solution: By Remarks 5.1, we know that D4 (x3 ) = 0. It follows that


D4 (8x3 − 5x2 + 1) = 0 and hence, L = D4 annihilates f (x) = 8x3 − 5x2 + 1.

e
Us
2. f (x) = xe−3x

Solution: By Remarks 5.2, with α = −3 and n = 2, we see that (D + 3)2 (x2 e−3x ) = 0

l
cia
or (D2 + 6D + 9)(xe−3x ) = 0. Therefore, L = (D + 3)2 annihilates f (x) = xe−3x .
To verify,
er
(D2 + 6D + 9)(xe−3x ) = D2 (xe−3x ) + 6D(xe−3x ) + 9(xe−3x )
m

= D[x(−3e−3x ) + e−3x (1)] + 6[x(−3e−3x ) + e−3x (1)] + 9xe−3x


om


  
= D(−3xe−3x + e−3x ) −  
18xe −3x
+ 6e−3x +  −3x
9xe
= −3D(xe−3x ) + D(e−3x ) − 9xe−3x + 6e−3x
C

  
= −3[x(−3e−3x ) + e−3x (1)] + 
(−3e 6e
−3x ) − 9xe−3x +  −3x
or

= 9xe−3x − 3e−3x + 3e−3x − 9xe−3x


tF

(D2 + 6D + 9)(xe−3x ) = 0
No

3. f (x) = 4e5x − 10xe5x + 7x2 e5x

Solution: By Remarks 5.2, with α = 5 and n = 3, we have


(D − 5)3 (4e5x − 10xe5x + 3x2 e5x ) = 0. Hence, L = (D − 5)3 annihilates
f (x) = 4e5x − 10xe5x + 7x2 e5x

4. f (x) = 5xe−x cos(2x) − 9xe−x sin(2x)

Solution: By Remarks 5.3, with α = −1, β = 2, and n = 2, we have


[D2 − 2(−1)D + (−1)2 + (2)2 ]2 = (D2 + 2D + 5)2 . Thus,
(D2 + 2D + 5)2 [5xe−x cos(2x) − 9xe−x sin(2x)] = 0 and L = (D2 + 2D + 5)2 annihilates
f (x) = 5xe−x cos(2x) − 9xe−x sin(2x).
71

Method of Undetermined Coefficients

Before proceeding, recall that the general solution of a nonhomogeneous linear DE L(y) =
g(x) is y = yc + yp , where yc is the complementary function or the general solution of the
associated homogeneous function L(y) = 0.

Now, the procedure for determining yp is called the method of undetermined coeffi-
cients. From Remarks 5, we know that g(x) can be annihilated by a differential operator
L1 of lowest order. Applying L1 to both sides of L(y) = g(x) yields L1 L(y) = L1 (g(x)) ⇐⇒
L1 L(y) = 0. By solving the homogeneous higher-order DE L1 L(y) = 0, we can find the
form of a particular solution yp for the original nonhomogeneous DE L(y) = g(x). We then

e
Us
substitute this assumed form into L(y) = g(x) to find an explicit particular solution yp . This
method will be illustrated in the following examples.

l
cia
Remarks 6. Basically, this method of undetermined coefficients is limited to non homoge-
er
neous linear equations with constant coefficients and g(x) is a constant, a polynomial, an
exponential of the form eαx , a sine or a cosine function sin βx or cos βx, or a finite sums or
m

products of these functions.


om

Example 7. Find the general solution y = yc + yp of the following differential equations.


C

1. y 00 + 3y 0 + 2y = 4x2
or

Solution:
tF

Step 1. First, we solve the associated homogeneous equation y 00 + 3y 0 + 2y = 0. We


give the auxiliary equations, the roots, and the complementary function yc .
No

m2 + 3m + 2 = (m + 1)(m + 2) = 0
m1 = −1 and m2 = −2
yc = c1 e−x + c2 e−2x

Step 2. We now determine yp by method of undetermined coefficients. Since g(x) =


4x2 is annihilated by the differential operator L1 = D3 , we see that
72

D3 (y 00 + 3y 0 + 2y) = D3 (4x2 )
D3 (D2 + 3D + 2)y = 0 ←− homogeneous higher-order DE
m3 (m2 + 3m + 2) = 0 ←− corresponding auxiliary equation
m3 (m + 1)(m + 2) = 0 ←− by factoring
m1 = m2 = m3 = 0, m4 = −1, m5 = −2 ←− roots of the auxiliary equation
y = c1 + c2 x + c3 x2 + c4 e−x + c5 e−2x ←− general solution y = yc + yp
| {z } | {z }
yp yc

Note that the terms c4 e−x + c5 e−2x constitute the complementary function yc . Hence,

e
Us
the remaining terms c1 + c2 x + c3 x2 constitute the form of yp . We can write yp as

yp = A + Bx + Cx2

l
cia
At this point, we need to find the specific values of the coefficients A, B, and C as
er
follows:
m
yp0 = B + 2Cx ← first derivative of yp
om

yp00 = 2C ← second derivative of yp


yp00 + 3yp0 + 2yp = 4x2 ← substitute yp , yp0 , yp00 to the given DE
C

Substitute values of yp , yp0 , and yp00


or
tF

(2C) + 3(B + 2Cx) + 2(A + Bx + Cx2 ) = 4x2


No

Group and equate the coefficients of powers of x

(2C)x2 + (2B + 6C)x + (2A + 3B + 2C) = 4x2 + 0x + 0


x2 : 2C = 4; x : 2B + 6C = 0; x0 : 2A + 3B + 2C = 0

Solving the above equations, we have A = 7, B = −6, and C = 2. Thus, yp =


7 − 6x + 2x2 .
Therefore, the general solution y = yc + yp is

y = c1 e−x + c2 e−2x + 7 − 6x + 2x2


73

2. y 00 − 3y 0 = 8e3x + 4 sin x
Solution:
Step 1. First, we solve the associated homogeneous equation y 00 − 3y 0 = 0. We give
the auxiliary equations, the roots, and the complementary function yc .

m2 − 3m = m(m − 3) = 0
m1 = 0 and m2 = 3
yc = c1 e0 + c2 e3x ⇐⇒ yc = c1 + c2 e3x

Step 2. We now determine yp by method of undetermined coefficients. Since (D −


3)e3x = 0 and (D2 + 1) sin x, we choose L1 = (D − 3)(D2 + 1). Then

e
Us
(D − 3)(D2 + 1)(y 00 − 3y 0 ) = (D − 3)(D2 + 1)(8e3x + 4 sin x)
(D − 3)(D2 + 1)(D2 − 3D)y = 0 ← homogeneous higher-order DE

l
cia
(m − 3)(m2 + 1)(m2 − 3m) = 0 ← corresponding auxiliary equation
m(m − 3)2 (m2 + 1) = 0
er
m1 = 0, m2 = m3 = 3, m4 = i, m5 = −i ← roots of auxiliary equation
m

y = c1 + c2 e3x + c3 xe3x + c4 cos x + c5 sin x ← general solution y = yc + yp


| {z } | {z }
om

yc yp

Note that the terms c1 + c2 e3x constitute the complementary function yc . Hence, the
C

remaining terms c3 xe3x + c4 cos x + c5 sin x constitute the form of yp . We can write yp
or

as
tF

yp = Axe3x + B cos x + C sin x

We find the specific values of the coefficients A, B, and C as follows:


No

yp0 = A(3xe3x + e3x ) + B(− sin x) + C cos x


yp0 = 3Axe3x + Ae3x − B sin x + C cos x
yp00 = 3A(3xe3x + e3x ) + A(3e3x ) − B cos x + C(− sin x)
yp00 = 9Axe3x + 6Ae3x − B cos x − C sin x
yp00 − 3yp0 = 8e3x + 4 sin x

(9Axe3x + 6Ae3x − B cos x − C sin x) − 3(3Axe3x + Ae3x − B sin x + C cos x)


= 8e3x + 4 sin x

3Ae3x + (−B − 3C) cos x + (3B − C) sin x = 8e3x + 4 sin x


74

Equating coefficients, we have

e3x : 3A = 8; cos x : −B − 3C = 0; sin x : 3B − C = 4

8 6 2
Solving the above equations, we obtain A = , B = , and C = − . Thus,
3 5 5
8 6 2
yp = xe3x + cos x − sin x
3 5 5

Therefore, the general solution y = yc + yp is

8 6 2
y = c1 + c2 e3x + xe3x + cos x − sin x

e
3 5 5

Us
3. y 00 + y = x cos x − cos x

l
Solution:
cia
Step 1. First, we solve the associated homogeneous equation y 00 + y = 0. We give the
er
auxiliary equations, the roots, and the complementary function yc .
m

m2 + 1 = 0
om

m1 = i and m2 = −i
C

yc = e0 (c1 cos x + c2 sin x) ⇐⇒ yc = c1 cos x + c2 sin x


or

Step 2. We now determine yp by method of undetermined coefficients. Verify that


the annihilator for g(x) = x cos x − cos x is L1 = (D2 + 1)2 . Then
tF

(D2 + 1)2 (y 00 + y) = (D2 + 1)(x cos x − cos x)


No

(D2 + 1)2 (D2 + 1)y = 0


(D2 + 1)3 y = 0 ← homogeneous DE
(m2 + 1)3 = 0 ← auxiliary equation
m1 = m3 = m5 = i,and m2 = m4 = m6 = −i ← roots of auxiliary equation

General solution y = yc + yp :

y = c1 cos x + c2 sin x + c3 x cos x + c4 x sin x + c5 x2 cos x + c6 x2 sin x


| {z } | {z }
yp yc
75

Note that the terms c1 cos x+c2 sin x constitute the complementary function yc . Hence,
the remaining terms c3 x cos x + c4 x sin x + c5 x2 cos x + c6 x2 sin x constitute the form
of yp .
We can write yp as

yp = Ax cos x + Bx sin x + Cx2 cos x + Ex2 sin x

We simplify and obtain

yp00 + yp = x cos x − cos x


4Ex cos x − 4Cx sin x + (2B + 2C) cos x + (−2A + 2E) sin x = x cos x − cos x

e
Us
Equating coefficients, we have

x cos x : 4E = 1; x sin x : −4C = 0; cos x : 2B+2C = −1; sin x : −2A+2E = 0

l
1
cia 1 1
Solving the above equations, we obtain A = , B = − , C = 0, and E = . Thus,
er
4 2 4
m
1 1 1
yp = x cos x − x sin x + x2 sin x
4 2 4
om

Therefore, the general solution y = yc + yp is


C

1 1 1
y = c1 cos x + c2 sin x + x cos x − x sin x + x2 sin x
or

4 2 4
tF

Remarks 7. The method of undetermined coefficients is not applicable to linear equations


with variable coefficients nor is applicable to linear equations with constant coefficients when
No

g(x) is a function such as

1
g(x) = ln x, g(x) = , g(x) = tan x, g(x) = sin−1 x,
x

and so on.
76

3.3 Variation of Parameters


Recall that the general solution of a nonhomogeneous linear higher order DE is y = yc +
yp . The method of Variation of Parameters is an alternative method to determine the
particular solution yp of a nonhomogeneous linear higher order DE. The complementary
function yc is similarly obtained from the associated homogeneous equation. In this lecture,
we restrict the use of variation of parameters to nonhomogeneous linear second-order DE

a2 (x)y 00 + a1 (x)y 0 + a0 (x)y = g(x).

First, transform the above equation into the standard form

e
y 00 + P (x)y 0 + Q(x)y = f (x).

Us
We use the assumption for yp as

l
cia
yp = u1 (x)y1 (x) + u2 (x)y2 (x),
er
where y1 and y2 form a fundamental set of solutions of the associated homogeneous equation.
m
We obtain u1 and u2 by integrating
om

W1 y2 f (x) W2 y1 f (x)
u01 = =− and u02 = = , respectively,
W W W W
C

where
or

y1 y2 0 y2 y1 0
W = , W1 = , W2 = .
y10 y20 f (x) y20 0
y1 f (x)
tF

The determinant W is called the Wronskian of y1 and y2 , denoted by W (y1 , y2 ).


No

Definition 5: Wronskian
Suppose each of the functions y1 , y2 , . . . , yn possesses at least n − 1 derivatives. The
determinant
y1 y2 ... yn
y10 y20 ... yn0
W (y1 , y2 , . . . , yn ) = y100 y200 ... yn00
.. .. ... ..
. . .
(n−1) (n−1) (n−1)
y1 y2 . . . yn
is called the Wronskian of the functions.
77

Example 8. Find the general solution y = yc + yp of the following DE using variation of


parameters.

1. y 00 − 4y 0 + 4y = (x + 1)e2x

Solution:
Step 1. First, we solve the associated homogeneous equation y 00 − 4y 0 + 4y = 0. We
give the auxiliary equations, the roots, and the complementary function yc .

e
Us
m2 − 4m + 4 = 0
(m − 2)2 = 0

l
m1 = m2 = 2

cia
yc = c1 e2x + c2 xe2x
er
Step 2. We now determine yp by method of variation of parameters. Note that
m
the given DE is already in the standard form. We identify f (x) = (x + 1)e2x , y1 =
om

e2x and y2 = xe2x . (Note: We can also use y1 = xe2x , and y2 = e2x which will give a
slight difference in the latter solution.) Now, we compute the Wronskian:
C

0 xe2x
W = W (y1 , y2 ) = W (e2x , xe2x ) W1 =
or

(x + 1)e2x 2xe2x + e2x


tF

e2x xe2x
W = W1 = −(x + 1)xe4x
2e2x 2xe2x + e2x
No

W = e2x (2xe2x + e2x ) − 2e2x (xe2x )


e2x 0
W = 2xe4x + e4x − 2xe4x W2 = 2x
2e (x + 1)e2x
W = e4x W2 = (x + 1)e4x

We now obtain u01 and u02 as follows,

W1 W2
u01 = u02 =
W W
(x + 1)x
e4x 
e4x
(x + 1)
u01 = − u02 =

e4x 
e4x
 
0 2 0
u1 = −x − x u2 = x + 1
78

Integrate u01 and u02 with respect to x, we have


Z Z
2 x 3 x2 x2
u1 = (−x − x)dx = − − and u2 = (x + 1)dx = + x.
3 2 2

Hence,
yp = u1 y1 + u2 y2
 3   2 
x x2 2x x
yp = − − e + + x xe2x
3 2 2
1 1
yp = x3 e2x + x2 e2x
6 2

Therefore, the general solution y = yc + yp is

e
Us
1 1
y = c1 e2x + c2 xe2x + x3 e2x + x2 e2x
6 2

l
Important Note: cia
er
Constants of Integration: When computing the indefinite integrals of
m

u01 and u02 , we need not introduce any constants of integration. This is because
om

y = yc + yp
C

= c1 y1 + c2 y2 + (u1 + a1 )y1 + (u2 + b1 )y2 , a1 , b1 are constants of integration


or

= (c1 + a1 ) y1 + (c2 + b1 ) y2 + u1 y1 + u2 y2
| {z } | {z }
constant constant
tF

= C1 y1 + C2 y2 + u1 y1 + u2 y2
No

2. 4y 00 + 36y = csc(3x)

Solution:
Step 1. First, we solve the associated homogeneous equation 4y 00 + 36y = 0. We give
the auxiliary equations, the roots, and the complementary function yc .

4m2 + 36 = 0
m2 + 9 = 0
m1 = 3i and m2 = −3i
yc = c1 cos(3x) + c2 sin(3x)
79

Step 2. We now determine yp by method of variation of parameters. We first write


the given DE into the standard form by dividing by 4 on both sides, thus we have

1
y 00 + 9y = csc(3x).
4
1
We identify f (x) = csc(3x), y1 = cos(3x), and y2 = sin(3x). Now, compute the
4
Wronskian:

0 sin(3x)
W = W (y1 , y2 ) = W (cos(3x), sin(3x)) W1 = 1
csc(3x) 3 cos(3x)
4

e
cos(3x) sin(3x) 1
 

Us
W = W1 = −  sin(3x)csc(3x)
−3 sin(3x) 3 cos(3x) 4
1
W = 3 cos2 (3x) − [−3 sin2 (3x)] W1 = −

l
4
W = 3[cos2 (3x) + sin2 (3x)]
cia
er
cos(3x) 0
W =3 W2 = 1
−3 sin(3x)
m
csc(3x)
4
1
om

W2 = cos(3x) csc(3x)
4
1 cos(3x)
W2 =
C

4 sin(3x)
or

We now obtain u01 and u02 as follows,


tF

W1 W2
u01 = u02 =
W W
No

1 1 cos(3x)
− 4 sin(3x)
u01 = 4 u02 =
3 3
1 1 cos(3x)
u01 = − u02 =
12 12 sin(3x)
80

Integrate u01 and u02 with respect to x, we have


Z Z
1 1 cos(3x)
u1 = − dx u2 = dx
12 12 sin(3x)
Z
1 1 1 cos(3x)(3dx)
u1 = − x u2 = ·
12 12 3 sin(3x)
1
u2 = ln | sin(3x)|
36

Hence,
yp = u1 y1 + u2 y2
1 1
yp = − x cos(3x) + ln | sin(3x)| sin(3x)

e
12 36

Us
Therefore, the general solution y = yc + yp is

1 1

l
y = c1 cos(3x) + c2 sin(3x) − x cos(3x) + ln | sin(3x)| sin(3x)

cia
er 12 36
m
C om
or

3. IVP: y 00 − y 0 − 2y = 4x2 , y(0) = 0 and y 0 (0) = 1


tF

Solution:
Step 1. First, we solve the associated homogeneous equation y 00 − y 0 − 2y = 0. We
No

give the auxiliary equations, the roots, and the complementary function yc .

m2 − m − 2 = 0
(m − 2)(m + 1) = 0
m1 = 2 and m2 = −1
yc = c1 e2x + c2 e−x

Step 2. We now determine yp by method of variation of parameters. Note that the


given DE is already in the standard form. We identify f (x) = 4x2 , y1 = e2x and y2 =
e−x .
Next, compute the Wronskian:
81

0 e−x
W = W (y1 , y2 ) = W (e2x , e−x ) W1 =
4x2 −e−x
e2x e−x
W = W1 = −4x2 e−x
2e2x −e−x
W = e2x (−e−x ) − 2e2x (e−x )
e2x 0
W = −3ex W2 =
2e2x 4x2
W2 = 4x2 e2x
We now obtain u01 and u02 as follows,

e
Us
W1 W2
u01 = u02 =
W W
2 −x

l
2 2x
−4x e 4x e

cia
u01 = u02 =
−3ex −3ex
4 4
u01 = x2 e−2x u02 = − x2 ex
er
3 3
m
We may use integration by parts to determine u1 and u2 as follows:
om

Z Z
4 2 −2x 4
u1 = xe dx u2 = − x2 ex dx
3 3
C

2 2 1 4 8 8
u1 = − x2 e−2x − xe−2x − e−2x u2 = − x2 ex + xex − ex
3 3 3 3 3 3
or

Hence,
tF

yp = u1 y1 + u2 y2
   
No

2 2 −2x 2 −2x 1 −2x 2x 4 2 x 8 x 8 x −x


yp = − x e − xe − e e + − x e + xe − e e
3 3 3 3 3 3
   
2 2 1 4 8 8
y p = − x2 − x − + − x2 + x −
3 3 3 3 3 3
yp = −2x2 + 2x − 3

The general solution y = yc + yp is

y = c1 e2x + c2 e−x − 2x2 + 2x − 3.


82

Lastly, we seek the solution for the IVP. If y(0) = 0, then, x = 0 and y = 0, and thus

0 = c1 e0 + c2 e0 − 2(0)2 + 2(0) − 3
3 = c1 + c2

The derivative of the general solution is

y 0 = 2c1 e2x − c2 e−x − 4x + 2

If y 0 (0) = 1, then, x = 0 and y 0 = 1, and thus

1 = 2c1 e0 − c2 e0 − 4(0) + 2

e
Us
−1 = 2c1 − c2

Solving the system

l
cia
3 = c1 + c2
−1 = 2c1 − c2
er
2 7
we obtain c1 = and c2 = . Therefore, the solution of the IVP is
m
3 3
om

2 7
y = e2x + e−x − 2x2 + 2x − 3.
3 3
C

Activity
or
tF

A. Find the general solution of the following differential equation.


No

1. y 000 + 3y 00 + 3y 0 + y = 0
2. y 000 + 3y 00 − 4y 0 − 12y = 0
3. y (4) − 2y 00 + y = 0
4. y (4) + y 000 + y 00 = 0
5. y 000 − 6y 00 = 3 − cos x
6. y 00 − 2y 0 + 5y = ex sin x
7. y 00 + y = sec x
8. 3y 00 − 6y 0 + 6y = ex sec x
83

Chapter 4: The Laplace Transforms


Chapter Overview
In this chapter, we discuss definition of Laplace transform, transforms and inverse
transforms of some basic functions, translation theorems, and solving initial-value
problems using Laplace transform and its inverse.

Learning Objectives
At the end of this chapter, the student is expected to

1. evaluate the Laplace transform or inverse Laplace transform of a function

e
2. use Laplace or inverse Laplace transforms and some translation theorems to solve

Us
initial-value problems involving linear ordinary differential equations

l
cia
4.1 Definition of Laplace Transforms er
In mathematical analysis, you have learned that differentiation and integration are trans-
forms, which means that these operations transform a function into another function. More-
m
over, these transforms possess the linearity property, that is, the transform of a linear
combination of functions is a linear combination of the transforms.
om

For α and β constants, provided that each derivative and integral exists,
C

d
[αf (x) + βg(x)] = αf 0 (x) + βg 0 (x)
dx
or

and Z Z Z
tF

[αf (x) + βg(x)]dx = α f (x)dx + β g(x)dx


In this section we will examine a special type of integral transform called the Laplace trans-
No

form and in addition to its linearity property, it has other properties that makes it useful in
solving linear IVP.
Before we proceed with the definition and examples, take note of the following: If f (x, y)
is a function of two variables, then a definite integral of f with respect to one of the vari-
ables
Z leads to a function of the other variable. That means, a definite integral such as

K(s, t)f (t)dt transforms a function f of the variable t into a function F of the variable
0
s. We are particularly interested in an integral transform, where the interval of integration
is the unbounded interval [0, ∞). If f (t) is defined for t ≥ 0, then we have
Z ∞ Z b
K(s, t)f (t)dt = lim K(s, t)f (t)dt
0 b→∞ 0

If the limit exists, the integral is said to be convergent, otherwise, it is divergent.


84

Definition 1: Laplace Transform


Let f be a function defined for t ≥ 0. Then the integral
Z ∞
L{f (t)} = e−st f (t)dt (1)
0

is said to be the Laplace transform of f , provided that the integral converges.

The integral in the definition of the transform is called an improper integral and it would
probably be best to recall how these kinds of integrals work. When the defining integral (1)
converges, the result is a function of s. In the following discussion we will use a lowercase
letter to denote the function being transformed and the corresponding capital letter to denote
its Laplace transform. For example,

e
Us
L{f (t)} = F (s), L{g(t)} = G(s), L{y(t)} = Y (s),
Example 1.

l
1. Evaluate L{1}
Solution: From the definition, we have
Z ∞ Z b cia   
er
b
−st e−st −e−sb 1 1
L{1} = e (1)dt = lim e−st dt = lim = lim − − =
b→∞ 0 b→∞ −s b→∞ s s s
m
0 0

provided s > 0. The integral diverges for s < 0.


om

b ∞
For the next examples, instead of writing lim ( ) 0 , we will use the notation ( ) 0
.
C

b→∞

2. Evaluate L{t}
or

Solution: From the definition, we have


Z ∞
tF

L{t} = e−st (t)dt ←− integrate by parts


0
∞ Z
te−st 1 ∞ −st
No

=− + e dt ←− note that e−st → 0 as t → ∞, s > 0


s 0 s 0
1
= 2
s
3. Evaluate L{e5t }
Solution: From the definition, we have
Z ∞ Z ∞ ∞
−st 5t
 e−(s−5)t 1
5t
L{e } = e e dt = e−(s−5)t dt = − = , s>5
0 0 (s − 5) 0 s−5

4. Evaluate L{sin 2t}


Solution: From the definition, we have
Z ∞ 
L{sin 2t} = e−st sin 2t dt
0
85

Use integration by parts and take note that e−st → 0 as t → ∞, s > 0


Z Z
 e−st sin 2t 2
e−st sin 2t dt = − + e−st cos 2tdt
s s
Z  Z 
 −st
e sin 2t 2 e−st cos 2t 2 
e−st sin 2t dt = − + − − e−st
sin 2t dt
s s s s
Z Z
 e−st sin 2t 2e−st cos 2t 4 
e−st sin 2t dt = − − − e −st
sin 2t dt
s s2 s2
Z
 s  1 
e−st sin 2t dt = − 2 e−st sin 2t − 2 2e−st cos 2t
s +4 s +4
Z ∞  
 s  1  ∞
e−st sin 2t dt = − 2 −st
e sin 2t − 2 −st
2e cos 2t
s +4 s +4
Z0 ∞ 0

e
 2
e−st sin 2t dt = 2 , s>0

Us
0 s +4

L is a Linear Transform

l
For a linear combination of functions, we can write
Z ∞ Z ∞ cia Z ∞
er
−st −st
e [αf (t) + βg(t)]dt = α e f (t)dt + β e−st g(t)dt
m
0 0 0

whenever both integrals converge for s > c. Hence it follows that


om

L{αf (t) + βg(t)} = αL{f (t)} + βL{g(t)} (2)


C

Because of this property in (2), L is said to be a linear transform.


or
tF

Example 2. Using the results from Example 1 and the fact that L is a linear transform,
we have
1 5
No

1. L{1 + 5t} = L{1} + 5L{t} = + 2


s s
4 20
2. L{4e5t − 10 sin 2t} = 4L{e5t } − 10L{sin 2t} = − 2
s−5 s +4
We state the generalization of some of the preceding examples by means of the next theorem.
From this point on we shall also refrain from stating any restrictions on s; it is understood
that s is sufficiently restricted to guarantee the convergence of the appropriate Laplace
transform.
86

Theorem 1: Transforms of Some Basic Functions


1 s
(a) L{1} = (e) L{cos kt} =
s s2
+ k2
n! k
(b) L{tn } = , n = 1, 2, 3, . . . (f) L{sinh kt} = 2
sn+1 s − k2
1 s
(c) L{eat } = (g) L{cosh kt} = 2
s−a s − k2
k
(d) L{sin kt} = 2
s + k2

Example 3. Using Theorem 1, find the Laplace transforms of the given functions.

e
1. f (t) = 6e−5t + e3t + 5t3 − 9

Us
Solution:

L{6e−5t + e3t + 5t3 − 9} = F (s) = 6L{e−5t } + L{e3t } + 5L{t3 } − 9L{1}

l
=6·
cia
1
+
1
s − −(5) s − 3
3!
+5· 4 −9·
s
1
s
er
6 1 30 9
= + + −
s + 5 s − 3 s4 s
m
om

2. g(t) = 4 cos 4t − 9 sin 4t + 2 cos 10t


Solution:
C

L{4 cos 4t − 9 sin 4t + 2 cos 10t} = G(s) = 4L{cos 4t} − 9L{sin 4t} + 2L{cos 10t)}
s 4 s
or

=4· 2 2
−9· 2 2
+2· 2
s + (4) s + (4) s + (10)2
tF

4s 36 2s
= 2 − 2 + 2
s + 16 s + 16 s + 100
No

Sufficient conditions guaranteeing the existence of L{f (t)} are that f be piecewise continu-
ous on [0, ∞) and that f be of exponential order for t > T .

Definition 2: Exponential Order


A function f is said to be of exponential order c if there exist constants c, M > 0,
and T > 0 such that f (t) ≤ M ect for all t > T .

Example 4. The functions f (t) = t, f (t) = e−t and f (t) = 2 cos t are all exponential order
c = 1 for t > 0 since we have, respectively,

|t| ≤ et , e−t ≤ et , and |2 cos t| ≤ 2et .


87

Piecewise Continuous
A function f is piecewise continuous on [0, ∞) if, in any interval 0 ≤ a ≤ t ≤ b, there
are at most a finite number of points tk , k = 1, 2, 3, · · · , n(tk−1 < tk ) at which f has
finite discontinuities and is continuous on each open interval tk−1 < t < tk .

In other words, a function is called piecewise continuous on an interval if the interval can
be broken into a finite number of subintervals on which the function is continuous on each
open subinterval (i.e. the subinterval without its endpoints) and has a finite limit at the
endpoints of each subinterval.

e
l Us
cia
er
Figure 1: a sketch of a piecewise continuous function
m
om

Theorem 2
If f is piecewise continuous on the interval [0, ∞) and of exponential order c for t > T ,
C

then L{f (t)} exists for s > c .


or


 −1 if 0 ≤ t < 1
tF

Example 5. Evaluate L{f (t)} if f (t) =


 1 if t ≥ 1
Solution: Since f is defined in two pieces, L{f (t)} is expressed as the sum of two integrals:
No

Z ∞ Z 1 Z ∞
−st −st
L{f (t)} = e f (t)dt = e (−1)dt + e−st (1)dt
0 0 1
−st 1 ∞
e e−st
= −
s s
 −s 0  1−s
e 1 e
= − +
s s s
−s
2e 1
= − , s>0
s s
Remarks 1. Throughout this chapter we shall be concerned primarily with functions that
are both piecewise continuous and of exponential order. We note, however, that these two
conditions are sufficient but not necessary for the existence of a Laplace transform.
88

4.2 Inverse Transforms


Finding the Laplace transform of a function is not that difficult if we have with us at least the
Laplace transforms of some basic functions from the previous section. In this section, we will
illustrate the use of Laplace transform to solve some ordinary differential equations and to
do this, we need some preliminary background on the Laplace transforms of derivatives and
the concept of inverse Laplace transform. The idea for the inverse is that, given a function
F (s), we are to determine a function f (t) such that L{f (t)} = F (s).

Inverse Transform
If F (s) represents the Laplace transform of a function f (t), that is, L{f (t)} = F (s),
we can say that f (t) is the inverse Laplace transform of F (s) and we write f (t) =

e
L−1 {F (s)}.

Us
Theorem 3: Some Inverse Transforms
   

l
−1 1 s

cia
−1
(a) 1 = L (e) cos kt = L 2 2
s  s + k 
−1 n! k
n
(f) sinh kt = L−1 2
er
(b) t = L n+1
, n = 1, 2, 3, . . . 2
s  s − k 
1
m
s
(c) eat = L−1 (g) cosh kt = L−1 2
s− a  s − k2
om

−1 k
(d) sin kt = L
s2 + k 2
C

A possible key to doing inverse transforms is to look at the denominator and try to identify
or

which among from the list in Theorem 3 may satisfy that denominator. If there is only
one entry in the list that has that particular denominator, the next step is to make sure
tF

the numerator is correctly set up for the inverse transform process. If there is more than
one entry in the list that has a particular denominator, then the numerators of each will be
No

different. We may fix up the numerator or the function of s by multiplying and dividing the
original given by an appropriate constant.

Example 6. Use Theorem 3 to evaluate the following.


 
1
1. L−1 5
s

Solution: Since the given has a denominator match that of part (b) of Theorem 3,
we identify n + 1 = 5 which implies that n = 4. Then the numerator must be n! = 4!.
Multiply and divide the given by 4!. We have now,
   
−1 1 1 −1 4! 1
L 5
= L 5
= t4
s 4! s 24
89
 
−1 5
2. L 2
s + 49

Solution: The given function match that of part (d) of Theorem 3 where we identify
k 2 = 49 or k = 7. Fix up the expression by factoring out 5 and multiplying and
1
dividing the expression by .
7
   
−1 5 5 −1 7 5
L 2
= L 2
= sin 7t
s + 49 7 s + 49 7

L−1 is a Linear Transform


The inverse Laplace transform is also a linear transform. That is, given two

e
Us
Laplace transforms F (s) and G(s),

L−1 {αF (s) + βG(s)} = αL−1 {F (s)} + βL−1 {G(s)} (3)

l
for any constants α and β.
cia
er
 
−1 2s − 6
3. L
m
s2 + 9
om

Solution: Let us first write the given function as a sum of two expressions and use the
fact that an inverse Laplace transform is also a linear transform. We identify k 2 = 9or
k=3
C

   
−1 2s − 6 −1 2s 6
or

L =L −
s2 + 9 s2 + 9 s2 + 9
   
tF

−1 s −1 3
= 2L − 2L ←− fix up the constants
s2 + 9 s2 + 9
 
= 2 cos 3t − 2 sin 3t ←− parts (d) and (e) of Theorem 3
No

Partial fractions play an important role in finding inverse Laplace transforms. The decom-
position of a rational expression into component fractions can be done quickly by means of a
single command on most computer algebra systems. Indeed, some CASs have packages that
implement Laplace transform and inverse Laplace transform commands. But if we do not
have access to such software, we have to deal with it using algebra in the important cases in
which the denominator of a Laplace transform F (s) contains distinct linear factors, repeated
linear factors, and quadratic polynomials with no real factors. At this point, we will use our
knowledge of partial fractions decomposition we have used in integral calculus.
90

Example 7. Evaluate the following inverse Laplace transforms.


 
−1 s
1. L
(s − 2)(s − 3)(s − 6)

Solution: The denominator of the given expression seems to suggest that we have got
a couple of exponentials, part (c) of Theorem 3. However in order to be exponentials
there can only be a single term in the denominator and no s0 s in the numerator. To
fix this, we will use partial fraction decomposition.
We want to determine constants A, B, C such that
s A B C
= + +
(s − 2)(s − 3)(s − 6) s−2 s−3 s−6
s = A(s − 3)(s − 6) + B(s − 2)(s − 6) + C(s − 2)(s − 3)

e
s = A(s2 − 9s + 18) + B(s2 − 8s + 12) + C(s2 − 5s + 6)

Us
By comparing coefficients of powers of s from both sides of the last equation, we will
have a system involving three equations in three unknowns A, B and C.

l
cia
0 = A + B + C, 1 = −9A − 8B − 5C, and 0 = 18A + 12B + 6C
However, recall that we can solve for the constants A, B and C from
er
s = A(s − 3)(s − 6) + B(s − 2)(s − 6) + C(s − 2)(s − 3)
m
if we set s = 2, s = 3, and s = 6. Hence we will have,
om

1
2 = A(−1)(−4) =⇒ A = ,
2
3 = B(1)(−3) =⇒ B = −1, and
C

1
6 = C(4)(3) =⇒ C =
2
or

1 1
tF

s 1
Thus = 2 − + 2 . From the linearity of L−1 and
(s − 2)(s − 3)(s − 6) s−2 s−3 s−6
using part (c) of Theorem 3,
No

       
−1 s 1 −1 1 1 1 1 −1 1
L = L −L + L
(s − 2)(s − 3)(s − 6) 2 s−2 s−3 2 s−6
1 1
= e2t − e3t + e6t
2 2
4.3 Solving Linear ODE Using Transforms
As what we have mentioned in the introduction of this chapter, our immediate goal is to use
the Laplace transform
  to solve
 2differential
 equations. With that, we will need to evaluate
dy dy
expressions like L ,L , and so on.
dt dt2
The Laplace transform of the nth derivative of f is given in the following theorem.
91

Theorem 4: Transform of a Derivative


If f, f 0 , · · · , f n−1 are continuous on [0, ∞) and are of exponential order and if f n (t) is
piecewise continuous on [0, ∞), then

L{f n (t)} = sn F (s) − s(n−1) f (0) − s(n−2) f 0 (0) − . . . − f (n−1) (0), (4)

where F (s) = L{f (t)}.




dn y
It is apparent from the general result given in Theorem 4 that L depends on Y (s) =
dtn
L{y(t)} and the n−1 derivatives of y(t) evaluated at t = 0. This property makes the Laplace
transform ideally suited for solving linear initial-value problems in which the differential
equation has constant coefficients. Such a differential equation is simply a linear combination

e
of terms y, y 0 , y 00 , . . . , y n :

Us
dn y dn−1 y
an + a n−1 + · · · + a0 y = g(t)
dtn dtn−1

l
cia
y(0) = y0 , y 0 (0) = y1 , . . . , y n−1 (0) = yn−1
where the ai , i = 1, 2, 3, . . . , n and y0 , y1 , . . . , yn−1 are constants. By the linearity prop-
er
erty of the Laplace transform, the Laplace transform of this linear combination is a linear
combination of Laplace transforms:
m
 n   n−1 
d y d y
om

an L n
+ an−1 L + · · · + a0 L{y} = L{g(t)}
dt dtn−1
C

We then apply Theorem 4 to solve for this last equation. The Laplace transform of a linear
differential equation with constant coefficients becomes an algebraic equation in Y (s).
or

The procedure can be summarised as follows


tF
No

Figure 2:
92

In particular, (from Theorem 4) if f 0 is continuous for t ≥ 0, we have the following


L{f 0 (t)} = sF (s) − f (0)
L{f 00 (t)} = s2 F (s) − sf (0) − f 0 (0) (5)
L{f 000 (t)} = s3 F (s) − s2 f (0) − sf 0 (0) − f 00 (0)
Example 8. Use the Laplace transform to solve the given initial-value problem.
dy
1. + 3y = 13 sin 2t, y(0) = 6
dt
Solution:
dy
+ 3y = 13 sin 2t ←− apply the Laplace transform
  dt
dy
L + 3L{y} = 13L{sin 2t}

e
dt

Us
26
sY (s) − y(0) + 3Y (s) = 2 ←− from (5) and part (d) of Theorem 1
s +4
26

l
(s + 3)Y (s) = 6 + 2 ←− use initial condition: y(0) = 6

cia
s +4
6 26
Y (s) = + ←− solve the last equation for Y (s)
s + 3 (s + 3)(s2 + 4)
er
6s2 + 50
Y (s) = ←− apply inverse Laplace transform
m
(s + 3)(s2 + 4)
 
om

−1 −1 6s2 + 50
L {Y (s)} = y(t) = L
(s + 3)(s2 + 4)
C

By partial fraction decomposition,


6s2 + 50 A B(2s) + C
or

2
= +
(s + 3)(s + 4) s+3 s2 + 4
tF

6s2 + 50 = A(s2 + 4) + [B(2s) + C](s + 3)


6s2 + 50 = A(s2 + 4) + B(2s2 + 6s) + C(s + 3)
No

By comparing coefficients of powers of s from both sides of the last equation, we will
have a system involving three equations in three unknowns A, B and C.
6 = A + 2B, 0 = 6B + C, and 50 = 4A + 3C

Solving for A, B, and C, we will have, A = 8, B = −1 and C = 6.


Hence, by linearity of transform and fixing up the constants,
   
−1 −1 6s2 + 50 −1 8 −(2s) + 6
L {Y (s)} = y(t) = L =L +
(s + 3)(s2 + 4) s+3 s2 + 4
     
−1 1 −1 s −1 2
y(t) = 8L − 2L + 3L
s+3 s2 + 4 s2 + 4
Therefore, from parts (c), (d), and (e) of Theorem 3,
y(t) = 8e−3t − 2 cos 2t + 3 sin 2t ←− solution of IVP
93

2. y” + 5y 0 + 4y = 0, y(0) = 1, y 0 (0) = 0
dy
Solution: Let y 0 =
dt

y” + 5y 0 + 4y = 0 ←− apply the Laplace transform


   
d2 y dy
L 2
+ 5L + 4L{y} = L{0}
dt dt
s2 Y (s) − sy(0) − y 0 (0) + 5[sY (s) − y(0)] + 4Y (s) = 0 ←− use initial conditions
s2 Y (s) − s(1) − 0 + 5[sY (s) − 1] + 4Y (s) = 0 ←− solve for Y (s)
 
Y (s) s2 + 5s + 4 = s + 5
s+5
Y (s) = ←− apply the inverse Laplace transform
(s + 4)(s + 1)

e
 

Us
−1 −1 s+5
L {Y (s)} = y(t) = L
(s + 4)(s + 1)

l
cia
1 4
s+5 −
By partial fraction decomposition, = 3 + 3 . (Verify!)
(s + 4)(s + 1) s+4 s+1
er
Hence, by linearity of transform and fixing up the constants,
m
 
s+5
om

−1 −1
L {Y (s)} = y(t) = L
(s + 4)(s + 1)
   
1 −1 1 4 −1 1
y(t) = − L + L
C

3 s+4 3 s+1
1 4
y(t) = − e−4t + e−t
or

3 3
tF

Remarks 2. The inverse Laplace transform of a function F (s) may not be unique.
No

4.4 Translation Theorems and Additional Operational


Properties
The last two examples from the previous lesson illustrated the basic procedure for using the
Laplace transform to solve a linear initial-value problem. There maybe a lot of algebra inher-
ent in the use of the Laplace transform, but observe that we do not have to use variation of
parameters or worry about the cases and algebra in the method of undetermined coefficients.
Moreover, we did not need to seek first for the general solution and then substitute the initial
conditions to determine the solution of the IVP. For this lesson, we will examine some other
operational properties of the Laplace transform and we will see how these can enable us to
solve more complex problems that can’t be easily done by the techniques we have learn in
the last two chapters.
94

Theorem 5: First Translation Theorem


If L{f (t)} = F (s) and a is any real number, then

L{eat f (t)} = F (s − a)

For emphasis, it is sometimes useful to use the symbolism

L{eat f (t)} = L{f (t)}|s→s−a

where s → s − a implies that in the Laplace transform F (s) of f (t), we replace the symbol
s by s − a, wherever it appears.

e
Example 9. Using Theorem 5, evaluate the following.

Us
1. L{e−2t t3 }
Solution: By part (b) of Theorem 1 and by Theorem 5,

l
L{e−2t t3 } = L{t3 }|s→s+2 =cia 3!
s4
=
6
(s + 2)4
er
s→s+2
m
2. L{et sin 3t}
om

Solution: By part (d) of Theorem 1 and by Theorem 5,

3 3
C

L{et sin 3t} = L{sin 3t}|s→s−1 = =


s2 +9 s→s−1 (s − 1)2 + 9
or
tF

Inverse Form of First Translation Theorem


To compute the inverse of F (s − a), we must recognize F (s), find f (t) by taking the
No

inverse Laplace transform of F (s), and then multiply f (t) by the exponential function
eat . This procedure can be summarized symbolically in the following manner:

L−1 {F (s − a)} = L−1 {F (s)|s→s−a } = eatf (t) (6)

where f (t) = L−1 {F (s)}

 
−1 s
Example 10. Evaluate: L
s2 + 4s + 5
Solution: Observe that the denominator s2 +4s+5 has no real zeros and so has no real linear
s s
factors. We apply completing the square and write the given as 2 = .
s + 4s + 5 (s + 2)2 + 1
Our goal is to recognize the expression on the right-hand side as some Laplace transform
F (s) in which s has been replaced by s + 2. However, we must fix up the numerator s so
95

that it will also become s + 2. That is,


   
−1 s −1 s
L =L
s2 + 4s + 5 (s + 2)2 + 1
 
−1 s+2 2
=L −
(s + 2)2 + 1 (s + 2)2 + 1
   
−1 s+2 −1 1
=L − 2L
(s + 2)2 + 1 (s + 2)2 + 1
   
−1 s −1 1
=L − 2L
s2 + 1 s→s+2 s2 + 1 s→s+2

By parts (d) and (e) of Theorem 3 and (6), we have

e
 

Us
−1 s
L = e−2t cos t − 2e−2t sin t
s2 + 4s + 5

l
Example 11. Solve: y 00 − y 0 = et cos t, y(0) = 0, y 0 (0) = 0

Solution: cia
er
y 00 − y 0 = et cos t ←− apply the Laplace transform
m

L{y 00 } − L{y 0 } = L{et cos t} ←− apply Theorem 4 and Theorem 5


om

s−1
s2 Y (s) − sy(0) − y 0 (0) − [sY (s) − y(0)] = ←− use initial conditions:
(s − 1)2 + 1
C

s−1
s2 Y (s) − sY (s) = ←− solve for Y (s)
(s − 1)2 + 1
or

s−1
Y (s)[s2 − s] =
(s − 1)2 + 1
tF

s−1
Y (s) = ←− apply inverse Laplace transform
s(s − 1)[(s − 1)2 + 1]
 
No

−1 −1 s−1
L {Y (s)} = y(t) = L
s(s − 1)[(s − 1)2 + 1]

By partial fraction decomposition,

s−1 A B C(s) + E
2
= + +
s(s − 1)[(s − 1) + 1] s s − 1 (s − 1)2 + 1
s − 1 = A(s − 1)[(s − 1)2 + 1] + B(s)[(s − 1)2 + 1] + [C(s) + E](s)(s − 1)

1
Let s = 0 : −1 = A(−1)(2) =⇒ A =
2
Let s = 1 : 0 = B(1)(1) =⇒ B = 0
Let s = 2 : 1 = A(1)(2) + B(2)(2) + [C(2) + E](2)(1) =⇒ 2C + E = 0
Let s = −1 : −2 = A(−2)(5) + [C(−1) + E](−1)(−2) =⇒ −3 = −2C + 2E
96

1
Solving the last two equations, we will have, C = − and E = 1.
2
Hence,
 
−1 −1 s−1
L {Y (s)} = y(t) = L
s(s − 1)[(s − 1)2 + 1]
1 1
− (s) + 1 
= L−1 2 + 2
s (s − 1)2 + 1
1 1
− (s − 1) + 
1
= L−1 2 + 2 2
2
s (s − 1) + 1
     
1 −1 1 1 −1 s−1 1 −1 1
= L − L + L

e
2 s 2 (s − 1)2 + 1 2 (s − 1)2 + 1

Us
1 1 1
y(t) = − et cos t + et sin t
2 2 2

l
Definition 3: Unit Step Function

The unit step function U(t − a) is defined to becia


er

 0 if 0 ≤ t < a
m
U(t − a) =
 1 if t ≥ a
om

A general piecewise-defined function of the type


C


 g(t) if 0 ≤ t < a
or

f (t) = (7)
 h(t) if t ≥ a
tF

is the same as
No

f (t) = g(t) − g(t) U(t − a) + h(t) U(t − a). (8)

Similarly, a function of the type




 0 if 0 ≤ t < a


f (t) = g(t) if a ≥ t < b (9)




0 if t ≥ b
can be written as

f (t) = g(t) [U(t − a) − U(t − b). (10)


97

Theorem 6: Second Translation Theorem


If F (s) = L{f (t)} and a > 0, then

L{f (t − a)U(t − a)} = e−as F (s).

Alternatively, a more simple version of Theorem 6 can be obtained by using the definition
of Laplace transform, the unit step function and the substitution u = t − a. That is,
Z ∞ Z ∞
−st
L{g(t)U(t − a)} = e g(t) dt = e−s(u+a) g(u + a) du
a 0

L{g(t)U(t − a)} = e−as L{g(t + a)} (11)

e
Us

 0 if 0 ≤ t < 1
Example 12. Let f (t) = . Express f (t) in terms of a unit step
 t2 if t ≥ 1

l
function and evaluate L{f (t)}.
cia
er
Solution: From (7) and (8), we identify a = 1, g(t) = 0 and h(t) = t2 . Hence, we have
m
f (t) = 0 − 0 U(t − 1) + t2 U(t − 1)
f (t) = t2 U(t − 1)
om

Now, for L{f (t)} = L{t2 U(t − 1)} , from (11), we identify g(t) = t2 and a = 1. Hence,
C

g(t + 1) = (t + 1)2 = t2 + 2t + 1. By the alternative form of the second translation theorem


and by using linearity of the transform and some basic Laplace transforms,
or

L{t2 U(t − 1)} = e−s L{t2 + 2t + 1}


tF

 
= e−s L{t2 } + 2L{t} + L{1}
 
−s 2! 1 1
No

=e +2 2 +
s3 s s
−s −s
2e 2e e−s
= 3 + 2 +
s s s

Inverse Form of Second Translation Theorem


If f (t) = L−1 {F (s)}, the inverse form of Theorem 6, a > 0 s

L−1 {e−as F (s)} = f (t − a) U(t − a) (12)


98

Example 13. Evaluate the following.


 −2s 
−1 e
1. L
s3
1
Solution: From (12) we identify a = 2, F (s) = and
s3
 
−1 −1 1 2 1
L {F (s)} = L · 3 = t2
2 s 2
. Hence we have  
−1 e−2s 1
L = (t − 2)2 U(t − 2)
s3 2
 
e−s

e
−1
2. L
s(s + 1)

Us
1
Solution: From (12) we identify a = 1, F (s) = and
s(s + 1)

l
 

cia
−1 −1 1
L {F (s)} = L ←− use partial fraction decomposition
s(s + 1)
   
er
−1 1 −1 1
=L −L ←− use Theorem 3
s s+1
m
= 1 − e−t .
om

Hence we have  
−1 e−s  
L = 1 − e−(t−1) U(t − 1)
C

s(s + 1)

or

 0 if 0 ≤ t < π
Example 14. Solve y 0 + y = f (t), y(0) = 5, where f (t) = .
 3 sin t if t ≥ π
tF

Solution: Let us first write f (t) in terms of unit step functions. From (7) and (8), we
identify a = π, g(t) = 0 and h(t) = 3 sin t. Hence, we have
No

f (t) = 0 − 0 U(t − π) + 3 sin t U(t − π)


f (t) = 3 sin t U(t − π).
We now have,
y 0 + y = 3 sin t U(t − π) ←− apply Laplace transform
L{y 0 } + L{y} = L{3 sin t U(t − π)} ←− apply Theorem 4 and Theorem 6
s
sY (s) − y(0) + Y (s) = −3 2 e−πs ←− solve for Y (s) and substitute initial condition
s +1
5 3s
Y (s) = − e−πs ←− apply partial fraction decomposition
s + 1 (s + 1)(s2 + 1)
 
5 3 1 −πs 1 −πs s −πs
Y (s) = − − e + 2 e + 2 e
s+1 2 s+1 s +1 s +1
99

From the last equation, apply the inverse Laplace transform


   
−1 −1 5 3 −1 1 −πs 1 −πs s −πs
L {Y (s)} = L − L − e + 2 e + 2 e
s+1 2 s+1 s +1 s +1
Using the inverse form of the second translation theorem on the terms inside the bracket,
we identify a = π and the inverses are
 
−1 1 −πs
L − e = e−(t−π) U(t − π)
s+1
 
−1 1 −πs
L e = sin(t − π) U(t − π) = − sin t U(t − π)
s2 + 1
 
−1 s −πs
L e = cos(t − π) U(t − π) = − cos t U(t − π)
s2 + 1

e
Us
Therefore,
3 3 3
L−1 {Y (s)} = y(t) = 5e−t + e−(t−π) U(t − π) + sin t U(t − π) + cos t U(t − π)

l
2 2 2

cia

−t

 5e if 0 ≤ t < π
= y(t) =
er
 5e−t + 3 e−(t−π) + 3 sin t + 3 cos t) if t ≥ π

2 2 2
m

Theorem 7: Derivatives of Transforms


om

If F (s) = L{f (t)} and n = 1, 2, 3, . . . , then


C

dn
L{tn f (t)} = (−1)n F (s)
dsn
or
tF

Example 15. Evaluate the following.


1. L{t sin kt}
No

Solution: From Theorem 7, we identify f (t) = sin kt and n = 1. We also use transform
of some basic functions from Theorem 1.
 
d d k 2ks
L{t sin kt} = − [L{sin kt}] = − 2 2
= 2 .
ds ds s + k s + k2
2

2. L{e−2t t3 }
Solution: From Theorem 7, we identify f (t) = e−2t and n = 3. We also use transform
of some basic functions from Theorem 1.
3
 
−2t 3 3 d −2t d3 1 6
L{e t } = (−1) 3 [L{e }] = − 3 =
ds ds s + 2 (s + 2)4
Note that L{e−2t t3 } has been evaluated also by the first translation theorem.
100

Activity

A. Use Theorem 1 to evaluate L{f (t)} of the following functions.

1. f (t) = 2t4

2. f (t) = t2 + 6t − 3

3. f (t) = 4t2 − 5 sin 3t


2
4. f (t) = 1 + e2t

5. f (t) = 1 − et + 3e−4t cos 5t

 sin t if 0 ≤ t < π

e
B. By definition, evaluate L{f (t)} if f (t) =

Us
 0 if t ≥ π
C. Use Theorem 3 to evaluate the following inverse transform

l
 
1. L−1 4
1
s cia
er
 
1 1 1
2. L−1 2 − +
m
s s s−2
 
om

−1 1
3. L
s3 + 5s
C

 
−1 1
4. L
(s2 + 1)(s2 + 4)
or

 
2s − 1
tF

−1
5. L
s2 (s + 1)3
No

D. Solve the following initial-value problems.

1. y 00 − 6y 0 + 13y = 0, y(0) = 0, y 0 (0) = −3


√ √
2. y 00 + y = 2 sin 2t, y(0) = 10, y 0 (0) = 0
101

Chapter 5: Applications of First Order Differential Equations

Chapter Overview
This chapter applies problems that give rise to some of the types of first order
ordinary differential equations studied in the previous chapters. Applications of the
first order differential equations presented through examples are widely applied to
model natural phenomena in engineering systems and many other situations.

Learning Outcomes
At the end of this chapter, the student is expected to:
1. formulate the problem mathematically, thereby obtaining a differential
equation;

e
2. solve the equation and attempt to interpret the solution in terms of the

Us
quantities involved in the original problem.

l
5.1 Differential Equations of Plane Curves

cia
The solution of differential equation can be displayed graphically as a family
er
of integral curves. It turns out that one can also solve the inverse problem: construct
a differential equation of the family of plane curves defined by an algebraic equation,
m

then use the step in the elimination of arbitrary constant.


om

Example 1.
C

Determine the differential equation for the family of curves defined by the equation
y = 𝑒 "#$ .
or

Solution.
Differentiating the given equation with respect to x gives:
tF

y’ = 𝑒 "#$
Eliminate the parameter C from the system of equations:
No

y = 𝑒 "#$ Equation 1
"#$
− ( y’ = 𝑒 ) Equation 2
"#$
Hence, y=𝑒
−y’ = −𝑒 "#$

y – y’ = 0

As a result, we obtain the following simplest homogeneous equation:


y’ = y or y – y’ = 0
102

Example 2.
Derive the differential equation for the family of plane curves defined by the equation
y = 𝑥 ' − 𝐶𝑥
Solution.
Differentiate the implicit equation with respect to x:
y’ = 2x − C
Write this equation jointly with the original algebraic equation and eliminate the
parameter C:
y = 𝑥 ' − 𝐶𝑥 Equation 1
y’ = 2𝑥 − C Equation 2
Solve C from equation 2, we have

e
C = 2 𝑥 − y’ Equation 3

Us
Substitute equation 3 from equation 1, we have
y = 𝑥 ' − 2x − 𝑦’ 𝑥 , simplifying
y = 𝑥 ' − 2𝑥 ' + 𝑦’𝑥 , thus

l
𝒚’𝒙 − 𝒚 − 𝒙𝟐 = 𝟎
cia
er
Example 3.
m
Derive the differential equation for the family of two-parameter plane curves
𝑦 = 𝐶4 + 𝐶' 𝑒 56" .
om

Solution.
Differentiating the given equation twice with respect to x and write the following
C

systems of three equations.


𝑦 = 𝐶4 + 𝐶' 𝑒 56" Equation 1
or

56"
𝑦’ = −4𝐶' 𝑒 Equation 2
56"
𝑦" = 16𝐶' 𝑒 Equation 3
tF

5;’
From equation 2, since 𝐶' 𝑒 56" = , substitute to equation 3, thus
6
No

5;’
𝑦" = 16 ( )
6
𝑦" = −4𝑦’
Thus, 𝒚" + 𝟒𝒚’ = 𝟎
103

Activity 1
Directions: Solve the following problems completely and give two (2) points for every
item. Your total score is 10 if you will get a perfect score. Put one (1) point if you will
finish half of the items and zero (0) if no solution at all. Answers are written on the
left side corner of the questionnaire.
Determine the differential equation for the family of plane curves defined by the
following equations:

1. 𝑦 = 𝑎𝑥 ' + 𝑏𝑥 + 𝐶 Ans. 𝒚AAA = 𝟎


4
2. 𝑦 = cos (𝑐𝑥 + 𝛼), where C is a parameter, 𝛼 is an arbitrary angle.
B
𝒙 𝟏5(𝒚A)𝟐
Ans. 𝒚’ = −𝒔𝒊𝒏 ( + 𝜶)
𝒚

e
3. 𝑦 = 𝐶4 𝑒 " + 𝐶' 𝑒 5" Ans. 𝒚AA − 𝒚 = 𝟎

Us
4. 𝐶𝑥 ' + 𝑥 + 𝑦 ' = 0 Ans. 𝒙 + 𝟐𝒚𝟐 − 𝟐𝒙𝒚𝒚′ = 𝟎
𝒅𝟐 𝒙
5. 𝑥 = 𝐴𝑠𝑖𝑛(𝜔𝑡 + 𝛽) ; 𝜔 a parameter not to be eliminated. Ans. + 𝜔𝟐 𝒙 = 𝟎
W𝒕𝟐

l
Progress Indicators:
cia
The following indicators below reflect your performance in this Activity 1.
er
Score Rating Interpretation
m
10 points Excellent Enrichment
om

Congratulations! You can


now proceed to the next
lesson.
C

5-9 points Average Reinforcement


or

Good! Study the part/s


where you commit
tF

mistake/s then solve it


over again until you get
No

the exact answer/s.


0-4 points Below Average Remedial Instruction
Study again from the very
beginning of this chapter.
To erase your difficulties,
be able to use additional
readings, watch youtube,
and other informative
materials. Solve it over
again until you master the
lesson and exercises, then
proceed to the next lesson.
Good Luck!
104

5.2 Isogonal and Orthogonal Trajectories

A trajectory (path) of a moving particle is said to be isogonal if a given family


of curves in a plane G, intersects another curve at plane T at a constant angle, θ.
Similarly, a trajectory is said to be orthogonal to the trajectory of another moving
particle if their tangent lines at their point of intersection makes an angle of 90
degrees with respect to each other. (See Figure 1)
For isogonal families that intersect at (x,y) with slope 𝑚Z and 𝑚^ , they intersect
at the constant angle θ, where:
𝒎𝟐 5𝒎𝟏
tan θ =
𝟏# 𝒎𝟏 𝒎𝟐

T.L. to T (x, y)

e
Us
T.L. to G (x, y)

l
cia
er G (x, y)

T (x, y)
m
om

Figure 1
C

Example 4.
or

Find the isogonal trajectory of 𝑥 ' = 𝑐𝑦 ` at θ= 90b for each family of curves.
tF

Solution.
Solve for 𝑚^ by getting the first derivative of the curve.
No

𝑥 ' = 𝑐𝑦 ` Equation 1
𝑑𝑦
2𝑥 = 𝑚𝑐𝑦 `54
𝑑𝑥
W;
Since 𝑚^ = ,
W"
'"
𝑚^ = Equation 2
`$; def
But 𝑥 ' = 𝑐𝑦 ` , therefore
"g
c= , substitute to equation 2
;d
2𝑥
𝑚^ =
𝑥'
𝑚( )𝑦 `54
𝑦`
'; W; ';
𝑚^ = or =
`" W" ^ `"
105

Derive the differential equation by getting the angle between two curves.
𝑚' − 𝑚4
tan 𝜃 =
1 + 𝑚4 𝑚'
𝑚4 − 𝑚^
tan 𝜃 =
1 + 𝑚4 𝑚^
𝑑𝑦 2𝑦
( ) − ( 4)
tan 90b = 𝑑𝑥 𝑚𝑥
𝑑𝑦 2𝑦
1 + ( )( )
𝑑𝑥 𝑚𝑥

𝑑𝑦 2𝑦
1 ( ) − ( 4)
= 𝑑𝑥 𝑚𝑥
0 1 + (𝑑𝑦)( 2𝑦 )

e
𝑑𝑥 𝑚𝑥

Us
𝑑𝑦 2𝑦
1+ =0
𝑑𝑥 𝑚𝑥
2𝑦𝑑𝑦 + 𝑚𝑥𝑑𝑥 = 0

l
By separation of variables, cia
er
2𝑦𝑑𝑦 + 𝑚𝑥𝑑𝑥 = 0
m
2 𝑦𝑑𝑦 + 𝑚 𝑥𝑑𝑥 = 0
om

`" g
𝑦' + =𝐶
'
𝒎𝒙𝟐 + 𝟐𝒚 = 𝑪
𝟐
C

Example 5.
or

Find the orthogonal trajectories of 𝑦 ' = 𝑐𝑥 k .


tF

Solution.
Eliminate the constant c by getting the first derivative of the curve.
𝑦 ' = 𝑐𝑥 k Equation 1
No

A '
2𝑦𝑦 = 3𝑐𝑥 Equation 2
Simplify.
3𝑐𝑥 '
𝑦A =
2𝑦
;g
But 𝑐 = ,
"m
𝑑𝑦 3𝑥 ' 𝑦 '
=
𝑑𝑥 2𝑥 k 𝑦
𝑑𝑦 3𝑦
=
𝑑𝑥 2𝑥
106

This represents the slope of the given curve, 𝑚^ , so we get the negative
reciprocal of this slope to get the slope of the tangents of the orthogonal trajectories,
𝑚Z , of this curve.
𝑑𝑦 −2𝑥
=
𝑑𝑥 3𝑦
Solve this differential equation by separation of variables:
3 𝑦𝑑𝑦 + 2 𝑥𝑑𝑥 = 0
3𝑦 '
+ 𝑥' = 𝐶
2
𝟑𝒚𝟐 + 𝟐𝒙𝟐 = 𝑪

e
Us
Example 6.
Find the orthogonal trajectories to the curve 2𝑥 ' 𝑦 ' = 𝐶 − 𝑦 6 .
Solution.

l
cia
Eliminate the constant C by getting the first derivative of the curve.
er
2𝑥 ' 𝑦 ' = 𝐶 − 𝑦 6
𝑑𝑦 𝑑𝑦
4𝑥𝑦 ' + 4𝑥 ' 𝑦 = −4𝑦 k
m
𝑑𝑥 𝑑𝑥
𝑑𝑦
om

4𝑥 ' 𝑦 + 4𝑦 k = −4𝑥𝑦 '


𝑑𝑥
C

Simplify.
𝑑𝑦 −4𝑥𝑦 '
or

=
𝑑𝑥 4𝑦 𝑥 ' + 𝑦 '
𝑑𝑦 −𝑥𝑦
tF

= '
𝑑𝑥 𝑥 + 𝑦 '
This represents the slope of the given curve, 𝑚^ , so we get the negative
No

reciprocal of this slope to get the slope of the tangents of the orthogonal trajectories,
𝑚Z , of this curve.
𝑑𝑦 𝑥 ' + 𝑦 '
=
𝑑𝑥 𝑥𝑦
Solve this differential equation using the method for homogeneous equations.
Let 𝑦 = 𝑚𝑥;
𝑑𝑦 = 𝑚𝑑𝑥 + 𝑥𝑑𝑚
𝑥 𝑚𝑥 𝑚𝑑𝑥 + 𝑥𝑑𝑚 = [𝑥 ' + 𝑚𝑥 ' ]𝑑𝑥
𝑥 𝑚𝑥 𝑚𝑑𝑥 + 𝑥𝑑𝑚 = [𝑥 ' + 𝑚𝑥 ' ]𝑑𝑥
(𝑚' 𝑥 ' − 𝑥 ' − 𝑚' 𝑥 ' )𝑑𝑥 + 𝑚𝑥 k 𝑑𝑚 = 0
−𝑥 ' 𝑑𝑥 + 𝑚𝑥 k 𝑑𝑚 = 0
107

𝑑𝑥
− + 𝑚𝑑𝑚 = 0
𝑥
`g
−𝑙𝑛𝑥 + = 𝑙𝑛𝐶
'
`g
= 𝑙𝑛𝐶𝑥
'
;
Since 𝑚 = ,
"
𝑦 '
= 2𝑙𝑛𝐶𝑥
𝑥
𝒚𝟐 = 𝟐𝒙𝟐 𝒍𝒏𝑪𝒙

Example 7.
Find the orthogonal trajectories to the curve 𝑥 ' − 𝑦 ' = 𝑐𝑥

e
Solution:

Us
" g 5; g
First we isolate c=
"
" '"5';; s 5(" g 5; g )
differentiate 0=

l
cia
"g
simplify 0 = 2𝑥 − 2𝑥𝑦𝑦 A − 𝑥 ' + 𝑦 '
'

" g #; g
solving for y’ y’ =
er
'";
the slope of the orthogonal trajectories to the given curve is the negative
m
4 5'";
reciprocal of y’ 𝑦′b = − =
om

;A " g #; g
W; 5 '";
=
W"t " g #; g
C

writing equation in general form 2𝑥𝑦 𝑑𝑥 + 𝑥 ' + 𝑦 ' 𝑑𝑦 = 0


uv uw
This is a homogeneous equation but it is also exact = = 2𝑥
or

u; u"
we can solve this by the method of exact equation but it is easier to solve this
tF

by inspection
expanding 2𝑥𝑦𝑑𝑥 + 𝑥 ' 𝑑𝑦 + 𝑦 ' 𝑑𝑦 = 0
No

𝑑 ( 𝑥 ' 𝑦) + 𝑦 ' 𝑑𝑦 = 0
integrating 𝑑 𝑥 ' 𝑦 + 𝑦 ' 𝑑𝑦 = 0
4
𝑥'𝑦 + 𝑦k = 𝑘
k
3𝒙𝟐 𝒚 + 𝒚𝟑 = 𝑲
108

Activity 2
Directions: Solve the following problems completely and give two (2) points for every
item. Your total score is 10 if you will get a perfect score. Put one (1) point if you will
finish half of the items and zero (0) if no solution at all. Answers are written on the
left side corner of the questionnaire.

1. Find the isogonal trajectories of the one-parameter family of curves (𝑥 + 𝐶)𝑦 ' = 1
if
𝒅𝐲
𝜃 = 𝐴𝑟𝑐𝑡𝑎𝑛 4 . Ans. 𝒙 + 𝟒𝒚 + 𝟑𝟒ʃ =𝑪
(𝒚5𝟐)(𝒚𝟐 #𝟐𝐲#𝟒)
' '
2. Find the orthogonal trajectories of the one-parameter family 𝑦 = 𝑥 + 𝐶 .
Ans. 𝒙𝒚 = 𝑪
3. Find the orthogonal trajectories of the family of curves 𝑦 = 𝐶𝑥 ' .

e
𝟏

Us
Ans. 𝒙𝟐 + 𝒚𝟐 = 𝑪 𝒐𝒓 𝒙𝟐 + 𝟐𝒚 = 𝑪
𝟐
4. Find the orthogonal trajectories of the family of curves 𝑥 ' + 𝑦 ' = 𝐶 ' .
𝒚
Ans. 𝒍𝒏 ∣ ∣ = 𝒌 𝒐𝒓 𝒚 = 𝑪𝒙 𝒘𝒉𝒆𝒓𝒆 𝑪 = ±𝒆𝒌

l
𝒙

cia
5. Find the orthogonal trajectories of the family of curves 𝑥 ' + 𝑦 ' = 𝐶𝑥 .
Ans. 𝒙𝟐 + 𝒚𝟐 = 𝑪𝒚
er
m
Progress Indicators: (See Activity 1)
om

Watch: Youtube
1. Differential equations introduction (video) Khan Academy
C

2. Youtube what is differential equation


3. Youtube Differential equations, studying the unsolvable.
or
tF
No
109

5.3 Newton’s Law of Cooling (and Heating)

Based on studies, it was noted that an approximation of the temperature of a


body undergoing a cooling or a heating process could be obtained by using Newton’s
Law of Cooling. The Law states that the rate of change of the body temperature is
proportional to the difference of the temperature of the body and the temperature of
the surrounding medium. It must be pointed out that several assumptions are to be
made so as to simplify the differential equations that would result. Among these is
that the constant of proportionality is the same for both heating and cooling. Another
is that the temperature of the medium is maintained constant. To simplify the
differential equation to be processed, the condition is that the heat released or
absorbed by the body is not large enough to cause a subsequent increase or decrease
in the medium temperature. The law stated in differential equation from is:

e
𝑇" = 𝑡𝑒𝑚𝑝𝑒𝑟𝑎𝑡𝑢𝑟𝑒 𝑜 𝑡ℎ𝑒 𝑏𝑜𝑑𝑦

Us
𝑇1 = 𝑡𝑒𝑚𝑝𝑒𝑟𝑎𝑡𝑢𝑟𝑒 𝑜𝑓 𝑡ℎ𝑒 𝑚𝑒𝑑𝑖𝑢𝑚

l
𝑇" −𝑇1 = 𝑡𝑒𝑚𝑝𝑒𝑟𝑎𝑡𝑢𝑟𝑒 𝑑𝑖𝑓𝑓𝑒𝑟𝑒𝑛𝑐𝑒
𝑑𝑇"
= 𝑟𝑎𝑡𝑒 𝑜𝑓 𝑐ℎ𝑎𝑔𝑒 𝑜𝑓 𝑇" cia
er
𝑑𝑡
𝑑𝑇"
m
𝛼 𝑇" −𝑇1
𝑑𝑡
om

𝒅𝑻𝒃
= −𝒌(𝑻𝒃 −𝑻𝒎 ), General formula
𝒅𝒕
C

where k= constant of proportionality


or

The figure below shows the general shape of T(t) when 𝑇" > 𝑇1 (i.e., in cooling
tF

scenario) and when 𝑇" < 𝑇C (i.e., in a heating scenario): See Figure 2.
No

𝑇" > 𝑇1
Cooling Process
𝑇1
------------------------------------------------------------

Heating Process
𝑇" < 𝑇1

Figure 2
110

Example 8.
A thermometer reading is 18F 𝐹 is brought into a room temperature where the
temperature is 70F 𝐹; 1 minute later, the thermometer reading is 31F 𝐹. Determine the
temperature reading as a function of time and, in particular, the temperature reading
5 minutes after the thermometer is brought into the room.
Solution.
𝑑𝑇"
= 𝑘(𝑇" −𝑇1 )
𝑑𝑡
NOTE: k is positive because the object (thermometer) is on the heating process
where 𝑇" > 𝑇1 .
𝑑𝑇"
= 𝑘(𝑇" −𝑇1 )
𝑑𝑡

e
Separate the variables and integrate.

Us
𝑑𝑇"
= 𝑘𝑑𝑡
(𝑇" −𝑇1 )

l
𝑙𝑛(𝑇" −𝑇1 ) = 𝑘𝑡 + 𝑙𝑛𝐶
𝑻𝒃 −𝑻𝒎 = 𝑪𝒆𝒌𝒕
cia General Formula
er
Solving for C:
m
When 𝑡 = 0 (start), 𝑇" = 18F 𝐹, 𝑇1 = 70F 𝐹 (Substitute in general formula)
om

18 − 70 = 𝐶𝑒 Q(R)
𝐶 = −52
C

Thus, 𝑻𝒃 − 𝟕𝟎 = −𝟓𝟐𝒆𝒌𝒕 Equation 1


F
When t=1 min, 𝑇" = 31 𝐹 (Substitute in Equation 1)
or

31 − 70 = −52𝑒 Q(Y)
tF

−39 = −52𝑒 Q
[\
𝑘 = 𝑙𝑛 , Substitute in equation 1
No

]^

𝟑𝟗
Therefore, 𝑻𝒃 − 𝟕𝟎 = −𝟓𝟐𝒆(𝒍𝒏𝟓𝟐)𝒕 Equation 2
a. After 5 minutes
[\
𝑇" − 70 = −52𝑒 (cd]^)(])
ef
𝑇" = 70 − 52𝑒 (cdgh)(])
𝑻𝒃 = 𝟓𝟕. 𝟔𝟔𝒐 𝑭 ≈ 𝟓𝟖𝒐 𝑭
111

Example 9.
A body at a temperature of 50F 𝐹 is placed in an oven whose temperature is
kept at 150F 𝐹. If after 10 minutes, the temperature of the body is 75F 𝐹, find an
expression for the temperature of the body at time t.
Solution.
The temperature of the surrounding medium is the temperature of the oven,
which is held constant at 150F 𝐹. Analyzing the problem, a body having a lower
temperature was brought to an atmosphere whose temperature is higher. Therefore,
it is heating.
Thus, the resulting equation is:
𝑑𝑇"
= −𝑘(𝑇1 −𝑇" )
𝑑𝑡
𝑑𝑇"

e
= −𝑘(150 − 𝑇" )

Us
𝑑𝑡
Separate the variables and integrate.
𝑑𝑇"
= − 𝑘 𝑑𝑡

l
(150 − 𝑇" )

Solving for C: cia


𝑙𝑛 150 − 𝑇" = −𝑘𝑡 + 𝑙𝑛𝐶
er
When t=0 (start), 𝑇" = 50F 𝐹
m
𝑙𝑛 150 − 50 = −𝑘(0) + 𝑙𝑛𝐶
𝑙𝑛𝐶 = 𝑙𝑛100
om

Thus,
C

𝑙𝑛 150 − 𝑇" = −𝑘𝑡 + 𝑙𝑛100


150 − 𝑇"
or

𝑙𝑛 = −𝑘𝑡
100
tF

To solve for k, substitute the second condition.


When t=10, 𝑇" = 75F 𝐹
No

150 − 75
𝑙𝑛 = −𝑘 10
100
R.n]
𝑘 = −𝑙𝑛
YR
𝒌 = 𝟎. 𝟎𝟐𝟖𝟕𝟕

Therefore, 𝑻𝒃 = 𝟏𝟓𝟎 − 𝟏𝟎𝟎𝒆𝟎.𝟎𝟐𝟖𝟕𝟕𝒕


112

Example 10.
A thermometer reading 40F 𝐶 is brought inside a room where the air
temperature is a cool 20F 𝐶. One minute later, it reads 35F 𝐶.
a. What will be the temperature of the thermometer after 5 minutes?
b. When will the reading be 25F 𝐶? When will the temperature of the body equal
the air temperature?
Solution.
𝑑𝑇"
= −𝑘(𝑇" −𝑇1 )
𝑑𝑡
𝑑𝑇"
= −𝑘(𝑇" − 20)
𝑑𝑡
Separate the variables and integrate.
𝑑𝑇"

e
= − 𝑘𝑑𝑡

Us
(𝑇" − 20)
𝑙𝑛 𝑇" − 20 = −𝑘𝑡 + 𝑙𝑛𝐶

l
cia
Solving for C:
When t=0 (start), 𝑇" = 40F 𝐶
er
𝑙𝑛 40 − 20 = −𝑘(0) + 𝑙𝑛𝐶
𝑙𝑛𝐶 = 𝑙𝑛20
m
Thus,
om

𝑙𝑛 𝑇" − 20 = −𝑘𝑡 + 𝑙𝑛20


𝑇" − 20
C

𝑙𝑛 = −𝑘𝑡
20
or

To solve for k, substitute the second condition.


tF

When t=1, 𝑇" = 35F 𝐶


35 − 20
𝑙𝑛 = −𝑘 1
No

20
Y]
𝑘 = −𝑙𝑛
^R

𝒌 = 𝟎. 𝟐𝟖𝟕𝟔𝟖

Therefore, 𝑻𝒃 = 𝟐𝟎 + 𝟐𝟎𝒆𝟎.𝟐𝟖𝟕𝟔𝟖𝒕

a. After 5 minutes
𝑇" = 20 + 20𝑒 R.^qnrq(])
𝑻𝒃 = 𝟏𝟎𝟒. 𝟐𝟖𝒐 𝑪
113

b. we use the logarithmic form of the equation if time is to be determined


𝑇" − 20
𝑙𝑛 = −𝑘𝑡
20
1 𝑇" − 20
𝑡 = − 𝑙𝑛
𝑘 20
For 𝑇" = 25F 𝐶;
1 25 − 20
𝑡=− 𝑙𝑛
0.28768 20
𝒕 = 𝟒. 𝟖𝟐 𝒎𝒊𝒏𝒖𝒕𝒆𝒔

c. Analyzing the formula


when 𝑇" = 𝑇1 = 20F 𝐶 will yield the value of infinity.

e
Us
𝑙𝑛𝑇" = −𝑘𝑡
𝒕=𝜶

l
cia
Note: Which means that it will take a very long time for the body temperature to
equal the air temperature.
er
Activity 3
m
Directions: Solve the following problems completely and accurately, and give two (2)
points for every answers. Your total score is 10 if you will get a perfect score. Give
om

one (1) point if your solution is not complete, zero (0) if no solution at all. Answers are
written on the right side corner of the questionnaire. Try to double check your work.
C

Relax and keep cool!


or

1. When a cake is removed from an oven, its temperature is measured at 300F 𝐹. Three
tF

minutes later, its temperature is 200F 𝐹. How long will it take for the cake to cool off
to a room temperature of 70F 𝐹?
Ans. About half an hour or 32.3 minutes
No

2. Assume that a refrigerator is maintained at a constant temperature of 45F 𝐹 and


that an object having a temperature of 80F 𝐹 is placed inside the refrigerator. If the
temperature of the object drops from 80F 𝐹 to 70F 𝐹 in 15 minutes, how long will it take
for the object’s temperature to decrease to 60F 𝐹?
Ans. 22.77 minutes
F F
3. A 5-lb roast, initially at 50 𝐹, is put into a 375 𝐹 oven at 5:00 PM; it is found that
the temperature T(t) of the roast is 125F 𝐹 after 75 minutes. When will the roast be
150F 𝐹?
Ans. Around 6:45 PM
4. At 12:00 midnight, with the temperature inside the warehouse at 70F 𝐹 and the
outside temperature at 20F 𝐹 the furnace breaks down. Two hours later, the
114

temperature in the building has fallen to 50F 𝐹. Determine when the temperatures in
the warehouse fall to 40F 𝐹.
Ans. Around 3:36 AM
F
5. A metal bar at a temperature of 100 𝐹 is placed in a room at a constant
temperature of 0F 𝐹. If after 20 minutes, the temperature of the bar is 50F 𝐹, find the
temperature of the bar after 10 minutes.
Ans. 𝟕𝟎. 𝟓𝒐 𝑭

Progress Indicators: (See Activity 1)

Watch: Youtube
1. Newton’s Law of Cooling Khan Academy.org

e
2. Newton’s Law of Cooling AIRichards314

Us
3. Unit 16-2 Newton’s Law of Heating and Cooling- Math121 Alan Ableson

l
5.4 Exponential Law of Growth and Decay
cia
er
The exponential law of growth and decay states that the rate of change of the
number of bacteria/ radio actives present at any time t, is directly proportional to the
m

number of bacteria present at any time t.


om

Problems involving radioactive decays and bacterial growths are the usual
problem of this type. It is very useful in the field of science like chemistry, nuclear
C

physics and nuclear medicine.


or

5.4a. Radioactive Decay


tF

Let x= amount of unconverted substance present at any time t.


No

k= proportionality constant (property of substance involved).


t= time
𝒅𝐱
= −𝒌𝒙
𝒅𝒕

Note: The negative sign indicates that as time goes on, the amount of radioactive
material is diminishing.
115

Example 11.
Geologist exploring the foothills of Mount Von Bulow in west coast New
Zealand have unearthed a piece of rock with residues of a certain radioactive
substance. Preliminary estimates have indicated that the substance have an
approximate half-life of 60 hours. Calculate:
a. the amount left after 7 days;
b. how long will it take for 90% of the radioactivity be dissipated?
Solution.
Since the element is dissipating, the problem is an exponential decay.
𝑑𝑥
= −𝑘𝑥
𝑑𝑡
𝑑𝑥
= − 𝑘𝑑𝑡

e
𝑥

Us
ln 𝑥 = −𝑘𝑡 + 𝑙𝑛𝐶
At t=0; 𝑥 = 𝑥.
𝑙𝑛 𝑥. = −𝑘(0) + 𝑙𝑛𝐶

l
cia
𝑥. = 𝐶
𝒙 = 𝒙𝟎 𝒆5𝒌𝒕
er
The half-life is a property of radioactive nuclides which measures their degree
m
of radioactivity. The half-life is defined as the time it takes for a radioactive material
om

to decay to half of its original value.


0.5 = 𝑒 5;<
𝑙𝑛0.5 = −𝑘(60ℎ𝑟𝑠)
C

ln (0.5)
𝑘=−
60
or

𝒌 = 𝟎. 𝟎𝟏𝟏𝟓𝟓/𝒉𝒓
tF

a. After 7 days.
No

.FFGG OPHIN
5.. (JKLMN)( )
𝑥 = 𝑥. 𝑒 HI KLM

𝐱 = 𝟎. 𝟏𝟒𝟑𝟔
14.36% of original amount will be left.

b. if 90% of radio nuclides is dissipated, 10% remains


thus, 𝑥 = 0.10𝑥.
𝑙𝑛0.10𝑥.
= −0.01155𝑡
𝑥.
ln (0.10)
𝑡=
−0.01155
𝒕 = 𝟏𝟗𝟗. 𝟑𝟔 𝒉𝒐𝒖𝒓𝒔
116

Example 12.
Radium decomposes at a rate proportional to the amount present. If 100mg set
aside now there will be left 96mg 100 years hence, find:
a. how much will be left t?
b. how much will be left after 2.58 centuries; and
c. the half-life of radium?
Solution.
Since the element is decomposing, the problem is an exponential decay.
𝑑x
= −𝑘𝑥
𝑑𝑡
𝑑𝑥
= − 𝑘𝑑𝑡

e
𝑥

Us
𝑙𝑛𝑥 = −𝑘𝑡 + 𝑙𝑛𝐶
At t=0; 𝑥 = 𝑥.
𝑙𝑛𝑥. = −𝑘(0) + 𝑙𝑛𝐶

l
𝑥. = 𝐶
cia
𝒙 = 𝒙𝟎 𝒆5𝒌𝒕
er
It is stated in the problem that from 100mg, it was reduced to 96mg after 100 years.
m

Thus,
om

96 = 1005F..;
𝑙𝑛0.96 = −𝑘(100 𝑦𝑒𝑎𝑟𝑠)
ln (0.96)
C

𝑘=−
100
or

𝒌 = 𝟎 − 𝟎. 𝟎𝟎𝟎𝟒𝟎𝟖𝟐𝟐/𝒚𝒆𝒂𝒓
tF

a. After 2.58 centuries


McLIN
5.....P.bOO/McLI(O.Gb dce<fIgcN)(F.. )
𝑥 = 100𝑒 dce<fIM
No

𝑥 = 𝟗𝟎. 𝟎𝟎𝟑𝟔𝐦𝐠
90.0036mg of radium will be left after 2.58 centuries of original amount will be left

b. if 50% of radium is decomposed, 50% remains


thus, 𝑥 = 0.50𝑥.
𝑙𝑛0.50𝑥.
= −0.00040822𝑡
𝑥.
ln (0.50)
𝑡=
−0.00040822
𝒕 = 𝟏𝟔𝟗𝟕. 𝟗𝟕 𝒚𝒆𝒂𝒓𝒔 𝒐𝒓 𝟏𝟕 𝒄𝒆𝒏𝒕𝒖𝒓𝒊𝒆𝒔
117

Example 13.
Bismuth-210 has a half-life of 5.0 days.
a. Suppose a sample originally has a mass of 800mg. Find a formula for the mass
remaining after t days.
b. Find the mass remaining after 30 days
c. when is the mass reduced to 1 mg.
Solution.
Kq
a. = −𝑘𝑥
K<
𝑑𝑥
= − 𝑘𝑑𝑡
𝑥
𝑙𝑛𝑥 = −𝑘𝑡 + 𝑙𝑛𝐶
At t=0; 𝑥 = 𝑥.

e
𝑙𝑛𝑥. = −𝑘(0) + 𝑙𝑛𝐶

Us
𝑥. = 𝐶
𝒙 = 𝒙𝟎 𝒆5𝒌𝒕

l
Since 𝑥. = 800;

cia
𝒙 = 𝟖𝟎𝟎𝒆5𝒌𝒕
To complete the equation that models this population, we need to find the
er
relative decay rate k. We can use the half-life formula of the substance to do this. The
m
half-life of Bismuth-210 is 5 days. This says that after t=5, the original population of
800mg has decay to half of its original amount, or 400mg. Mathematically, since x
om

represents that amount population of the substance left after time t, this says that
x= 400. Using the decay equation, we have
C

400 = 800𝑒 5G;


Then, solve for k;
or

400
𝑒 5G; =
800
tF

𝑒 5G; = 0.5
𝑙𝑛𝑒 5G; = 𝑙𝑛(0.5)
No

−5𝑘𝑙𝑛𝑒 = 𝑙𝑛(0.5)
−5𝑘(1) = 𝑙𝑛(0.5)
𝑙𝑛 0.5
𝑘=−
5
𝒌 = 𝟎. 𝟏𝟑𝟖𝟔
Therefore,
𝒙 = 𝟖𝟎𝟎𝒆5𝟎.𝟏𝟑𝟖𝟔𝒕

b. when t=30
𝒙 = 800𝑒 5..Frbs(r.)
𝒙 = 𝟏𝟐. 𝟓𝟏 𝐠𝐫𝐚𝐦𝐬
118

c. when is the mass reduced to 1 mg


In this problem, we want the time t it takes for the mass to have reduced down
to 1 mg. that is, we want t when x=1. To solve for t, we perform the following steps
using ,
x = 800𝑒 5..Frbs<
1 = 800𝑒 5..Frbs<
1
𝑒 5..Frbs< =
800
1
𝑙𝑛𝑒 5..Frbs< = 𝑙𝑛
800
1
−0.1386 𝑙𝑛𝑒 = 𝑙𝑛
800
1

e
−0.1386𝑡(1) = 𝑙𝑛
800

Us
1
𝑙𝑛
𝑡 = − 800
0.1386

l
cia
𝒕 = 48.23
Thus, it takes approximately 48.23 days for the substance to decay to 1 mg.
er
Example 14.
m
One hundred grams of an unspecified radioactive material was brought in a
laboratory and tests indicate that it has a half-life of 33 years. How much of the
om

material will remain after 5 years? How long will it take to decay 35% of this
material?
C

Solution:
yz O
First, we determine the decay constant 𝑘 , 𝑘 = = 0.021/yr
or

rr
a) when t = 5 years,
tF

x = 𝑥{ 𝑒 -;< = 100 ( 𝑒 -(...OF)(G) ) = 90 grams


b) when x = 65 grams,
No

}
− k t = ln
}~
sG
− (0.021) t = ln
F..
t = 20.5 years
119

5.4b Bacterial Growth

Let x = amount of unconverted substance present at any time t.


k = proportionality constant (property of substance involved).
t = time
𝒅𝒙
= 𝒌𝒙
𝒅𝒕

Note: The positive sign indicates that as time goes on, the amount of bacteria is
increasing.

Example 15.

e
Us
Researchers at the Institute of Disease Control have isolated a certain
bacterial strain that follows the exponential growth curve. In their study, the bacteria
have doubled its population in just 2 hours. Assuming other controlling factors

l
cia
considered constant,
a. what will be the expected population after 6 hours?
er
b. when will the population be a hundred times that what it was originally?
Solution.
m
𝑑𝑥
= 𝑘𝑥
om

𝑑𝑡
𝑑𝑥
= 𝑘𝑑𝑡
𝑥
C

𝑙𝑛x = 𝑘𝑡 + 𝑙𝑛𝐶
𝑒 €e} = 𝑒 ;<•€ed
or

𝑥 = 𝑪𝒆𝒌𝒕
tF

when t=0, 𝑥 = 𝑥. (original amount)


No

𝑥. = 𝐶𝑒 ;(.)
𝑥. = 𝐶
𝑥 = 𝑥. 𝑒 ;<
Then we solve for k.
when t=2 hours, 𝑥 = 2𝑥. ;
2𝑥. = 𝑥. 𝑒 O;
1
𝑘 = 𝑙𝑛2
2
𝒌 = 𝟎. 𝟑𝟒𝟔𝟔/𝐡𝐫

Therefore, the equation for the exponential growth is;


𝐱 = 𝒙𝟎 𝒆𝟎.𝟑𝟒𝟔𝟔𝒕
120

a. when t=6 hours


𝐱 = 𝒙𝟎 𝒆𝟎.𝟑𝟒𝟔𝟔(𝟔)
𝐱 = 𝟖𝒙𝟎 , the population will be eight times the original population.

b. when x = 100𝑥.
𝑥 = 𝑥. 𝑒 ..rPss<
100𝑥. = 𝑥. 𝑒 ..rPss<
𝑒 ..rPss< = 𝑙𝑛100
𝑙𝑛100
𝑡=
0.3466
𝒕 = 𝟏𝟑. 𝟐𝟗 𝒉𝒐𝒖𝒓𝒔

e
Us
Example 16.
A certain population of bacteria is known to grow at a rate proportional to the
amount present in a culture that provides plentiful food and space. Initially, there

l
cia
are 250 bacteria and after 7 hours, 800 bacteria are observed in the culture.
a. Find an expression for the approximate number of bacteria present in the
er
culture at any time t.
b. Approximate the number of bacteria present in the culture after 24 hours.
m

Solution.
om

𝑑x
= 𝑘𝑥
𝑑𝑡
𝑑𝑥
C

= 𝑘𝑑𝑡
𝑥
𝑙𝑛x = 𝑘𝑡 + 𝑙𝑛𝐶
or

𝑒 €e} = 𝑒 ;<•€ed
tF

𝑥 = 𝐶𝑒 ;<
when t=0, 𝑥 = 𝑥. (original amount)
No

𝑥. = 𝐶𝑒 ;(.)
𝑥. = 𝐶
𝒙 = 𝒙𝟎 𝒆𝒌𝒕

Then we solve for k.


when t=7 hours, 𝑥 = 250, 𝑥. = 800
250 = 800𝑒 J;
1 16
𝑘 = 𝑙𝑛
7 5
𝒌 = 𝟎. 𝟏𝟔𝟔/𝐡𝐫
121

a. Therefore the equation for the exponential growth is;


𝒙 = 𝒙𝟎 𝒆𝟎.𝟏𝟔𝟔𝒕

b. when t=24 hours


𝑥 = 𝑥. 𝑒 ..Fss(OP)
𝑥 = 53.73𝑥.
𝒙 = 𝟏𝟑𝟒𝟑𝟑 𝐛𝐚𝐜𝐭𝐞𝐫𝐢𝐚 𝐚𝐫𝐞 𝐩𝐫𝐞𝐬𝐞𝐧𝐭

Example 17.
Υ
The number of bacteria in a culture is growing at a rate of 3000𝑒 Ž per unit
time t. At t=0, the number of bacteria present was 7,500. Find the number present
at t=5.

e
Solution.

Us
𝑑𝑥 O<
= 3000𝑒 G
𝑑𝑡

l
O<

cia
d𝑥 = 3000𝑒 G 𝑑𝑡
O< 5
𝑥 = 3000𝑒 G ( )
er
2
O<
m
𝑥 = 7500𝑒 G
O(G)
om

𝑥 = 7500𝑒 G
𝒙 = 𝟓𝟓𝟒𝟏𝟖 bacteria are present
Example 18.
C
or

A certain bacterial organism quadruples its population 𝑃 every 3 hours. How many
hours are needed to make the population be 100 times than the original population
tF

𝑃. ? What is the population after a day?



Solution: Using ln = 𝑘 𝑡, we substitute the given values to solve for 𝑘,
‘~
No

P‘~
ln = k (3)
‘~
thus, 𝑘 = 0.462/hr
yz F..
a) when P = 100𝑃{ , t = = 9.966 hours
..PsO
b) when t = 24 hours,
P = 𝑃{ 𝑒 (..PsO)(OP) = 65.38 𝑃{
122

Activity 4

Directions: Solve the following problems completely and give two (2) points for every
item. Your total score is 10 if you will get a perfect score. Put one (1) point if you will
finish half of the items and zero (0) if no solution at all. Answers are written on the
left side corner of the questionnaire.

1. A certain population of bacteria is known to grow at a rate proportional to the


amount present in a culture that provides plentiful food and space. Initially there are
250 bacteria, and after seven hours 800 bacteria are observed in the culture. Find an
expression for the approximate number of bacteria present in the culture at any time
t.
Ans. 𝑵 = 𝟐𝟓𝟎𝒆𝟎.𝟏𝟔𝟔𝒕

e
Us
2. Determine the approximate number of bacteria that will be present in the culture
described in problem 1 after 24 hours.
Ans. 𝑵 = 𝟏𝟑 𝟒𝟑𝟑 𝐛𝐚𝐜𝐭𝐞𝐫𝐢𝐚

l
cia
3. A bacteria culture is known to grow at a rate proportional to the amount present.
After one hour, 1000 bacteria are observed in the culture; and after four hours, 3000.
Find an expression for the number of bacteria present in the culture at any time t.
er
Ans. 𝑵 = 𝟔𝟗𝟒𝒆𝟎.𝟑𝟔𝟔𝒕
m
4. A certain radioactive material is known to decay at a rate proportional to the
amount present. If initially there is 100 mg of the material present and if after 2 years
om

I is observed that 5 percent of the original mass has decayed, find an expression for
the mass at any time t.
C

Ans. 𝑵 = 𝟏𝟎𝟎𝒆5𝟎.𝟐𝟓𝟔𝒕 , t in years


5. A certain radioactive material is known to decay at a rate proportional to the
or

amount present. If initially there is 50 mg of the material present and after 2 hours
it is observed that the material has lost 10 percent of its original, find an expression
tF

for the mass of the material remaining at any time t.


Ans. 𝑵 = 𝟓𝟎𝒆5𝟎.𝟎𝟓𝟑𝒕 , t in hours
No

Progress Indicators: (See Activity 1)


123

5.5 Simple Electric Circuits

Electricity is a substance which flows through conductors such as wires. One


unit of electricity is coulomb. The rate of flow of electricity is called current. If x
coulomb of electricity per second are passing a point in a conductor, the current in
the conductor is x ampere.

e
l Us
cia
Figure 3
er
For a simple electric circuit,
m
𝒅𝐢 𝒒
𝐿 + 𝑹𝒊 + = 𝑬
𝒅𝒕 𝒄
om

Where:
𝒅𝐪
𝑖=
C

𝒅𝒕
E = emf or electromotive force (in volts).
or

I = current (in amperes)


Q = charge or the quantity of electricity on the capacitor (in coulombs).
tF

L = constant; inductance to mass or inertia (in henrys).


R = constant; resistance to friction (in ohms).
No

and C = constants; capacitance to the size of the storage tank (in farads).

𝐪
If there is no capacitor in the circuit, the equation will no longer contain the term .
𝒄

In this section, we consider the application of differential equations to series


circuits containing (1) an electromotive force, and (2) resistors, inductors, and
capacitors. We assume that the reader is somewhat familiar with these items and so
we shall avoid an extensive discussion. Let us simply recall that the electromotive
force (for example, a battery or generator) produces a flow of current in a closed circuit
and that this current produces a so-called Voltage Drop across each resistor,
inductor, and capacitor. Further, the following three laws concerning the voltage
drops across these various elements are known to hold:
124

1.The voltage drop across a resistor is given by


𝑬𝑹 = 𝑹𝒊
where R is a constant of proportionality called the resistance, and i is the current.

2. The voltage drop across an inductor is given by


𝒅𝒊
𝑬𝑳 = 𝑳
𝒅𝒕
where L is a constant of proportionality called the inductance, and i again denotes
the current.

3. The voltage drop across a capacitor is given by


𝟏
𝑬𝒄 = 𝒒

e
𝑪

Us
where C is a constant of proportionality called the capacitance and q is the
𝒅𝒒
instantaneous charge on the capacitor. Since 𝒊 = , this is often written as
𝒅𝒕
𝟏

l
cia
𝑬𝒄 = ʃ𝒊𝒅𝒕
𝑪
The units in common use are listed in Table 1 below:
er
TABLE 1
m

Quantity and Unit


om

Symbol
emf or voltage volt (V)
C

E
current i ampere
or

charge q coulomb
tF

resistance R ohm (Ω)


inductance L henry (H)
No

capacitance C farad

The fundamental law in the study of electric circuits is in the following:

Kirchhoff’s Voltage Law (Form 1). The algebraic sum of the instantaneous
voltage drops around a closed circuit in a specific direction is zero.

Since voltage drops across resistors, inductors, and capacitors have the
opposite sign from voltages arising from electromotive forces, we may state this law
in the following alternative form:
125

Kirchhoff’s Voltage Law (Form 2). The sum of the voltage drops across
resistors, inductors, and capacitors is equal to the total electromotive force in a closed
circuit.

E C

e
L

l Us
cia
Figure 5

Let us apply Kirchhoff’s Law to the circuit of Figure 5. Letting E denote the
er
electromotive force, and using the laws 1, 2 and 3 for voltages that were given above.
We are led at once to the equation;
m

𝒅𝒊 𝟏
𝑳 + 𝑹𝒊 + 𝒒 = 𝑬
om

𝒅𝒕 𝑪
This equation contains two dependent variables i and q. However, we recall
that these two variables are related to each other by the equation
C

𝒅𝒒
𝒊=
or

𝒅𝒕
Eliminate i from equation 2, forming a second-order linear differential equation
tF

𝒅𝟐 𝒒 𝒅𝒒 𝟏
𝑳 𝟐 +𝑹 + 𝒒=𝑬
𝒅𝒕 𝒅𝒕 𝑪
No

Differentiate equation 3, forming a second-order linear differential equation


in the single dependent variable i.
𝒅𝟐 𝒊 𝒅𝒊 𝟏
𝑳 𝟐 +𝑹 + 𝒊=𝑬
𝒅𝒕 𝒅𝒕 𝑪
If the circuit contains no capacitor, it reduces directly to;
𝒅𝒊
𝑳 + 𝑹𝒊 = 𝑬
𝒅𝒕
While if no inductor is present, it reduces to;
𝒅𝒒 𝟏
𝑹 + 𝒒=𝑬
𝒅𝒕 𝑪
126

Figure 4

e
Us
ELECTRICAL CIRCUIT
SYMBOLS

l
cia
The electrical system described above is analogous to mechanical system.
er
TABLE 2
m

Mechanical System Electrical System


om

mass m inductance L
damping constant a resistance R
C

spring constant k Reciprocal of capacitance


1/C
or

impressed force F(t) impressed voltage or emf E


tF

displacement x charge q
velocity v=dx/dt current i=dq/dt
No

Example 19.
𝒅𝐢
Solve 𝐿 + 𝑹𝒊 = 𝑬 for i in terms of t and constants L, R, and E if i=0 when t=0.
𝒅𝒕
Solution.
𝑑i
𝐿 + 𝑅𝑖 = 𝐸
𝑑𝑡
𝑑i 𝑅𝑖 𝐸
+ =
𝑑𝑡 𝐿 𝐿
𝑑i 𝑅 𝐸
+ ( )𝑖 =
𝑑𝑡 𝐿 𝐿
=
𝜆 = 𝑒 ʃ > ?@
127

=@
𝜆=𝑒>
=@ =@ 𝐸
𝑖𝑒 > = ʃ𝑒 > 𝑑𝑡
𝐿
=@ E =@
𝑖𝑒 > = 𝑒 > + 𝐶D
R
𝐄 𝑹𝒕
𝒊 = + 𝑪 𝟏 𝒆H 𝑳
𝐑
Substitute t=0, i=0;
E =(J)
0= + 𝐶D 𝑒 H >
R
0 = E + 𝑅𝐶D
𝐸
𝐶D = −
𝑅

e
Therefore, the solution is:

Us
E =@
𝑖= + 𝐶D 𝑒 H >
R
E 𝐸 H=@

l
𝑖= −

cia
𝑒 >
R 𝑅D
𝐄 𝑹𝒕
𝒊 = (𝟏 − 𝒆H 𝑳 )
er
𝐑
m

Example 20.
om

Find i when t=0.1 second if L=2 henrys, R= 20 ohms, and


a. E= 10 volts; and
b. E= 100 volts.
C

Solution.
or

From:
E =@
tF

𝑖 = (1 − 𝑒 H > )
R
(NO)(O.Q) (NO)(O.Q)
DJ DJJ
a. 𝑖 = (1 − 𝑒 H N ) b. 𝑖 = (1 − 𝑒 H N )
No

MJ MJ
𝒊 = 𝟎. 𝟑𝟔𝟏 volt 𝒊 = 𝟑. 𝟔𝟏 volt

Example 21.
Find the inductance L if R= 20 ohms and i= 1 ampere when t= 0.01 second and
a. E= 40 volts
b. E= 200 volts

Solution.
From:
E =@
𝑖 = (1 − 𝑒 H > )
R
123

(NO)(O.OQ) (NO)(O.OQ)
UJ MJJ
a. 𝑖 = (1 − 𝑒 H V ) b. 𝑖 = (1 − 𝑒 H V )
MJ MJ
20 J.M 20 J.M
= 1 − 𝑒H > = 1 − 𝑒H >
40 200
J.M J.M
𝑒 H > = 0.5 𝑒 H > = 0.9
0.2 0.2
− = 𝑙𝑛 0.5 − = 𝑙𝑛 0.9
𝐿 𝐿
0.2 0.2
𝐿=− 𝐿=−
𝑙𝑛 0.5 𝑙𝑛 0.9
𝑳 = 𝟎. 𝟐𝟖𝟖𝟓 𝒉𝒆𝒏𝒓𝒚 𝑳 = 𝟏. 𝟖𝟗𝟖𝟐 𝒉𝒆𝒏𝒓𝒚

Activity 5
Directions: Solve the following problems completely and give two (2) points for every

e
item. Your total score is 10 if you will get a perfect score. Put one (1) point if you will

Us
finish half of the items and zero (0) if no solution at all. Answers are written on the
left side corner of the questionnaire.

l
2 Ω, an inductor of 0.1 H, and a capacitor of cia
1. A circuit has in series an electromotive force given by E=100sin 60t V, a resistor of
D
farads. If the initial current and the
er
MdJ
initial charge on the capacitor are both zero, find the charge on the capacitor at any
m
time 𝑡 > 0.
Ans. 𝒒 = 𝟎. 𝟕𝟕𝒆H𝟏𝟎𝒕 𝐜𝐨𝐬 𝟓𝟎𝒕 − 𝟎. 𝟖𝟖 − 𝟎. 𝟔𝟒 𝐜𝐨𝐬 (𝟔𝟎𝒕 − 𝟎. 𝟔𝟗)
om

2. An RL circuit has an emf given (in volts) by 3sin 2t, a resistance of 10 Ω, an


C

inductance of 0.5 H, and an initial current of 6 A. Find the current in the circuit at
any time t.
or

𝟔𝟎𝟗 𝟑𝟎 𝟑
Ans. 𝑰 = 𝒆H𝟐𝟎𝒕 + 𝒔𝒊𝒏𝟐𝒕 − 𝒄𝒐𝒔𝟐𝒕
tF

𝟏𝟎𝟏 𝟏𝟎𝟏 𝟏𝟎𝟏


3. Rewrite the steady-state current of Problem 2 in the form 𝑨𝒔𝒊𝒏(𝟐𝒕 − 𝜽). The angle
θ is called the phase angle.
No

𝟏
Ans. 𝜽 = 𝒂𝒓𝒄𝒕𝒂𝒏
𝟏𝟎
4. A resistor of 15 Ω and an inductance of 3 H are connected in series with a 60-Hz
sinusoidal voltage source having amplitude 110 V. Find an expression for the steady-
state current at any time t if initially there is no current in the system.
𝟐𝟐 𝒔𝒊𝒏 𝟏𝟐𝟎п𝒕H𝟐𝟒п𝒄𝒐𝒔𝟏𝟐𝟎п𝒕
Ans. 𝑰 = ( )
𝟑 𝟏r𝟓𝟕𝟔п𝟐
5. An RL circuit with no source of emf has an initial current given by 𝑰𝟎 . Find the
current at any time t.
𝑹
𝒕
Ans. 𝑰 = 𝑰𝟎 𝒆 𝑳

Progress Indicators: (See Activity 1)


129

5.6 Mixture Problems

A very useful application of first-order differential equation solution is in the


determination of the amount of solute as a function of time for a problem involving
the mixing of two different solutions. Customarily, for this type of problem a brine
solution is mixed with freshwater or another brine solution of a different
concentration. The object of the problem is to determine the concentration of the
mixture with respect to the time that has elapsed. To simplify the problem, it is
assumed that only physical mixing is involved (no chemical reactions) and that the
mixture inside the tank is completely mixed, that is, the concentration of the brine
solution in the tank is uniform all over. (See Figure 6)

e
Us
Incoming
𝐶' & 𝑅'

l
cia
er
m
S
om

=
Outgoing 𝐶) & 𝑅)
Depends upon on
C

Figure 6 the condition


or
tF

Let S = amount of substance in the mixture at any instant


!"
= rate of change of S (amount gained − amount lost)
No

!#
𝐶' = concentration of incoming solution
𝑅' = rate of inflow
𝐶) = concentration of outgoing solution
𝑅) = rate of outflow
𝒅𝐒
= 𝑪𝒊 𝑹𝒊 − 𝑪𝒐 𝑹𝒐 General formula
𝒅𝒕

𝐒
Note: 𝑪=
𝑽
where: 𝐶 = concentration of solution
𝑆 = amount of substance
𝑉 = volume of solution
130

There are 3 possible conditions:


1. If 𝑹𝒊 > 𝑹𝒐 , then V increases, the tank may overflow.
2. If 𝑹𝒊 = 𝑹𝒐 , then V remains constant, water level doesn’t increase or decrease.
3. If 𝑹𝒊 < 𝑹𝒐 , then V decreases, the tank may become empty.

Example 22.
A certain manufacturing plant has a mixing tank initially filled with 100
gallons of brine containing 1 lb of salt per gallon of solution. Brine from another
processing area containing 2 lbs of salt per gallon is piped into this tank at the rate
of 5 gallons per minute. The mixture is kept uniform by constant stirring and flows
out of the tank at the same rate.

e
a. How much salt is in the tank after 10 minutes?

Us
b. How long will it take for the salt content to reach 150 lbs?
Solution.
𝑙𝑏

l
𝐶' = 2

𝐶) = 𝑆
𝑔𝑎𝑙
<=
cia
(since the volume is kept constant)
er
>?? @A<
𝑔𝑎𝑙
𝑅' = 5
m
𝑚𝑖𝑛
𝑔𝑎𝑙
om

𝑅) = 5
𝑚𝑖𝑛
𝑑S 𝑔𝑎𝑙 𝑙𝑏 𝑔𝑎𝑙 𝑆𝑙𝑏
= 5 2 − (5 )( )
C

𝑑𝑡 𝑚𝑖𝑛 𝑔𝑎𝑙 𝑚𝑖𝑛 100𝑔𝑎𝑙


𝑑S 5𝑆
= 10 −
or

𝑑𝑡 100
tF

Solve by linear equation of order.


𝑑S 𝑆
+ = 10
No

𝑑𝑡 20
O
The integrating factor is 𝑒 PQR .
Thus:
> >
𝑆𝑒 S?# = 10𝑒 S?# 𝑑𝑡
Solving for S:
T#
𝑆 = 200 + 𝐶𝑒 S?
When t=0, there were 100 lb of salt, S=100;
T(?)
100 = 200 + 𝐶𝑒 S?

𝐶 = −100
Z𝒕
𝑺 = 𝟐𝟎𝟎 − 𝟏𝟎𝟎𝒆𝟐𝟎 Working Formula
131

a. after 10 minutes, b. When S= 150 lbs,


T>? T>?
𝑆 = 200 − 100𝑒 S? 𝑆 = 200 − 100𝑒 S?
T>?
𝑺 = 𝟏𝟑𝟗. 𝟑𝟒 𝒍𝒃𝒔
150 = 200 − 100𝑒 S?
T>?
100𝑒 S? = 50
𝑡
− = ln 0.5
20
𝒕 = 𝟏𝟑. 𝟖𝟔 𝒎𝒊𝒏𝒖𝒕𝒆𝒔

Example 23.
A tank contains initially 100 gal of brine holding 150 lb of dissolved salt in

e
solution. Salt water containing 1 lb of salt per gallon enters the tank at the rate of 2

Us
gals/min, and the brine flows out at the same rate. If the mixture is kept uniform by
stirring, find the amount of salt in the tank at the end of 1 hr.
Solution.

l
𝐶' = 1
𝑙𝑏
𝑔𝑎𝑙 cia
er
<=
𝐶) = 𝑆 (since the volume is kept constant)
>?? @A<
m
𝑔𝑎𝑙
𝑅' = 2
𝑚𝑖𝑛
om

𝑔𝑎𝑙
𝑅) = 2
𝑚𝑖𝑛
C

𝑑S 𝑔𝑎𝑙 𝑙𝑏 𝑔𝑎𝑙 𝑆𝑙𝑏


= 2 1 − (2 )( )
𝑑𝑡 𝑚𝑖𝑛 𝑔𝑎𝑙 𝑚𝑖𝑛 100𝑔𝑎𝑙
or

𝑑S 2𝑆
=2−
𝑑𝑡 100
tF

𝑑S 2𝑆
=−
𝑑𝑡 50
No

Solve by linear equation of order.


𝑑S 𝑆
+ =2
𝑑𝑡 50
O
The integrating factor is 𝑒 iQR .
Thus:
> >
𝑆𝑒 j?# = 2𝑒 j?# 𝑑𝑡

Solving for S:
T#
𝑆 = 100 + 𝐶𝑒 j?
132

When t=0, there were 150 lb of salt, S=150;


*(,)
150 = 200 + 𝐶𝑒 .,

𝐶 = 100
4𝒕
𝑺 = 𝟏𝟎𝟎 + 𝟓𝟎𝒆𝟓𝟎 Working Formula

Thus, after 60 minutes,


*7,
𝑆 = 100 + 50𝑒 .,
𝑺 = 𝟏𝟏𝟓. 𝟏 𝒍𝒃𝒔

Example 24.
A large tank holds 300 gallons of brine solution with 40 lbs of salt. A

e
concentration of 2lbs/gal is pumped in a rate of 4 gals/min. The concentration leaving

Us
the tank is pumped out at a rate of 3 gals/min. How much salt is in the tank after 12
minutes?
Solution.

l
𝐶< = 2
𝑙𝑏𝑠
𝑔𝑎𝑙 cia
er
C HIJ
𝐶B = ( 𝑥 ) (since the volume is kept constant)
D,,EF KLH
m
𝑔𝑎𝑙
𝑅< = 4
om

𝑚𝑖𝑛
𝑔𝑎𝑙
𝑅B = 3
𝑚𝑖𝑛
C

𝑑S 𝑔𝑎𝑙 𝑙𝑏𝑠 𝑆 𝑙𝑏𝑠 𝑔𝑎𝑙


= 4 2 −( 𝑥 )(3 )
𝑑𝑡 𝑚𝑖𝑛 𝑔𝑎𝑙 300 + 𝑡 𝑔𝑎𝑙 𝑚𝑖𝑛
or

𝑑S 𝑙𝑏𝑠 3
=8 − 𝑆
tF

𝑑𝑡 𝑚𝑖𝑛 300 + 𝑡

Solve by linear equation of order.


No

𝑑S 3
+ 𝑆=8
𝑑𝑡 300 + 𝑡
The integrating factor is (300 + 𝑡)D .
Thus:
𝑑S 3
(300 + 𝑡)D + (300 + 𝑡)D 𝑆 = 8(300 + 𝑡)D
𝑑𝑡 300 + 𝑡

d[(300 + 𝑡)D 𝑆] = 8(300 + 𝑡)D 𝑑𝑡


Solving for S:
𝑆 = 600 + 2𝑡 + 𝐶(300 + 𝑡)*D
133

When t=0, there were 40 lb of salt, S=40;


40 = 600 + 2 0 + 𝐶(300 + 0)*D
𝐶 = −1.512 𝑥 10\,
𝑆 = 600 + 2𝑡 − (1.512 𝑥 10\, )(300 + 𝑡)*D Working Formula

Thus, after 12 minutes,


𝑆 = 600 + 2 12 − 1.512 𝑥 10\, 300 + 12 *D

𝑺 = 𝟏𝟐𝟔. 𝟏𝟔 𝒍𝒃𝒔

Activity 6
Directions: Solve the following problems completely and give two (2) points for every
item. Your total score is 10 if you will get a perfect score. Put one (1) point if you will

e
finish half of the items and zero (0) if no solution at all. Answers are written on the

Us
left side corner of the questionnaire.

1. A tank initially holds 100 gal of a brine solution containing 20 lb of salt. At t=0,

l
cia
fresh water is poured into the tank at the rate of 5 gal/in, while the well-stirred
mixture leaves at the same rate. Find the amount of salt in the tank at any time t.
4𝒕
er
Ans. 𝑺 = 𝟐𝟎𝒆𝟐𝟎
2. A tank initially holds 100 gal of a brine solution containing 1 lb of salt. At t=0,
m

another brine solution containing 1 lb of salt per gallon is poured into the tank at the
om

rate of 3 gal/min, while the well-stirred mixture leaves at the same rate. Find the
amount of salt in the tank at any time t.
Ans. 𝑺 = −𝟗𝟗𝒆*𝟎.𝟎𝟑𝒕 + 𝟏𝟎𝟎
C
or

3. Find the time t at which the mixture described in Problem 2 contains 2 lb of salt.
Ans. 𝒕 = 𝟎. 𝟑𝟑𝟖 𝒎𝒊𝒏
tF

4. A 50-gal tank initially contains 10 gals of fresh ater. At t=0, a brine solution
containing 1 lb of salt per gallon is poured into the tank at the rate of 4 gals/min,
No

while the well-stirred mixture leaves at the rate of 2 gal/min. Find the amount of time
required for overflow to occur.
Ans. 𝒕 = 𝟐𝟎 𝒎𝒊𝒏

5. A tank contains 100 gal of brine solution made by dissolving 60 lb of salt in water.
Salt water containing 1 lb of salt per gallon runs in at the rate of 2 gal/min, and the
mixture, kept uniform by stirring, runs out at the rate of 3 gal/min. Find the amount
of salt in the tank at the end of 1 hour.
Ans. 𝑺 = 𝟑𝟕. 𝟒𝟒 𝒍𝒃𝒔

Progress Indicators: (Activity 1)


134

5.7 Velocity of Escape from the Earth


Derivation:

V g
𝑎= 1
hi

a 𝑑𝑉 𝑘 2
𝑎= = m
𝑑𝑡 𝑟
r

R When 𝑎 = −𝑔, substitute in 2;

𝑘
−𝑔 =
𝑅m

e
𝑘 = −𝑔𝑅m 3

Us
Figure 7

l
From 1, 𝑘 = 𝑎𝑟 m , substitute in 3,

𝑎𝑟 m = −𝑔𝑅m cia
er
*Kn i
𝑎= (4)
m
hi
From 2,
om

𝑑𝑉 𝑑𝑟
𝑎= ( )
𝑑𝑡 𝑑𝑟
op oh
𝑎= ( ) (5)
C

oh oF
But,
or

oh
𝑉= (6)
oF
tF

Substitute 6 in 5
op
𝑎= (𝑉) (7)
oh
No

Substitute 4 in 7
−𝑔𝑅m 𝑑𝑉
= (𝑉)
𝑟m 𝑑𝑟
−𝑔𝑅m
𝑑𝑟 = 𝑉𝑑𝑣
𝑟m
−𝑔𝑅m (𝑟 *m )𝑑𝑟 = 𝑉𝑑𝑣
h p
−𝑔𝑅m 𝑟 *m 𝑑𝑟 = 𝑉𝑑𝑣
n pr

m *\ m
𝑉 m 𝑉\ m
*\
𝑔𝑅 𝑟 − 𝑔𝑅 𝑅 = −
2 2
m m m m
𝑔𝑅 𝑔𝑅 𝑉 − 𝑉\
− =
𝑟 𝑅 2
135

𝑔𝑅m
2 − 𝑔𝑅 = 𝑉 m − 𝑉\ m
𝑟
𝑔𝑅m
𝑉 m = 𝑉\ m + 2 − 𝑔𝑅
𝑟
𝑔𝑅m
m
𝑉 =2 + 𝑉\ m − 2𝑔𝑅
𝑟
If 𝑉\ m − 2𝑔𝑅 < 0 there is a value of r to make V=0. For this condition, the body
ill stop and reverse its direction back to earth, so that it will not return V>0 if 𝑉\ m −
2𝑔𝑅 ≥ 0.
𝑉\ m = 2𝑔𝑅
𝑽𝟏 = 𝟐𝒈𝑹

e
Example 25.

Us
The radius of the moon is roughly 1080 miles. The acceleration of gravity at
the surface of the moon is about 0.165g, where g is the acceleration of gravity at the

l
surface of the earth. Determine the velocity of escape for the moon.
Solution.
cia
er
𝑚 1609.344 𝑚
𝑉\ = 2(0.165)(9.8 )(1080 𝑚𝑖𝑙𝑒𝑠)( )
𝑠m 𝑚𝑖𝑙𝑒𝑠
m

𝒎
𝑽𝟏 = 𝟐𝟑𝟕𝟎. 𝟖𝟔
om

Example 26.
C

A spring of negligible weight hangs vertically. A mass of m slugs is attached to


or

the other end. If the mass is moving with velocity 𝑣, ft/s when the spring is
unstretched, find the velocity v as a function of the stretch x in feet.
tF

Solution.
According to Hooke’s Law, the spring force (the force opposing the stretch) is
No

proportional to the stretch. Thus, we have,


𝑑𝑣 1
𝑚 = 𝑚𝑔 − 𝑘𝑥
𝑑𝑡
Rewriting equation 1, we obtained,
oy oz oy oz
𝑚 = 𝑚𝑣 = 𝑚𝑔 − 𝑘𝑥, 2 since =𝑣
oz oF oz oF

Integrating equation 2 gives,


𝑚𝑣 m = 2𝑚𝑔𝑥 − 𝑘𝑥 m + 𝐶 3

When 𝑣 = 𝑣, , 𝑥 = 0 so that 𝐶 = 𝑚𝑣, m and,


𝒎𝒗𝟐 = 𝟐𝒎𝒈𝒙 − 𝒌𝒙𝟐 + 𝒎𝒗𝟎 𝟐
136

Activity 7
Directions: Solve the following problems completely and give two (2) points for every
item. Your total score is 10 if you will get a perfect score. Put one (1) point if you will
finish half of the items and zero (0) if no solution at all. Answers are written on the
left side corner of the questionnaire.

1. A parachutist is falling with speed 176 ft/s when his parachute opens. If the air
~y i
resistance is 𝑙𝑏 , where W is the total weight of the man and parachute, find his
m.7
speed as a function of the time t after the parachute opens.
𝟔E𝟓𝒆4𝟒𝒕
Ans. 𝒗 = 𝟏𝟔
𝟔*𝟓𝒆4𝟒𝒕
2. A mass is being pulled across ice on a sled, the total weight including the sled being
80 lb. the resistance offered by the ice to the runners is negligible, and the air offers

e
Us
a resistance in pounds equal to five times the velocity (ft/s) of the sled. Find the
constant force in pounds that must be exerted on the sled to give it a terminal velocity
of 10 mi/h, and the velocity and distance traveled at the end of 48 s.

l
𝟐𝟐𝟎 𝟒𝟒

cia
Ans. 𝑭 = 𝒍𝒃 , 𝒗 = 𝒇𝒕/𝒔 & 𝒔 = 𝟔𝟗𝟕 𝒇𝒕
𝟑 𝟑
3. A boat is being towed at the rate of 12 mi/h. At the instant (t=0) that towing line is
er
cast off, a man in the boat begins to row in the direction of motion exerting a force of
20 lb. if the combined weight of the man and boat is 480 lb and the resistance in
m
pounds is equal to 1.75v, where v is measured in feet/second, find the speed of the
boat after ½ min.
om

𝒇𝒕
Ans. 𝒗 = 𝟏𝟏. 𝟔
𝒔
C

4. The gravitational pull on a mass m at a distance s feet from the center of the earth
is proportional to m and inversely proportional to 𝑠 m . (a) Find the velocity attained
or

by the mass in falling from rest at a distance 5R from the center of the earth’s surface,
where R=4000 mi is the radius of the earth. (b) what velocity would correspond to a
tF

fall from an infinite distance, that is, with what velocity must the mass be propelled
vertically upward to escape the earth’s gravitational pull? (All other forces, including
No

friction, are to be neglected.)


𝒇𝒕 𝒎𝒊 𝒇𝒕 𝒎𝒊
Ans. (a). 𝒗 = 𝟐𝟓𝟔𝟎 𝟏𝟔𝟓 𝒐𝒓 𝟔 & (b). 𝒗 = 𝟔𝟒𝟎𝟎 𝟑𝟑 𝒐𝒓 𝟕
𝒔 𝒔 𝒔 𝒔
5. A body of mass m falls from rest in a medium for which the resistance (in pounds)
is proportional to the velocity (in feet per second). If the specific gravity of the medium
is one-fourth that of the body and if the terminal velocity is 24 ft/s, find the velocity
at the end of 3 s, and the distance traveled in 3 s.
𝒇𝒕
Ans. 𝒗 = 𝟐𝟐. 𝟖 & 𝒔 = 𝟒𝟗. 𝟐 𝒇𝒕
𝒔
Progress Indicators: (See Activity 1)
137

5.8 Newton’s Second Law

Newton’s second law states that force on a body is proportional to the product of its
mass and acceleration, symbolized as
𝐹 = 𝑚𝑎 (1)
For falling bodies,
𝑛𝑒𝑡 𝐹 = 𝑊 − 𝑅, (2)
where: W is the weight of the body
𝑅, is the resisting force to the motion
substituting (2) to (1), we get
𝑊 − 𝑅, = 𝑚𝑎
converting to differential form,

e
-.

Us
𝑊 − 𝑅, = 𝑚 (3)
-/
but 𝑊 = 𝑚𝑔 ,
2 -.
thus, (4)

l
𝑊 − 𝑅, = ( )

cia
3 -/

Example 27.
er
The figure shown below shows a weight , 𝑊 newtons sliding down an inclined plane
m
which makes an angle of 𝛽 with the horizontal. Assume that no other force other than
gravity affects the motion of the weight. At the start , 𝑥 = 𝑥8 and initial velocity is
om

𝑣8 . Find 𝑥 at any time 𝑡. (no air resistance and friction between surfaces is negligible)
C

Solution:
Applying Newton’s second law and considering the force parallel to the plane,
or

𝐹: = 𝑊 𝑠𝑖𝑛 𝛽 = 𝑚𝑎
tF

-.
transforming to the derivative form, 𝑚𝑔 𝑠𝑖𝑛 𝛽 = 𝑚
-/
separating the variables 𝑑𝑣 = 𝑔 𝑠𝑖𝑛𝛽 𝑑𝑡
No

integrating, 𝑣 = 𝑔𝑡 sin 𝛽 + 𝑐
substituting given conditions, when 𝑡 = 0, 𝑣 = 𝑣8 , 𝑣8 = 𝑐
-: -:
however v = , thus, = 𝑔𝑡 𝑠𝑖𝑛𝛽 + 𝑣8
-/ -/
separating the variables again, 𝑑𝑥 = ( 𝑔𝑡 sin 𝛽 + 𝑣8 ) 𝑑𝑡
D
integrating 𝑥= 𝑔𝑡 E sin 𝛽 + 𝑣8 𝑡 + 𝑐D
E
substituting conditions, when t = 0, x = 𝑥8 𝑥8 = 𝑐D

D
thus 𝑥= 𝑔𝑡 E sin 𝛽 + 𝑣8 t + 𝑥8
E
138

5.9 Flow of Water Through an Orifice

Example 28.
A hemisphere having a radius of 1 ft and base up was initially filled with water.
Water runs out through an orifice , 1 inch in diameter at the bottom. How long will it
take for the water to run out?

Solution:
The differential equation relating to the flow of liquids through an orifice is
-F
= − 0.6 𝐵8 2𝑔ℎ where V = volume
-/
𝐵8 = orifice cross-sectional area
h = water elevation

e
g = acc. due to gravity

Us
t = time
Separating the variables , 𝑑𝑉 = − 0.6 𝐵8 2𝑔ℎ dt

l
cia
Since we have a hemisphere, the differential volume of the water removed from the
tank is
er
𝑑𝑉 = 𝜋𝑥 E 𝑑ℎ but 𝑥 E =1 − ( 1 − ℎ)E (Pythagorean theorem)
𝑥 E = 2h - ℎE
m

Thus, 𝜋 2ℎ − ℎE 𝑑ℎ = −0.6 𝐵8 2𝑔ℎ 𝑑𝑡


om

O
Separating the variables, 𝜋 ( 2 ℎ - ℎP ) 𝑑ℎ = −0.6 𝜋𝑟8 E 2𝑔 dt
C

For convenience we determine the value of the constant 𝑘


8.S
where 𝑘 = 0.6 [ ( )E ]
or

2 (32.2) = 0.00836
DE
tF

D U
( 2ℎE − ℎE )dh = −𝑘 𝑑𝑡
No

O X
E E
Integrating , 2 ( ) ℎP - ℎP = − 𝑘𝑡 + 𝑐
U S
When 𝑡 = 0 , ℎ = 1 , 𝑐 = 14/15
O X
\ E
ℎ − ℎP = − 𝑘𝑡 + 14/15
P
U S
When the tank is empty ℎ = 0 , 0 = − 0. 00836 𝑡 + 14/15
Solving for 𝑡, 𝑡 = 111. 65 𝑠𝑒𝑐𝑜𝑛𝑑𝑠
139

Chapter 6: System of Linear First-Order Dif-


ferential Equations
Chapter Overview
In this chapter, we discuss an introduction to solution of homogeneous linear systems
with constant coefficients using eigenvalues and eigenvectors and matrix exponential.

Learning Objectives
At the end of this chapter, the student is expected to

1. solve homogeneous linear systems with constant coefficients using eigenvalues

e
and eigenvectors;

Us
2. use the matrix exponential function to solve the linear system x = Ax, where A
is a 2 × 2 matrix with constant entries.

l
6.1 System of Differential Equations cia
er
m
We have seen that a single differential equation can serve as a mathematical model for a
single population in an environment. But if there are, say, two interacting and perhaps
om

competing species living in the same environment (for example, rabbits and foxes), then a
model for their populations x(t) and y(t) might be a system of two first-order differential
C

equations such as

dx
or

= g1 (t, x, y)
dt (1)
tF

dy
= g1 (t, x, y).
dt
No

When g1 and g2 are linear in the variables x and y, that is, g1 and g2 have the forms

g1 (x, y) = c1 x + c2 y + f1 (t) and g2 (x, y) = c3 x + c4 y + f2 (t).

where the coefficients ci could depend on t, then (1) is said to be a linear system. A system
of differential equations that is not linear is said to be nonlinear.

A Predatory-Prey Model. Suppose that two different species of animals interact within
the same environment or ecosystem, and suppose further that the first species eats only
vegetation and the second eats only the first species. In other words, one species is a preda-
tor, and the other is a prey. For example, wolves hunt grass-eating caribou, sharks devour
little fish, and the snowy owl pursues an arctic rodent called the lemming. For the sake of
discussion, let us imagine that the predators are foxes and the prey are rabbits.
140

Let x(t) and y(t) denote the fox and rabbit populations, respectively, at time t. If there were
no rabbits, then one might expect that the foxes, lacking an adequate food supply, would
decline in number according to
dx
= −ax, a > 0 (2)
dt
When rabbits are present in the environment, however, it seems reasonable that the number
of encounters or interactions between these two species per unit time is jointly proportional
to their populations x and y, that is, proportional to the product xy. Thus when rabbits are
present, there is a supply of food, so foxes are added to the system at a rate bxy, b > 0??.
Adding this last rate to (2) gives a model for the fox population:
dx
= −ax + bxy (3)

e
dt

Us
On the other hand, if there were no foxes, then the rabbits would, with an added assumption
of unlimited food supply, grow at a rate that is proportional to the number of rabbits present
at time t:

l
dy
dt cia
= dy, d > 0 (4)
er
But when foxes are present, a model for the rabbit population is (4) decreased by cxy, c > 0,
m
that is, decreased by the rate at which the rabbits are eaten during their encounters with
the foxes:
om

dy
= dy − cxy (5)
dt
C

Equations (3) and (5) constitute a system of nonlinear differential equations


or
tF

dx
= −ax + bxy = x(−a + by)
dt (6)
dy
No

= dy − cxy = y(d − cx)


dt
where a, b, c, and d are positive constants. This famous system of equations is known as the
Lotka-Volterra predator-prey model.
d a
Except for two constant solutions, x(t) = 0, y(t) =??0 and x(t) = , y(t) = ??, the nonlin-
c b
ear system (6) cannot be solved in terms of elementary functions. However, we can analyze
such systems quantitatively and qualitatively.

Definition 1: Solution of a System


A solution of a system of differential equation is a set of sufficiently differentiable
functions x = φ1 (t), y = φ2 (t), z = φ3 (t), and so on, that satisfies each equation in
the system on some common interval I.
141

A system of linear differential equations can be solved by Laplace transforms, by matrices


and by systematic elimination. In this section, we will focus only on systems of first-order
equations that have the normal form

dx1
= g1 (t, x1 , x2 , . . . , xn )
dt
dx2
= g2 (t, x1 , x2 , . . . , xn )
dt (7)
.. ..
. .
dxn
= gn (t, x1 , x2 , . . . , xn )
dt
A system such as (7) of n first-order equations is called a first-order system.

e
Linear Systems

Us
When each of the functions g1 , g2 , . . . , gn in (7) is linear in the dependent variables
x1 , x2 , . . . , xn , we get the normal form of a first-order system of linear equations

l
dx1
dt cia
= a11 (t)x1 + a12 (t)x2 + · · · + a1n (t)xn + f1 (t)
er
dx2
= a21 (t)x1 + a22 (t)x2 + · · · + a2n (t)xn + f2 (t)
m
dt (8)
.. ..
. .
om

dxn
= an1 (t)x1 + an2 (t)x2 + · · · + ann (t)xn + fn (t)
dt
C

We refer to this system of form (7) as a linear system. We assume that the co-
or

efficients aij and the functions fi are continuous on a common interval I. When
fi (t) = 0, i = 1, 2, 3, . . . , n, the linear system is said to be homogeneous, otherwise,
tF

it is nonhomogeneous.
No

A linear system can be written in matrix form. If X, A(t), and F(t) denote the respective
matrices

     
x1 (t) a11 (t) a12 (t) · · · a1n (t) f1 (t)
 x2 (t)   a21 (t) a22 (t) · · · a2n (t)   f2 (t) 
     
X =  ..  , A =  .. ..  , F(t) =  .. 
 .   . .   . 
xn (t) an1 (t) an2 (t) · · · ann (t) fn (t)
then the system of linear first-order differential equations (8) can be written as
      
x1 a11 (t) a12 (t) · · · a1n (t) x1 f1 (t)
  
d  2   21 (t)
x a a22 (t) · · · a2n (t)     
  x2   f2 (t) 
 . = . ..   ..  +  .. 
dt  ..   .. .  .   . 
xn an1 (t) an2 (t) · · · ann (t) xn fn (t)
142

or simply

X’ = AX + F. (9)

If the system is homogeneous, its matrix form is

X’ = AX. (10)

Example 1.
 
x
1. If X = , then the matrix form of the homogeneous system
y

dx
= 3x + 4y  
dt 3 4

e
is X’ = X.

Us
dy 5 −7
= 5x − 7y
dt
 

l
x

z cia
2. If X = y , then the matrix form of the nonhomogeneous system
er
dx
m
= 6x + y + z + t
dt    
om

6 1 1 t
dy
= 8x + 7y − z + 10t is X’ = 8 7 −1 X + 10t
dt 2 9 −1 6t
C

dz
= 2x + 9y − z + 6t
dt
or
tF

Definition 2: Solution Vector


A solution vector on an interval I is any column matrix
No

 
x1 (t)
 x2 (t) 
 
X =  .. 
 . 
xn (t)

whose entries are differentiable functions satisfying the system (9) on the interval.

A solution vector of (9) is equivalent to n scalar equations x1 = φ1 (t), x2 = φ2 (t), . . . , xn =


φn (t) and can be interpreted geometrically as a set of parametric equations of a space curve.
In the important case n = 2, the equations x1 = φ1 (t), x2 = φ2 (t) represent a curve in the
x1 x2 -plane. It is common practice to call a curve in the plane a trajectory and to call the
x1 x2 -plane the phase plane.
143

Example 2. Verify that on the interval (−∞, ∞)


   −2t     6t 
1 −2t e 3 6t 3e
X1 = e = and X2 = e =
−1 −e−2t 5 5e6t
 
0 1 3
are solutions of X = X.
5 3
   6t 
−2e−2t 18e
Solution: From X01 = −2t
0
and X2 = we see that
2e 30e6t

      
1 3 e−2t e−2t − 3e−2t −2e−2t
AX1 = = = = X01

e
5 3 −e−2t 5e−2t − 3e−2t 2e−2t

Us
   6t   6t   6t 
1 3 3e 3e + 15e6t 18e
and AX2 = 6t = 6t 6t = = X02 .
5 3 5e 15e + 15e 30e6t

l
cia
Much of the theory of systems of n linear first-order differential equations is similar to that
er
of linear nth-order differential equations.
Initial-Value Problem
m

Let t0 denote a point on an interval I and


om

   
x1 (t0 ) γ1
 x2 (t0 )   γ2 
C

   
X(t0 ) =  ..  , and X0 =  .. 
 .  .
or

xn (t0 ) γn
tF

where γi , i = 1, 2, . . . , n are given constants. Then the problem

Solve : X’ = A(t)X + F(t)


No

(11)
Subject to : X’(t0 ) = X0

is an initial-value problem on the interval.

Theorem 1: Existence of a Unique Solution

Let the entries of the matrices A(t) and F(t) be functions continuous on a common
interval I that contains the point t0 . Then there exists a unique solution of the initial-
value problem (11) on the interval.

In the next several definitions and theorems we are concerned only with homogeneous sys-
tems. Without stating it, we shall always assume that the aij and the fi in (8) are continuous
functions of t on some common interval I.
144

Theorem 2
Let X1 , X2 , . . . , Xk be a set of solution vectors of the homogeneous system (10) on
an interval I. Then the linear combination

X = c1 X 1 + c2 X 2 + · · · + ck X k ,

where the ci , i = 1, 2, . . . , k are arbitrary constants, is also a solution on the interval.

Definition 3: Linear Dependence/Independence

Let X1 , X2 , . . . , Xk be a set of solution vectors of the homogeneous system (10) on an


interval I. We say that the set is linearly dependent on the interval if there exist
constants c1 , c2 , ..., ck , not all zero, such that

e
Us
c1 X 1 + c2 X 2 + · · · + ck X k = 0

for every t in the interval. If the set of vectors is not linearly dependent on the interval,

l
cia
it is said to be linearly independent.

In the case where k = 2, two solution vectors X1 and X2 are linearly dependent if one is
er
a constant multiple of the other, and conversely. For cases where k > 2, a set of solution
m
vectors is linearly dependent if we can express at least one solution vector as a linear com-
bination of the remaining vectors.
om

Theorem 3: Criterion for Linearly Independent Solutions


C

Let     
x11 x12 x1n
or

 x21   x22   x2n 


     
X1 =  ..  , X2 =  ..  , . . . , Xn =  .. 
tF

 .   .   . 
xn1 xn2 xnn
No

be n solution vectors of the homogeneous system (10) on an interval I. Then the set
of solution vectors is linearly independent on I if and only if the Wronskian

x11 x12 · · · x1n


 x21 x22 · · · x2n
W X 1 , X2 , . . . , Xn = .. .. 6= 0 (12)
. .
xn1 xn2 · · · xnn

for every t in the interval.


145
   −2t 
1 −2t e
Example 3. From Example 2, we have seen that X1 = e = and
   6t   −1 −e−2t
3 6t 3e 0 1 3
X2 = e = 6t are solutions of the system X = X. Clearly, X1 and X2
5 5e 5 3
are linearly independent on the interval (−∞, ∞) since neither vector is a constant multiple
of the other. Also, we have
 e−2t 3e6t
W X1 , X2 = = 8e4t 6= 0
−e−2t 5e6t

for all real values of t.

Definition 4: Fundamental Set of Solutions

e
Any set X1 , X2 , . . . , Xn of n linearly independent solution vectors of the homogeneous

Us
system (10) on an interval I is said to be a fundamental set of solutions on the
interval.

l
cia
Theorem 4: Existence of a Fundamental Solution
There exists a fundamental set of solutions for the homogeneous system (10) on an
er
interval I.
m
Theorem 5: General Solution - Homogeneous Systems
om

Let X1 , X2 , . . . , Xn be a fundamental set of solutions of the homogeneous system (10)


on an interval I. Then the general solution of the system on the interval is
C

X = c 1 X1 + c 2 X2 + · · · + c n X n ,
or

where the ci , i = 1, 2, . . . , n are arbitrary constants.


tF

   
1 −2t 3 6t
Example 4. From Example 3, we have shown that X1 = e and X2 = e
No

  −1 5
0 1 3
are linearly independent solutions of X = X on (−∞, ∞). Hence X1 and X2 form
5 3
a fundamental set of solutions on the interval. The general solution of the system on the
interval is then    
1 −2t 3 6t
X = c 1 X1 + c 2 X2 = c 1 e + c2 e .
−1 5
146

Theorem 6: General Solution - Nonhomogeneous Systems

Let Xp be a given solution of the nonhomogeneous system (9) on an interval I, and


let
Xc = c 1 X1 + c 2 X2 + · · · + c n X n ,
denote the general solution on the same interval of the associated homogeneous system
(10). Then the general solution of the nonhomogeneous system on the interval is

X = Xc + X p .

The general solution Xc of the associated homogeneous system (10) is called the com-
plementary function of the nonhomogeneous system (9).

e
Us
6.2 Homogeneous Linear Systems with Constant Coef-
ficients

l
cia
We have seen from Example  4 of the previous lesson thatthegeneral solution
  of the
1 3 1 3 6t
homogeneous system X0 = X is X = c1 X1 + c2 X2 = c1 e−2t + c2 e . Since
er
5 3   −1 5
k 1 λi t
m
both solution vectors have the form Xi = e , i = 1, 2, where k1 and k2 are constants,
k2
we are prompted to ask whether we can always find a solution of the form
om

 
k1
 k2 
C

 
X =  ..  eλt = Keλt (13)
.
or

kn
tF

for the general homogeneous linear first-order system

X0 = AX,
No

(14)

where A is an n × n matrix of constants.

If (13) is to be a solution vector of the homogeneous linear system (14), then X0 = Kλeλt
so that (14) becomes Kλeλt = AKeλt . After dividing both sides by eλt and rearranging, we
have AK = λK or AK − λK = 0. Since K=IK, this last equation is the same as

(A − λI)K = 0. (15)
147

The matrix equation (15) is equivalent to the simultaneous algebraic equations



a11 − λ k1 + a12 k2  + ··· + a1n kn = 0
a21 k1 + a22 − λ k2 + · · · + a2n kn = 0
.. ..
.  .
an1 k1 + an2 k2 + · · · + ann − λ kn = 0
Thus to find a nontrivial solution X of (14), we must first find a nontrivial solution of the
foregoing system; in other words, we must find a nontrivial vector K that satisfies (15). But
in order for (15) to have solutions other than the obvious solution k1 = k2 = · · · = kn = 0,
we must have
det(A − λI) = 0
This polynomial equation in λ is called the characteristic equation of the matrix A; its so-

e
lutions are the eigenvalues of A. A solution K 6= 0 of (15) corresponding to an eigenvalue λ

Us
is called an eigenvector of A. A solution of the homogeneous system (14) is then X = Keλt .

Similar with the roots of the auxiliary equation in the discussion of higher-order linear ordi-

l
cia
nary differential equations, the eigenvalues of A that can be determined are either (i) real
and distinct; (ii) repeated; or (iii) complex.
er
CASE 1: Distinct Real Eigenvalues
m

Theorem 7
om

Let λ1 , λ2 , . . . , λn be n distinct real eigenvalues of the coefficient matrix A of the


homogeneous system (14), and let K1 , K2 , . . . , Kn be the corresponding eigenvectors.
C

Then the general solution of (14) on the interval (−∞, ∞) is given by


or

X = c1 K1 eλ1 t + c2 K2 eλ2 t + · · · + cn Kn eλn t .


tF

Example 5. Solve:
dx
No

= 2x + 3y
dt
dy
= 2x + y
dt
Solution: We first determine the eigenvalues and eigenvectors of the matrix of coefficients.
2−λ 3
det(A − λI) = = (2 − λ)(1 − λ) − 6 = λ2 − 3λ − 4 = 0
2 1−λ
(λ + 1)(λ − 4) = 0 ←− characteristic equation
λ1 = −1 and λ2 = 4 ←− eigenvalues
For λ1 = −1, (A − λI)K = 0 is equivalent to
3k1 + 3k2 = 0
2k1 + 2k2 = 0.
148
 
1
Thus k1 = −k2 . When k2 = −1, the related eigenvector is K1 = .
−1
For λ2 = 4, (A − λI)K = 0 is equivalent to
−2k1 + 3k2 = 0
2k1 − 3k2 = 0.
 
3 3
Thus k1 = k2 . When k2 = 2, the corresponding eigenvector is K2 = .
2 2
Since the matrix coefficients of A is a 2 × 2 matrix and since we have found
  two linearly
1 3 4t
independent solutions of the given system, X1 = e−t and X2 = e , we then
−1 2
conclude that the general solution of the given system is
   

e
1 −t 3 4t

Us
X = c 1 X1 + c 2 X2 = c 1 e + c2 e .
−1 2
Similarly, we can write the above general solution in a more familiar statement,

l
cia
x(t) = c1 e−t + 3c2 e4t , y(t) = −c1 e−t + 2c2 e4t
which can be interpreted as parametric equations of a curve in the xy− plane or phase
er
plane. The curve is called a trajectory. A collection of representative trajectories in the
m
phase plane is said to be a phase portrait of a given linear system.
CASE 2: Repeated Eigenvalues
om

Not all of the eigenvalues λ1 , λ2 , . . . , λn of an n × n ??matrix A need be distinct, that is,


C

some of the eigenvalues may be repeated.


or

If m is a positive integer and (λ − λ1 )m is a factor of the characteristic equation while


(λ − λ1 )m+1 is not a factor, then λ1 is said to be an eigenvalue of multiplicity m.
tF

• For some n × n matrices A, it may be possible to find m linearly independent eigen-


vectors K1 , K2 , . . . , Km corresponding to an eigenvalue λ1 of multiplicity m ≤ n. In
No

this case the general solution of the system contains the linear combination
c1 K1 eλ1 t + c2 K2 eλ1 t + · · · + cm Km eλ1 t .

• If there is only one eigenvector corresponding to the eigenvalue λ1 of multiplicity m,


then m linearly independent solutions of the form
X1 = K11 eλ1 t
X2 = K21 teλ1 t + K22 eλ1 t
..
.
tm−1 λ1 t tm−2 λ1 t
Xm = Km1 e + Km2 e + · · · + Kmm eλ1 t
(m − 1)! (m − 2)!
where Kij are column vectors, can always be found.
149

Suppose that λ1 is an eigenvalue of multiplicity two and that there is only one eigenvector
associated with this value. A second solution can be found of the form
X2 = Kteλ1 t + Peλ1 t , (16)
where    
k1 p1
 k2   p2 
   
K =  ..  and P =  .. 
. .
kn pn
To see this, we substitute (16) into the system X’ = textbf AX and simplify.
(AK − λ1 K)teλ1 t + (AP − λ1 P − K)teλ1 t = 0

e
Since this last equation is to hold for all values of t, we must have

Us
(A − λ1 I)K = 0 (17)

l
and

cia
(A − λ1 I)P = K (18)
er
Equation (17) simply states that K must be an eigenvector of A associated with λ1 . By
solving (17), we find one solution X1 = Keλ1 t . To find the second solution X2 we only need
m

to solve the additional system (18) for the vector P.


om

 
0 3 −18
Example 6. Solve: X = X
2 −9
C

Solution: Determine the eigenvalues and eigenvectors of the matrix coefficients.


3 − λ −18
or

det(A − λI) = = (3 − λ)(−9 − λ) − (−36) = λ2 + 6λ + 9 = 0


2 −9 − λ
tF

(λ + 3)2 = 0 ←− characteristic equation


λ1 = λ2 = −3 ←− eigenvalue of multiplicity 2
No

For λ = −3, (A − λI)K = 0 is equivalent to


6k1 − 18k2 = 0
2k1 − 6k2 = 0.
 
3
Thus k1 = 3k2 . When k2 = 1, the related eigenvector is K1 = and that one solution is
  1
3 −3t
X1 = e .
1    
3 p1
We identify K = and P = , we find from (18) that we must now solve
1 p2

(A+3I)P=K or 6p1 − 18p2 = 3


2p1 − 6p2 = 1.
150

2p1 − 1
The above system obviously have an infinite number of solutions , that is p2 = , and
6
so, we have an infinite number of choices for p1 and p2 . For an instance, by choosing p1= 1,
1 1 1
we will have p2 = . For simplicity, we will choose p1 = so that p2 = 0. Hence, P = 2 .
6 2 0
Thus, from (16), we have   1
3
X2 = −3t
te + 2 e−3t
1 0
and the general solution of the given is
    1 
3 −3t 3 −3t 2 −3t
X = c1 e + c2 te + e .
1 1 0

CASE 3: Complex Eigenvalues

e
Us
If λ1 = α + βi and λ2 = α − βi, β > 0, i2 = −1 are complex eigenvalues of the coefficient
matrixA, we can then certainly expect their corresponding eigenvectors to also have complex

l
entries. (Take note that if a characteristic equation has real coefficients, complex eigenvalues
always appear in conjugate pairs.)
cia
Theorem 8: Solutions Corresponding to a Complex Eigenvalue
er
Let A be the coefficient matrix having real entries of the homogeneous system (14), and
m

let K1 be an eigenvector corresponding to the complex eigenvalue λ1 = α + βi, αandβ


om

real. Then
K1 eλ1 t and K1 eλ1 t
C

are solutions of (14).


or

Example 7. Solve:
tF

dx
= 6x − y
dt
No

dy
= 5x + 4y
dt
Solution: Determine the eigenvalues and eigenvectors of the matrix coefficients.

6 − λ −1
det(A − λI) = = (6 − λ)(4 − λ) − (−5) = λ2 − 10λ + 29 = 0
5 4−λ
λ2 − 10λ + 29 = 0 ←− characteristic equation
λ1 = 5 + 2i and λ2 = 5 − 2i ←− eigenvalues, by quadratic formula

For λ1 = 5 + 2i, (A − λI)K = 0 is equivalent to

(1 − 2i)k1 − k2 = 0
5k1 − (1 + 2i)k2 = 0
151

Since k2 = (1−2i)k1 , we can choose k1 = 1 which will give us k2 = 1−2i with the eigenvector
and corresponding solution vector:
   
1 1
K1 = , X1 = e(5+2i)t .
1 − 2i 1 − 2i

Similarly, for λ2 = 5 − 2i, we find


   
1 1
K2 = , X2 = e(5−2i)t .
1 + 2i 1 + 2i

By Wronskian, we can verify that these solution vectors are linearly independent and the
general solution of the given system is
   

e
1 (5+2i)t 1
X = c1 e + c2 e(5−2i)t .

Us
1 − 2i 1 + 2i

It is desirable and relatively easy to rewrite a solution such as in the previous example in

l
terms of real functions. To this end we first use Euler’s formula to write

cia
e(5+2i)t = e5t e2ti = e5t (cos 2t + i sin 2t)
er
e(5−2i)t = e5t e−2ti = e5t (cos 2t − i sin 2t).
m

After we multiply complex numbers, collect terms and replace c1 + c2 by C1 and c1 − c2 i
om

by C2 , we now have

X = C 1 X1 + C 2 X2 , (19)
C

where
or

         
1 0 5t 0 1
sin 2t e5t
tF

X1 = cos 2t − sin 2t e and X2 = cos 2t +


1 −2 −2 1

Take note that the two vectors X1 and X2 in (19) are linearly independent real solutions
No

of the original system. Consequently, we are justified in ignoring the relationship between
C1 , C2 and c1 , c2 , and we can regard C1 and C2 as completely arbitrary and real. In other
words, the linear combination (19) is an alternative solution of the original system.
152

Theorem 9: Real Solutions Corresponding to a Complex Eigenvalue


Let λ1 = α + iβ be a complex eigenvalue. Of the coefficient matrix A on the homoge-
neous system (14), and let B1 and B2 denote the column vectors defined as
1  i 
B1 = K1 + K2 and B2 = − K1 + K2 (20)
2 2
Then
 
X1 = B1 cos βt − B2 sin βt eαt
  (21)
X2 = B2 cos βt + B1 sin βt eαt

are linearly independent solutions of (14) on (−∞, ∞).

e
Us
The matrices B1 and B2 in (21) are often denoted by
 
B1 = Re K1 and B2 = Im K1 (22)

l
cia
since this vectors are, respectively, the real and imaginary parts of the eigenvector K1 .
er
Example 8. Solve the initial-value problem
   
m
0 3 −13 3
X = X, X(0) =
−10
om

5 1

Solution: Determine the eigenvalues and eigenvectors of the matrix coefficients.


C

3 − λ −13
det(A − λI) = = (3 − λ)(1 − λ) − (−65) = λ2 − 4λ + 68 = 0
or

5 1−λ
λ2 − 4λ + 68 = 0 ←− characteristic equation
tF

λ1 = 2 + 8i and λ2 = 2 − 8i ←− eigenvalues, by quadratic formula


No

For λ1 = 2 + 8i, (A − λI)K = 0 is equivalent to

(1 − 8i)k1 − 13k2 = 0
5k1 − (1 + 8i)k2 = 0

(1 + 8i)k1
Since k1 = , we can choose k2 = 5 which will give us k1 = 1 + 8i with the
5
eigenvector:      
1 + 8i 1 8
K1 = = +i
5 5 0
Now, from (22), we form
   
 1  8
B1 = Re K1 = and B2 = Im K1 =
5 0
153

Since α = 2 (real part of the complex eigenvalues), it follows from (21) that the general
solution of the given system is
         
1 8 2t 8 1
X = c1 cos 8t − sin 8t e + c2 cos 8t + sin 8t e2t
5 0 0 5
   
cos 8t − 8 sin 8t 2t 8 cos 8t + sin 8t 2t
X = c1 e + c2 e
5 cos 8t 5 sin 8t
Equivalently, we can have,
x(t) = c1 (cos 8t − 8 sin 8t)e2t + c2 (8 cos 8t + sin 8t)e2t
y(t) = c1 (5 cos 8t)e2t + c2 (5 sin 8t)e2t
 
3
Applying the initial condition X(0) = or equivalently, x(0) = 3 and y(0) = −10, we
−10

e
will have the algebraic system,

Us
3 = c1 + 8c2 5
=⇒ c1 = −2 and c2 =
−10 = 5c1 8
   

l
cia
cos 8t − 8 sin 8t 2t 5 8 cos 8t + sin 8t 2t
Thus, the solution to the IVP is X = −2 e + e
5 cos 8t 8 5 sin 8t
er
6.3 Matrix Exponential
m
Matrices can be used in an entirely different manner to solve a system of linear first-order
om

differential equations. Recall that the simple linear first-order differential equation x0 = ax,
where a is constant, has the general solution x = ceat , where c is a constant. It seems natural
then to ask whether we can define a matrix exponential function eAt , where A is a matrix
C

of constants, so that a solution of the linear system X0 = AX is eAt .


or

We shall now see that it is possible to define a matrix exponential eAt so that
tF

X = eAt C (23)
is a solution of the homogeneous system X0 = AX where A is an n × n matrix of constants,
No

and C is an n × 1 column matrix of arbitrary constants. Note in (23) that the matrix C post
multiplies eAt because we want eAt to be an n × n matrix. While the complete development
of the meaning and theory of the matrix exponential would require a thorough knowledge of
matrix algebra, one way of defining eAt is inspired by the power series representation of the
scalar exponential function eat .

(at)2 (at)k
eat = 1 + at + + ··· + + ···
2! k!
2 k X∞ (24)
at 2t kt tk
e = 1 + at + a + ··· + a + ··· = ak
2! k! k=0
k!
The series in (24) converges for all t. Using this series, with 1 replaced by the identity matrix
I and the constant a replaced by an n × n matrix A of constants, we arrive at a definition
for the n × n matrix eAt .
154

Definition 5: Matrix Exponential


For any n × n matrix A,
2 k ∞
X
2t kt tk
eAt
= I + At + A + ··· + A + ··· = Ak (25)
2! k! k=0
k!

It can be shown that the series given in (25) converges to an n × n matrix for every value of
t. Also, A2 = AA, A3 = A(A2 ), and so on.

 
At 1 0
Example 9. Determine e for the matrix A = .
0 2
Solution: From the previous powers,

e
 2     3     4   

Us
2 1 0 1 0 3 1 0 1 0 4 1 0 n 1 0
A = = , A = = , A = ,...,A = ,...,
0 22 0 4 0 23 0 8 0 24 0 2n

l
cia
We see from the definition of matrix exponential that
t2
eAt = I + At + A2 + · · ·
er
   2!     
1 0 1 0 1 0 t2 1 0 tn
m
= + t+ + ··· + + ···
0 1 0 2 0 22 2! 0 2n n!
 
om

t2
1 + t + 2! + · · · 0 
= 2 
2t
C

0 1 + 2t + 2 + ···
2!
or

Using (24), we can identify a = 1 and a = 2. The power series in the first and second rows
of the matrix represent, respectively, et and e2t and so we have
tF

 t 
At e 0
e = .
0 e2t
No

In general, an n × n matrix A s a diagonal matrix if all its entries off the main diagonal
are zero, that is,  
a11 0 . . . 0
 0 a22 . . . 0 
 
A =  .. .. .. 
 . . . 
0 0 ... ann
Hence, if A is an n × n diagonal matrix, it follows that
 
ea11 t 0 ... 0
 0 ea22 t . . . 0 
At  
e =  .. .. .. 
 . . . 
ann t
0 0 ... e
155

The derivative of the matrix exponential is analogous to the differentiation property of the
d at
scalar exponential e = aeat . That is,
dx
 2 k

d At d 2t kt
e = I + At + A + ··· + A + ···
dt dt 2! k!
1
= A + A2 t + A3 t2 + · · ·
 2! 
2
2t
= A I + At + A
2!
= AeAt

Hence, we have

e
Us
d At
e = AeAt (26)
dt
From (26), we can now prove that X = eAt C is a solution of X’=AX for every n × 1 vector

l
C of constants. That is,
d At cia 
er
X0 = e C = AeAt C = A eAt C = AX
dt
m
d At
If we denote the matrix exponential eAt by the symbol Ψ(t), then e = AeAt is equivalent
om

dt
to the matrix differential equation Ψ0 (t) = AΨ(t). In addition, it follows immediately from
the definition of matrix exponential that Ψ(0) = eA0 = I, and so detΨ(0) 6= 0. It turns out
C

that these two properties are sufficient focus to conclude that Ψ(t) is a fundamental matrix
of the system X0 = AX.
or

Computation of eAt using the Laplace Transform


tF

If x(s) = L{X(t)} = L{eAt }, then the Laplace transform of the initial value problem
No

X0 = AX, X(0) = I (27)

is

sx(s) − X(0) = Ax(s) or sI − A x(s) = I (28)
−1 −1 −1
Multiplying (28) by sI − A implies x(s) = sI − A I = sI − A . In other words,
−1  −1
L{eAt } = sI − A or eAt = L−1 sI − A (29)
156
 
At 1 −1
Example 10. Using Laplace transform, compute for e if A =
2 −2
Solution: Compute for the matrix sI − A and find its inverse:
 
s−1 1
sI − A =
−2 s + 2
 
s−1 1
sI − A =
−2 s + 2
   
a11 a12 −1 1 a22 −a12
Recall: Given a 2 × 2 nonsingular matrix A = ,A =
a21 a22 detA −a21 a11
 s+2 −1 
 

e
−1 1 s−1 1  s(s + 1) s(s + 1) 
= 

Us
sI − A = = 
s(s + 1) −2 s + 2 2 s−1 
s(s + 1) s(s + 1)

l
cia
Decompose each entry of the last matrix into partial fractions.
 
2 1 1 1
er
− − +
−1  s s + 1 s s + 1
sI − A = 
m
2 2 1 2 
− − +
s s+1 s s+1
om

It follows from (29) that


C

 
2 1 1 1  
− − + 2 − e −t
−1 + e −t
 s s+1 
s s + 1
or

eAt = L−1 
2 = 
2 1 2  2 − 2e−t
−1 + 2e −t
− − +
tF

s s+1 s s+1
Activity 1
No

Find the general solution of the following sys- Find the solution of the following initial-value
tem. problems.
dx
1. = −4x + 2y 1   
dt 0 0 3
1. X = 2 1 X, X(0) =
dy 5 1 − 2
5
= x + 2y
dt 2    
  0 2 4 −1
0 −1 3 2. X = X, X(0) =
2. X = X −1 6 6
−3 5
     
0 4 −5 0 3 9 2
3. X = X 3. X = X, X(0) =
5 −4 −4 −3 −4
157

References:
1. Rainville, E.D., Bedient, P.E., and Bedient, R.E., Elementary Differential Equations,
8th Edition, Pearson, 1996

2. Zill, D.G., Differential Equations with Modeling Applications 7th Edition, Brooks/Cole,
2000

3. Blanchard P., Devaney, R.L., and Hall, G.R., Brooks/Cole, Differential Equations, 4th
Edition, 2002

4. Zill, D.G. and Cullen, M.R., Differential Equations with Boundary-Value Problem, 7th
Edition, Brooks/Cole, 2009

5. Boyce, W. E. and DiPrima, R.E., Elementary Differential Equations, 10th Edition,

e
Us
John Wiley and Sons Inc., 2012

6. Dawkins, Paul, Differential Equations,


https://tutorial.math.lamar.edu/Classes/DE/DE.aspx

l
cia
7. Shepley L. Ross, Introduction to Ordinary Differential Equations, 3rd Edition, 1996
er
8. Richard Bronson, Differential Equations Schaums Solved Problem Series, 1989
m
9. Berico, E. et al. Differential Equations
om

10. https://www.math.ust.hk>dif...PDF
C

11. https://www.ugrad.math.ubc.ca>cool
or

12. https://www.math24.net>newtonslawofcooling
tF
No

You might also like